Ευκλειδης Β 51

84

Upload: demi-de

Post on 09-Mar-2016

228 views

Category:

Documents


8 download

DESCRIPTION

 

TRANSCRIPT

Page 1: Ευκλειδης Β 51
Page 2: Ευκλειδης Β 51

Νεκτάριος Πρωτοπαπάς

Φυσική Α' Ενιαίου Λυκείου

Κ. Α. Κυριακόπουλος

Μαθηματικά θετικής Κατεύθυνσης, Β' Ενιαίου Λυκείου

ιουοική Γ' Ενιαίου Λvκε!συ

111 τόμος �

Τ ριαντάφυλλος Μελισσαρόπουλος

Φυσική θετικής και Τεχνολογικής Κατεύθυνσης,

Γ' Ενιαίου Λυκείου

Γ. Βιδάλη ς - Β. Γ κιμίσης

Άλγεβρα r ενικής Παιδείας, Β' Ενιαίου Λυκείου

ΜΕΘΟΔΟΛΟΓΙΑ

Φυσικής 3' ε...-ιο.Ο'..J λ""κε!ο� ." 1::W�!

ΙIHHJI(>4 θiC'",ιΙ.. JY-1 λl"CH ο θf�.ι,V�λωlι;Η • ο· · • w ο ·ο • � � • " ·" • • • • ι • • · �ι.

Αντώνης Σαρρηγιάνvης Μεθοδολογία Φυσικής

θετικής και Τεχνολογικής Κατεύθυνσης, Β' Ενιαίου Λυκείου

Τ άσος Βαρδόπουλος

Μεθοδολογία Χημείας θετικής Κατεύθυνσης, Γ' Ενιαίου Λυκείου

Μ. Γεωργάκης, Αθ. Μακρίδης κ.ά.

Ευκλείδεια Γεωμετρία r ενικής Παιδείας, Β' Ενιαίου Λυκείου

χημε},g Β' Ενιαίου Λυκεfου

I. Κεφαλλωvίτης, Β. Χρηστίδης

Χημείο θετικής Κιiτεύθυνσης. Β' Ενιαίου Λυκείου

κριτήρια : αξιολόγησης _ "'ηΦυσική [ Ej

ο. .... ·---··1,01-· ... ; ,.....,.,_ .. .,., •••• -. .... οι-• .. οι 1 _ .... � ... -....... �'"'"""""'""*"'"'(

; , , ,,,,,,,,,.,, •• _ ΝΕΚΤΑΡΙΟ[ ΠΡΟΤοnΑΠΑΣ

Νεκτάριος Πρωτοπαπάς

Κριτήρια αξιολόγησης στη Φυσική

fενικής Παιδείας, Γ' Ενιαίου Λυκείου

Νεκτάριος Πρωτοπαπάς

Φυσική fενικής Παιδείας, Β' Ενιαίου Λυκείου

μαθηματικά r· Ενιαίου Λυκεfου

Ματθαίος Τ σιλπιρίδης

Μαθηματικά f ενικής Παιδείας, Γ' Ενιαlου Λυκείου

Ελευθέριος Πρωτοπαπάς Μαθηματικά

θετικής και Τεχνολογικής Κατεύθuv:- ·

Γ' Ενιαίου Λυκείου

Page 3: Ευκλειδης Β 51

HELLENIC l\1ATHEMATICAL SOCIETY

HELLAS2004

Alίtlli®illl�9 ffiΓP®®CS® dJωΠy

WwW®w ι1w® kWQilJU®U®Q ®U ι1w® MitwuQulfy �a Nαιι1tl®wmU !Eww®αιuti®w & lliwUti�twwι;; �uuαιtiwQ

illww ιtw� MltiιιtiQU!fy wfi ιtwtiιtWll'®

�([)([)�

Page 4: Ευκλειδης Β 51

Αθήνα 4-18 Ιουλίου 2004 Η Ελλάδα το καλοκαίρι του 2 004 εκ τός από τους Ολυμπιακούς Αγώνες φιλοξενεί και τη

Διεθνή Γv!αθημαηκή Ολυμπιάδα, αναδεικνύοντας έτσι την διαχρονικότητα του αρχαιοελληνικο ιδεώδους, της ισόρροπης δηλαδή ανάπτυξης της πνευματικής δύναμης και της σωματικής ρώμης. Η Διεθνής Μαθηματική Ολυμπιάδα� Δ.Μ.Ο., ένας θεσμός υψηλότατου ενδιαφέροντος, πραγματική πρόκληση για τη διοργανώτρια χώρα, που πραγματοποιείται για 45η συνεχή χρονιά θα φιλοξενηθεί εδώ, τον ΙοίJλιο. Και όλοι θα είμαστε εκεί. . . Στην Αθήνα και τους Δελφούς . . . Μαζί με τους δεκάδες μαθητές από όλον τον κόσμο, οι οποίοι μαγεμένοι από τη γοητεία των Μαθηματικών έχουν για μέρες, μήνες ή χρόνια δουλέψει σκληρά πάνω στη λύση δύσκολων μαθηματικών προβλημάτων με στόχο τη συμμετοχή τους στη Διεθν11 Μαθημαηκή Ολυμιηό:δυ.

Η Ολυμπιάδα θα πραγματοποιηθεί από 4 - 18 Ιουλίου. Οι ξένοι μαθητές θα έρθουν στις 9 Ιουλίου. και θα φύγουν στις 18 Ιουλίου Η χώρα μας θα φιλοξενήσει 90 περίπου αντιπροσωπίa;

από διαφορετικές χώρες από όλο τον πλανήτη. Κάθε αντιπροσωπία θα αποτελείται από 6 μαθητές συνοδευόμενους από τον αρχηγό τους και τον υπαρχηγό τους. Επίσης θα υπάρχει διεθνής επιτροπή επιλογής θεμάτων και εξαίρετες προσωπικότητες της Μαθηματικής Επιστήμης μέλη των επιτροπών και η εννεαμελής Διεθνής Επιτροπή των Μαθηματικών Ολυμπιάδων. Τα θέματα, τα οποία είναι πρωτότυπά, είναι κοινά για όλα τα παιδιά και θα μεταφραστούν σε 56 γλώσσες, επειδή κάθε παιδί διαγωνίζεται στην γλώσσα του. Το «εξεταστικό)) τμήμα της Ολυμπιάδας διαρκεί ουσιαστικά 2 ημέρες από 4,5 ώρες και το υπόλοιπο τμήμα της είναι το πολιτιστικό πρόγραμμα. Ελπίζουμε ότι από τα 800 περίπου μέλη των ξένων αποστολών και επιτροπών θα προκύψουν αρκετοί «πρεσβευτές» της χώρας μας στις πατρίδες τους.

Η Ελλάδα συμμετέχει στις Διεθνείς Μαθηματικές Ολυμπιάδες από το 1974. Από την 24" Διεθνή Μαθηματική Ολυμπιάδα του Παρισιού ή χώρα μας έχει μια επιτυχή πορεία, με ικανοποιητική συγκομιδή μεταλλίων και υψηλή θέση κατάταξης. Αξιοσημείωτο είναι ότι το 199 4 στ ο Χονγκ Κονγκ και το 1996 στην Ινδία, η χώρα μας ήλθε πρώτη στον κόσμο στη ΓεωμΗρία.

Η Διεθνής Μαθηματική Ολυμπιάδα (fnternational Mat!1ematical Ol�'mpiad.· l_\!0 2004) αποτελεί μια πολύ καλή ευκαιρία να συνδυαστεί το σύγχρονο υψηλό μαθηματικό επίπεδο της χώρας με την αρχαιοελληνική μαθηματική κληρονομιά μας. Στην αρχή του 2lou αιώνα είναι περισσότερο επιβεβλημένη από ποτέ η προώθηση υψηλών προτύπων, από το χώρο του πνεύματος

και της επιστήμης. Η επιστημονικ�1 αρηότητα της Ελληνικής Μαθηματικής Εταιρείας, η εμπειρία που διαθέτa

από τη διοργάνωση Μαθηματικών Διαγωνισμών σε διεθνές επίπεδο, οι καλές σχέσεις που έχa αναπτύξει με τη Διεθνή Εκπαιδευτική και Μαθηματική Κοινότητα, αποτελούν εγγύηση για την άρτια προετοιμασία των Ελληνικών Ολυμπιακών Ομάδων και την άψογη διοργάνωση της ΙΜΟ 2004. Από το 1997, η ΕΜΕ άρχισε την ενημέρωση και προετοιμασία της Διεθνούς Μαθηματικής Κοινότητας για τη διεκδίκηση της διοργάνωσης της Διεθνούς Μαθηματικής Ολυμπιάδας 2004. Την επιτυχή κατάληξη της τελικής διεκδίκησης - που έγινε το 1999 στο Βουκουρέσu - και r οποία αποδεικνύει το διεθνές κύρος της Εταιρείας μας, ακολούθησε ένας κύκλος δραστηριοτήτων με σκοπό την προβολή και περαιτέρω προετοιμασία του όλου εγχειρήματος.

Η 45η Διεθνής Μαθηματική Ολυμπιάδα είναι μια εθνική προσπάθεια. Το τεράστιο αυτό εγχείρημα απαιτεί την ενθουσιώδη συμμετοχή όλων μας, στο Σ<δμα Εθελοντό>ν της Μαθηματικής Ολυμπιάδας 2004, που θα συσταθεί από την Ελληνική Μαθηματική Εταιρεία. Απαραίτητο κριτήριο συμμετοχής είναι η άριστη γνώση της Αγγλικής γλώσσας . Η επιλογ ή θο γίνει με προσωπική συνέντευξη και στη συνέχεια οι εθελοντές και οι εθελόντριες θα εκπαιδευθούν κατάλληλα ώστε να αποτελέσουν τον υποστηρικτικό ιστό της Μαθηματικής Ολυμπιάδας το 200 . Μετά το τέλος της Ολυμπιάδας θα τους δοΟεί επίσημη βεβαίωση συμμετοχής τους ως εθελοντές, θα εγγραφούν, τιμής ένεκεν, πάρεδρα μέλη της Ελληνικ1Ίς ΜαθηματικΊΊς Εταφsiας και θα πάρουν δωpεάν εκδόσεις τη� Ε. Μ. Ε ..

Page 5: Ευκλειδης Β 51

Υ πcύ8υνοι Έκδοσης Ευσταθίου Ευάγγελος, Μπαραλής Γεώργιος.

Συνιακηκή Ομάδα: Ανδρουλακόκης Νίκος Βακαλόπουλος Κώστας Βισκαδουρόκης Βασίλης Βλάχου Αγyελιχή Γιούσης θεόδωρος Γράψ05 Κωνσταντίνος Δούν<ψης Αντώνης Ευθτη.ιογλου Πέψος θεοδωρέλος Χριστος Καλίκας Σταμάτης Καρακάσοyλου Αλέξανδρος Καρακατσανης Βασίλης Καρκάνης Βασίλης Κατσούλης Γιώργος Κερασαρίδης Γιάννης ΚΊΊπουρός Χρήστος Κόντζιας Νίκος Κυριακόπουλος Αντώνης Κυριακόπουλος θανάσης Κυριακοπούλου-Κυβερνήτου Χρυστ. Λαι,αρίδης Χρήστος Λουρίδας Σωιήρης Μαλαφέκας θανάσης ΜαλιδώαΊ Ελένη Μεταξάς Νίκος Μυλωνάς Δημήτρης Μώκος Χρήστος Ντζι�' ας Ηλίας Παπ ιάν Μαριλένα Πωm ς Γιώργος Ρήγας θεόδω_ρος Σ�ελλάpης Βασίλης Σαϊ�Ευα Σταθόπουλος Γιώργος Στάϊκος Κωνσταντίνος Στάϊκος Παναγιώτης Στρατής Ιωάννης Ταπεινός Νικόλαος Τασσόπουλος Γιώργος Τριάντος Γεώργιος Τσικαλουδ�ς Γιώργος Τσιούp:ας θανάσης Τυρλής ΙωάνVης ΧαραλαμπίδηςΓιώργος Χαραλαμποπούλου Λίνα Χαραλάμπους θάνος Χριστόπουλος Παναγιώτης

Συνερyάιες: Φελλούρης Ανόργuρος, Πάμφιλος Πόρης, Μ Στολάκης, Μπ6pπης Τοuμούνης, Μ Παπαδόπουλος, Αντώνης Τσιπρόπουλος, Ν. Ζανταρίδης, Κ Παπαδόπουλος, Χαρού­λα Σαραφοπούλου, Τριαντάφυλλος Τρια­νταφύλλου,

2.. 1:<:..\Vi, ΊΚΗ2.. \ 1\ 11 ..::F \11' ' Σ.

ΠΑl\'ΕΓΠΣΊΉΜΙΟΥ 34 - 106 79 ΑΘΗΝΑ Τηλ; 210-36 17 784, 210-36 16 532 Fa.x: 210-36 41 025 Εκδόιης: Αλε(αvδρής Νικόλαος Διcυθυνιής: Κ:αρκούλιας Γιώργος

ISSN: 1105- 7998 ΣΥΝΔΡΟΜΕΣ: Τεύχο� 2,50 ευρώ Ετήσια συνδρομή 10,00 ευρώ (Σχολ.) Σuvδρο_ι.τητέ� 10,00+2,00 ευρώ (Ταχ.) Οργανισμοί: 18,00 ευρώ Ταχ. Επιιαyές Τ. Γραφcίο Αι.Ιήvα 54, Τ.Θ. 30044

ΕΛΛΗΝΙΚΗ ΜΑΘΗΜΑΤΙΚΗ ETAIPEIA Τεύχος 51 • Ιανουάριος • Φεβρουάριοι; • Μάρτιοι; 2004 • Ευρώ: 2,50

e-mail: [email protected] www.hms.gr

ΜΑΘΗΜΑτΙΚΟ ΠΕΡΙΟΔΙΚΟ ΠΑ ΤΟ ΛΥΚΕΙΟ

ΠΕΡΙΕΧΟΜΕΝΑ 4 ΤΟ ΒΗΜΑ ΤΟΥ ΕΥΚΛΕΙΔΗ

14 Ο Ευκλείδη!; προιείνει ... Ευκλείδη και ... Διόφαvιο

33 ΗΟΜΟ MATHEMAτiCUS 36 «Ολυμπιακέ!; προσεγγίσει!;>>

I Μα8ημαιικά για ιφ· Α' τάξη ιου Λυκείου I

39 Γραμμικά συσιήμαια εξισώσεων α' 6α8μού

45 Μια περιήγηση σιο χώρο ιων παραλληλογράμμωΥ · ιραπcζίωΥ

I Μα8ημαηκά για ιην Β' Τάξη ιου Λυκείου I

51 Εκ8ειική και λογαρι8μική συνάριηση

54 Κανονικά πολύγωνα - Μέιρηση κύκλου

58 Θεωρία αρι8μών

62 Κωνικέ!; ιομέ!;

67 Η σιήλη ιου μα8ηιή

Μα8ημαηκά για την Γ' Τάξη ιου Λυκείου

68 Απόδειξη ανισοιικών σχέσεων και επίλυση!; ανισώσεων

71 Πι8ανόιψε!;

75 Θεώρημα μέση!; ιιμή!;

80 21° Πανελλήνιο Συνέδριο

ΣιοιχcιοιJcοία - Σελιδοποίηση Ν. Αλεξόπουλος Ακαδημίας 43, τηλ 210-3606826

Εκτύπωση ΙΝΤΕΡΠΡΕΣ Α.Ε., Ιερά οδός 81 - 83

ΥπευΟ. Τυπογραφείοu Γl Τρtκεριώτης·Τηλ:3474654

Page 6: Ευκλειδης Β 51

π I

Δυναμική Γεωμετρία και EucliDraw Ένα σύντομο μάθημα, με αφορμή την κυκλοφορία ενός ελληνικού προγράμματος

Πάρης Πάμφιλος Αναπλ. Καθηγητής - Μαθηματικό Τμήμα Παν. Κρήτης

Με αφορμή το γνωστό πρόβλημα του Feπnat (1601-1665), θα Α κάνω μερικά σχόλια γιά την χρήση της Δυναμικής Γεωμετρίας και το αντίστοιχο νέο ελληνικό πρόγραμμα EucliDraw. Θα ξεκινήσω από το πρόβλημα και έναν από τους πολλούς τρόπους ανάλυσής του.

Ας υποθέσουμε ότι έχουμε τρεις πόλεις στις κορυφές ενός οξυγωνίου τριγώνου ΑΒΓ και ψάχνουμε την θέση σημείου Ρ, έτσι ώστε το άθροισμα των αποστάσεων λ = ΡΑ+ΡΒ+ΡΓ να είναι το ελάχιστο δυνατόν.

σyήμα-1

Γ

Το πρόβλημα θα μπορούσε να είναι ρεαλιστικό και να αφορά την εγκατάσταση ενός αεροδρομίου ή εργοστασίου που εξυπηρετεί τρεις πόλεις.

Α Μεθοδολογικά, το πρόβλημα παρουσιάζει μια δυσκολία που οφείλεται στο ότι τα τμήματα των οποίων το άθροισμα θέλουμε να

Ρ

Β σχήμα-2

Γ

ελαχιστοποιήσουμε, δεν είναι διαδοχικά μίας τεθλασμένης γραμμής αλλά έχουν την ειδική διάταξη του σχήματος-Ι. Αν είχαν την διάταξη του σχήματος-2 και μάλιστα ένωναν δύο σταθερά σε θέση σημεία Α, Β, τότε θα ξέραμε ότι το ελάχιστο δυνατόν συνολικό μήκος θα ήταν το μήκος του ευθυγράμμου τμήματος ΑΒ. Το ευτύχημα είναι ότι μπορούμε να πετύχουμε μιαν αναγωγή στην κατάσταση του σχήματος-2.

Η μέθοδος είναι γνωστή από παλιά (Feπnat). Διαλέγουμε μία κορυφή, π.χ. την Α και περιστρέφουμε το τρίγωνο ΑΡΓ, περί το Α, κατά 60 μοίρες, ώστε να έλθει στην θέση ΑΡ 'Β' . Δημιουργούνται τότε τα ισόπλευρα τρίγωνα ΑΡΡ' και ΑΓΒ'. Το άθροισμα λ ισούται πλέον με το λ = ΑΡ+ΒΡ+ΓΡ = ΒΡ+ΡΡ'+Ρ'Β' . Η θέση του Β' είναι σταθερή και συμπίπτει με την κορυφή του ισοπλεύρου ΑΓΒ '. Συνεπώς αν θέλουμε να ελαχιστοποιήσουμε την τιμή του λ, πρέπει το λ να συμπίπτει με το μήκος του ευθυγράμμου τμήματος ΒΒ' και το Ρ να ευρίσκεται πάνω στην ΒΒ ' . Γιατί

ΕΥΚΛΕΙΔΗ Β' λζ' τ.3/4

σχήμα-3

Page 7: Ευκλειδης Β 51

Το βήμα του Ευκλείδη

όμως να προτιμήσομε την κορυφή Α; Θα μπορούσαμε να σκεφτούμε παρόμοια περιστρέφοντας περί το Β ή Γ το τρίγωνο ΒΡ Α ή ΓΡ Α αντίστοιχα. Θα καταλήγαμε τότε στο ότι το Ρ θα έπρεπε να περιέχεται επί αντιστοίχων ευθειών ΑΑ ', ΓΓ, που ενώνουν τις κορυφές του ΑΒΓ με τις απέναντι κορυφές των ισοπλεύρων, που κατασκευάζονται στις πλευρές του ΑΒΓ.

Το σημείο Ρ, εάν υπάρχει, θα πρέπει λοιπόν να περιέχεται και στις τρεις αυτές ευθείες, οι οποίες, συνεπώς θα τέμνονται (στο ζητούμενο Ρ). Επίσης θα πρέπει να είναι ίσες μεταξύ τους, αφού κάθε μία θα υλοποιεί το ελάχιστο ζητούμενο άθροισμα λ. Για ν' αποδείξουμε λοιπόν την ύπαρξη του Ρ, πρέπει να δούμε ότι οι τρεις αυτές ευθείες ΑΑ ', ΒΒ', ΓΓ διέρχονται από κοινό σημείο Ρ.

Δεν θα προχωρήσω αμέσως στην απόδειξη. Θα παρεμβάλλω κάποια ζητήματα διδακτικής: Πόσο γρήγορα και αποτελεσματικά μπορεί να χειρισθεί ο δάσκαλος ένα παρόμοιο θέμα στην τάξη; Πόση προετοιμασία

Γ'

Α' σχήμα-4

πρέπει να κάνει για το θέμα; Πόσο χρόνο να ασχοληθεί με το σχήμα και πόσο χρόνο με τον συλλογισμό; Μπορεί να διερευνήσει το θέμα με κάποιο είδος πειράματος;

Όλα τα προηγούμενα διδακτικά θέματα συνδέονται με ένα σύγχρονο μέσον διδασκαλίας, που τείνει πλέον να καθιερωθεί παγκοσμίως. Την Δυναμική Γεωμετρία. Πρόκειται για ένα εποπτικό μέσον διδασκαλίας με την βοήθεια του υπολογιστή και καταλλήλου προγράμματος. Το πρόγραμμα λειτουργεί σε γραφικό περιβάλλον και επιτρέπει τον σχεδιασμό σχημάτων με την βοήθεια του «ποντικού>>. Πέρα από τον σχεδιασμό όμως το πρόγραμμα επιτρέπει την ιεραρχική δόμηση του σχήματος. Η «δυναμική» επιτρέπει να μεταβάλλουμε το σχήμα, π.χ. την μορφή του ΑΒΓ, χωρίς να αλλοιώνονται οι γεωμετρικές σχέσεις. Π.χ. στο προηγούμενο σχήμα, παρατηρούμε στην οθόνη ότι ανεξάρτητα του σχήματος του βασικού τριγώνου ΑΒΓ, τα ισόπλευρα επανασχεδιάζονται σωστά στις πλευρές του και οι ευθείες ΑΑ ',

ΒΒ' , ΓΓ, παρόλο που αλλάζουν θέση εξακολουθούν να διέρχονται από ένα σημείο Ρ. Έτσι και εάν ακόμη δεν έτυχε να γνωρίζομε την ιδιότητα αυτή, η δυναμική γεωμετρία μας επιτρέπει να την διαπιστώσουμε ή παρατηρήσουμε πειραματικά. Τουλάχιστον

-οπτικά. Βέβαια αυτό δεν είναι απόδειξη,

αλλά είναι κίνητρο για απόδειξη. Και τα κίνητρα και η δημιουργία τους είναι , κατά την γνώμη μου, από τις σημαντικότερες διαδικασίες μέσα στην τάξη.

Τα τελευταία χρόνια παρατηρείται μια ταχύρυθμη ανάπτυξη λογισμικού δυναμικής Γεωμετρίας. Στην Ελλάδα έγιναν προσπάθειες να εισαχθεί στα σχολεία αντίστοιχο λογισμικό και καταναλώθηκαν αρκετοί πόροι και εργασία, ώστε να γίνουν μεταφράσεις ξένων προγραμμάτων και αντίστοιχη εκπαίδευση σε επιλεγμένα προγράμματα.

ΕΥΚΛΕΙΔΗ Β' λζ' τ.3/5

Page 8: Ευκλειδης Β 51

Το βήμα του Ευκλείδη

Προσφάτως κυκλοφόρησε στην Ελλάδα και την Κύπρο (Βιβλιοπωλεία Παπασωτηρίου) και το αντίστοιχο ελληνικό πρόγραμμα 'ΈucliDraw'Ό Το πρόγραμμα ξεκίνησε το 1990 και αναπτύχθηκε στο Πανεπιστήμιο Κρήτης. Με την ιδιότητα ενός από τους συντελεστές, γνώστες και χρήστες του προγράμματος, θα σταθώ λίγο περισσότερο σ' αυτό.

Το πρόγραμμα διακρίνεται για κάποια χαρακτηριστικά, τα οποία, πιστεύω, το καθιστούν ιδιαίτερα παραγωγικό. Τα χαρακτηριστικά αυτά επικεντρώνονται σε ορισμένους aυτοματισμούς που εξοικονομούν τον χρόνο του δασκάλου, τόσο κατά την προετοιμασία, όσο και κατά την παρουσίαση μέσα στην τάξη. Είναι σημαντικό για τον δάσκαλο, πέρα από το γνωστικό φορτίο, να μην επιβαρύνεται με την πολυπλοκότητα ενός δύσχρηστου εργαλείου. Επίσης είναι σημαντικό ο σχεδιασμός την ώρα του μαθήματος και ο πειραματισμός με δυναμική μεταβολή των σχημάτων να μην τρώει χρόνο από την ουσία του μαθήματος. Το πρόγραμμα είναι ένα βοήθημα, ένα εργαλείο, ένα σύγχρονο είδος χαρτιού, κανόνα και διαβήτη. Και δεν επιτρέπεται, νομίζω, να υποβαθμίσει ή να υποκαταστήσει τον ρόλο του δασκάλου. Ο δάσκαλος είναι αναντικατάστατος, ο χρόνος του είναι πολύτιμος, ο χρόνος του μαθήματος περιορισμένος. Αυτοί που φτιάχνουν το εργαλείο θα πρέπει να αναλώσουν δυνάμεις και χρόνο, ώστε την ώρα της δράσης οι κατασκευές να εξελίσσονται γρήγορα και με ακρίβεια. Κατά την μακρόχρονη ανάπτυξη του προγράμματος ελήφθη ιδιαίτερη μέριμνα γι' αυτό. Έτσι, για παράδειγμα, το σχήμα-4, μπορεί να γίνει μέσα στην τάξη, χωρίς καμία προετοιμασία, σε 8 κινήσεις: 2 για το βασικό τρίγωνο ΑΒΓ, 3 για τα τρία ισόπλευρα, και 3 για τα ευθύγραμμα τμήματα ΑΑ ', ΒΒ ', ΓΓ. Οι διασυνδέσεις και η ιεραρχική οργάνωση του σχήματος γίνονται εσωτερικά «διαβάζοντας», κατά κάποιο τρόπο, τις κινήσεις του χρήστη. Μετά την κατασκευή, μπορεί κανείς να μεταβάλλει το σχήμα του τριγώνου χωρίς να αλλοιώνει τις γεωμετρικές σχέσεις.

Ένα άλλο προτέρημα του προγράμματος είναι ότι συμπεριλαμβάνει, εκτός του σχεδιαστικού και

Α: σχήμα-5

Β'

ένα δεύτερο περιβάλλον προγραμματισμού. Εδώ βέβαια έχουμε μια δεύτερη φάση της χρήσης του προγράμματος, για πεπειραμένους: Την παραγωγή ειδικών εργαλείων, κατά την φαντασία και επινοητικότητα του χρήστη.

Τέλος θα μπορούσα να προσθέσω και ένα ακόμη προτέρημα, ότι το πρόγραμμα είναι ελληνικό, αναπτύσσεται από ομάδα ταλαντούχων ελλήνων επιστημόνων και εξελίσσεται εμπλουτιζόμενο συνεχώς με νέα εργαλεία. Συνεπώς μπορεί, κατά την γνώμη μου, να προσαρμοσθεί στις ιδιαιτερότητες και την παράδοση της ελληνικής μαθηματικής κοινότητας, να αφομοιώσει προτάσεις της και υποδείξεις και να καλύψει με αποτελεσματικότητα και ευαισθησία τις ανάγκες που θα εκφρασθούν.

Ας γυρίσουμε όμως στην απόδειξη που αφήσαμε στην μέση. Να κλείσουμε με λίγη από την ομορφιά του «σχήματος» που δεν είναι τίποτε άλλο παρά το αντίθετο του «άσχημοω>.

ΕΥΚΛΕΙΔΗ Β' λζ' τ.3/6

Page 9: Ευκλειδης Β 51

Το βήμα του Ευκλείδη

Η ισότητα των ΑΑ ', ΒΒ' οφείλεται στο γεγονός ότι το τρίγωνο ΒΒΤ προκύπτει από το ΑΑΤ μέσω περιστροφής του τελευταίου περί το Γ, κατά γωνία 60 μοιρών. Πράγματι, εκ κατασκευής, στα τρίγωνα ΑΑΤ και ΒΒΤ έχουμε: ΑΤ=ΒΓ, ΑΓ=ΒΤ και η περιεχόμενη γωνία <)ΑΤΑ = γ+60 = <)ΒΓΒ'. Επομένως, στο Ρ, λόγω της στροφής, η γωνία θα είναι 60 μοιρών.

σχήμα-6 σχήμα-7

Συνέπεια άμεση είναι ότι τα Ρ και Γ «βλέπουν» την ΑΒ' υπό γωνίες ίσες (60 μοιρών), άρα το ΑΒΤΡ είναι εγγράψιμο σε κύκλο και συνεπώς η ΑΡΓ, ως απέναντι της ΑΒΤ είναι 120 μοιρων, άρα η γωνία Β 'ΡΓ είναι και αυτή 60 μοιρών. Αμέσως βλέπουμε λοιπόν ότι αν ενώσουμε το Ρ με τα Γ και Γ, τότε όλες οι γωνιές περί το Ρ είναι 60 μοιρών, πράγμα που

ολοκληρώνει τ� θέμα μας. Τα Γ, Ρ, Γ είναι επί ευθείας, οι ΑΑ ', ΒΒ', ΓΓ διέρχονται όλες απ' το Ρ, και είναι ίσες μεταξύ τους και επιπλέον το Ρ «βλέπει» κάθε πλευρά του οξυγωνίου τριγώνου ΑΒΓ υπό γωνία 120 μοιρών.

Το σημείο «του Fennat» Ρ, του τριγώνου είναι ένα από τα εκατοντάδες αξιόλογα «κέντρα του τριγώνου)). Το σημείο αυτό κατασκευάζεται με τον προηγούμενο τρόπο ακόμη και στην περίπτωση αμβλυγωνίων τριγώνων, που όμως η αμβλεία γωνία είναι μικρότερη των 120 μοιρών. Αυτό φαίνεται πειραματικά ( σχήμα-6) και από την δυναμική μεταβολή του τριγώνου. Αν η αμβλεία ξεπεράσει τις 120 μοίρες τότε έχουμε μία κατάσταση όπως αυτή του σχήματος-7.

Γ'

Β' z

�Ε Η

Α'

σχήμα-8

Μιά άλλη κατεύθυνση εφαρμογής της Δυναμικής Γεωμετρίας είναι η «γενίκευση)) . Π.χ. θα μπορούσαμε να αντικαταστήσουμε το ισόπλευρο τρίγωνο με ένα αυθαίρετο τρίγωνο και να θέσουμε το επόμενο πρόβλημα.

Δίδονται δύο τρίγωνα ΑΒΓ και ΕΖΗ. Στις πλευρές του πρώτου κατασκευάζουμε όμοια προς το δεύτερο τρίγωνα, aντιστοιχώντας πάντοτε την ίδια πλευρά και προσανατολισμό. Κατόπιν θεωρούμε τις ευθείες ΑΑ ', ΒΒ', ΓΓ, που ενώνουν απέναντι κορυφές. Για ποία τρίγωνα ΕΖΗ, οι ευθείες αυτές διέρχονται από το ίδιο σημείο;

Με την βοήθεια της δυναμικής γεωμετρίας, μπορούμε να «δούμε)) ότι κάτι τέτοιο συμβαίνει, ακριβώς τότε, όταν οι πλευρές ΕΖ = ΕΗ. Δηλαδή το τρίγωνο ΕΖΗ είναι ισοσκελές και τοποθετήται (καθ' ομοιότητα) με την βάση του πάνω στις πλευρές του ΑΒΓ.

ΕΥΚΛΕΙΔΗ Β' λζ' τ.3Π

Page 10: Ευκλειδης Β 51

Το βήμα του Ευκλείδη

Δημιουργείται αμέσως το ερώτημα: εάν για κάθε ισοσκελές, κάνω την προηγούμενη κατασκευή και βρώ το κοινό σημείο Ρ των ΑΑ ' , ΒΒ ' , ΓΓ' , ποίος είναι ο γεωμετρικός τόπος του Ρ;

Και πάλι μπορεί κανείς να «δεί>) την απάντηση. Τουλάχιστον το EucliDraw έχει αντίστοιχο εργαλείο για γεωμετρικούς τόπους, ώστε με λίγες κινήσεις να δει κανείς ότι το Ρ κινείται πάνω σε μια ορθογώνια υπερβολή, γνωστή ως υπερβολή του Κiepert του τριγώνου ΑΒΓ.

Δυστυχώς το χαρτί, που μεγαλώσαμε μαζί του, τόσο φιλικό και οικείο, δεν μας δίδει την δυνατότητα να «ζωντανέψουμε» τα προηγούμενα σχήματα. Να τα μεταβάλλουμε δυναμικά και να διαπιστώσουμε πειραματικά την αλήθεια των ισχυρισμών μας. Το ελληνικό πρόγραμμα δυναμικής γεωμετρίας «EucliDraW)) καλύπτει αυτήν την αδυναμία.

Ο ενδιαφερόμενος μπορεί να κατεβάσει ελεύθερα στον υπολογιστή του την δοκιμαστική έκδοσή του προγράμματος, από την διεύθυνση http://www.euclidraw.com και να πειραματιστεί μόνος του, στον μαγικό κόσμο των σχημάτων

σχήμα-9

και της δυναμικής τους, για όσο χρονικό διάστημα θέλει. Στην ίδια διεύθυνση μπορεί ο ενδιαφερόμενος να απευθυνθεί για βοήθεια, τόσο στο πρόγραμμα, όσο και σε γεωμετρικά θέματα γενικότερα. Η ομάδα του EucliDraw φιλοδοξεί να συμβάλλει σε μια αναζωπύρωση του ενδιαφέροντος για την γεωμετρία, πιστεύοντας ότι αυτή αποτελεί μια από τις κορυφαίες εκδηλώσεις του ελληνικού πνεύματος και πολιτισμού.

Τρεις λύσεις σε ένα πρόβλημα και η ΓΕΝΙΚΕΥΣΗ του.

Πρόβλημα Δίνονται οι κύκλοι (Κ, r) και (Λ, R) έτσι ώστε

οι κοινές εσωτερικές εφαπτόμενες να τέμνονται κάθετα.

Να αποδειχθεί ότι το εμβαδόν του τριγώνου που σχηματίζεται από τις δύο εσωτερικές εφαπτόμενες και μια από τις κοινές εξωτερικές εφαπτόμενες είναι:

E=R·r .

ι η Λύση Έχουμε ΒΔ = ΒΕ = χ και ΓΖ= ΓΗ= y .

-

Επίσης ΔΚΕ = ΑΒΓ = ω= 2 · ΒΚΔ τότε ω

ΑΒ= χ+ r= r·εφ-+r και 2

ΑΓ=y+R=Rεφφ

+R . 2

Μ. Στολάκης

Άρα (ΑΒΓ)=�(rεφ� +r)(R·εφ� +R) ή

(ΑΒΓ)= �R· r( 1+ εφ � ) ( 1+ εφ �) (1).

ω φ εφ-+εφ-

Τώρα: 1=εφ450=εφω+φ

= 2 2 2 1-εφ

ω ·εφ�

2 2

ή 1=εφ� +εφ� +εφ�·εφ� (2)

ΕΥΚΛΕΙΔΗ Β' λζ' τ.3/8

Page 11: Ευκλειδης Β 51

Το βήμα του Ευκλείδη

Η ( 1 ) γράφεται: (ΑΒΓ) = _!_R · r ( 1 + εφ ω + εφ φ + εφ ω · εφ φ ) και 2 2 2 2 2 από (2) θα έχουμε:

(ABΓ) = _!_R · r · 2 ή (ABΓ) = R · r . 2 2η Λύση Ο κύκλος (Λ, R) είναι παρεγγεγραμμένος του

τριγώνου ΑΒΓ . Άρα (ΑΒΓ) = ρβ(τ - β) ή

(ΑΒΓ) =R( α+�+γ β) (βλ. ΓΕΩΜΕΤΡΙΑ Β ' Λ ΥΚΕΙΟΥ) Ομοίως για τον κύκλο

(Κ, r) : (ΑΒΓ) = r ( α +�+ γ - γ) . Πολλαπλασιάζοντας τις δύο ισότητες έχουμε: (ΑΒΓ)2 = R · r ( α +;- β ) - ( α +�- γ ) =

R · r[α + (γ - β)] [α - (γ - β)] 4

ή (ΑΒΓ)2 = R · r(αz - γ2 - β2 + 2βγ) . 4

Όμως (Πυθαγόρειο Θεώρημα).

Άρα (ΑΒΓ)2 = R · r · βγ ή 2 (ΑΒΓ)2 = R · r · (ΑΒΓ) . Τέλος (ABΓ) = R · r .

3η Λύση Έχουμε ΒΔ = ΒΕ = χ και ΓΖ = ΓΗ = y (ως

εφαπτόμενα τμήματα). Επίσης: ΓΕ = ΓΜ και ΒΗ = ΒΝ ή !α + χ = y + R + r

α + y = x + r + R Προσθέτοντας κατά μέλη

2α = 2(R + r) ή α = R + r ( 1 ). έχουμε

Τώρα το τετράπλευρο ΚΛΗΕ είναι ορθογώνιο τραπέζιο με βάσεις R, r ενώ το ΑΗ' που είναι το ύψος του τριγώνου ΑΒΓ θα είναι παράλληλο προς τις βάσεις.

r · ΑΛ + R · ΑΚ Άρα θα έχουμε ΑΗ' = όπου ΑΚ + ΑΛ

ΑΚ = r · .J2 και ΑΛ = R · .J2 .

Σ , ΑΗ' r · R.fi + R · r · .fi , υνεπως = r;; r;; η R · ν2 + r · ν2

ΑΗ' = 2R · r (2) . R + r Τέλος από ( 1 ) και (2) έχουμε

1 2Rr (ΑΒΓ) = - · (R + r) · -- = R · r . 2 R + r Γενίκευση του προβλήματος Δίνονται οι κύκλοι (Κ, r) και (Λ, R) όπου οι

εσωτερικές εφαπτόμενες σχηματίζουν μια γωνία

«μέτρου ω».

Να αποδειχθεί ότι το εμβαδόν του τριγώνου

που σχηματίζεται από τις δύο εσωτερικές

εφαπτόμενες και μια από τις δύο κοινές

εξωτερικές εφαπτόμενες είναι (ΑΒΓ) = R · �. 2

Λύση Είναι παρόμοια με την 3η Λύση. Πράγματι έχουμε: ΒΓ = α, Γ Δ = ΓΕ = χ και

ΒΜ = ΒΝ = y. Επίσης ΒΕ = ΒΖ και ΓΝ = ΓΤ ( 1 ) και ΑΜΤ = ΑΤ, ΑΔ = ΑΖ (2) .

Οι σχέσεις ( 1 ) γράφονται !α + χ = y + ΑΜ + ΑΖ όπου α = ΒΓ. α + y = χ + ΑΔ + ΑΤ Με πρόσθεση κατά μέλη έχουμε:

2α = 2 · ΑΜ + 2 · ΑΔ (λόγω της (2)) ' ΑΜ ΑΔ' R ω ω η α = + η α = σφ- + rσφ- .

Τέλος α = (R + r) · σφ ω (3) 2

2 2

Τ ώρα το τετράπλευρο ΚΑΝΕ είναι ορθογώνιο τραπέζιο και ΑΗ' το ύψος του τριγώνου ΑΒΓ. Παράλληλο με τις βάσεις R, r όπου: ΚΑ = _r­ω ημ-2

R και ΑΔ =-- . ω ημ-2 ΕΥΚΛΕΙΔΗ Β ' λζ' τ.3/9

Page 12: Ευκλειδης Β 51

Το βήμα του Ευκλείδη

Από ιδιότητα τραπεζίου έχουμε: ΑΗ ' = ΚΕ · ΑΛ+ ΛΝ· ΑΚ ή

ΚΑ + ΑΛ R r r · -- + R · --ω ω ημ- ημ-

ΑΗ ' = ----�2 ______ �2� r R ή AH' = 2 · Rr (4). R + r

-- + --ω ω ημ- ημ-2 2

Τώρα από (3) και (4) έχουμε: ω (R + r)σφ2 2 · Rr (ΑΒΓ) ·-- / 2 R + r

Τέλος (ΑΒΓ) = R · r · σφ ω . 2 Ειδική περίπτωση Αν ω = 90° (δηλ. το αρχικό πρόβλημα) τότε

90° (ΑΒΓ) = R · r · σφ- = R · r . 2

r "'Ένας διάλογος για Μαθηματικά και ... Λογική Παραθέτουμε μία επιστολή του συναδέλφου Μπάμπη Τουμάση και στη συνέχεια απάντηση από το

συνάδελφο Αντώνη Κυριακόπουλο.

ΔΕΝ XPEIAZET ΑΙ ΤΟ ΑΝτΙΣΤΡΟΦΟ

Μπάμπης Τουμάσης

Στο τεύχος 48 του Ευκλείδη Β' στις επαναληπτικές ασκήσει<:;, πpοτείνονταν 11 άσκηση: Να βρείτε τις συναρτήσεις f, οι οποίες είναι

ορισμένες και συνεχείς στο R και για τις οποίες

για κάθε χ ε IR , ισχύει: χ f(x) = χ2 + J e-1f(x- t)dt ο

(άσκηση 11 σελ. 73).

Στη λύση ακολουθείται η εξής πορεία:

(1)

Θέτοντας ω = χ - t η ( 1) μετασχηματίζεται στην χ

f(x) = x2 - e-xJ eωf(ω)dω (2) . ο

Παραγωγίζοντας τη (2) έχουμε τελικά χ3

f(x) = -- + x2 + c (3). 3

Από την (3) παίρνουμε f(O) = c και από την

(1) έχουμε f(O) = Ο οπότε c = Ο. Έτσι από την 3

(3), έχουμε ότι f(x) = �+ x2 3

(4).

Στη συνέχεια εξετάζεται το αντίστροφο. Εάν, δηλαδή, η συνάρτηση (4) πληροί τη δοσμένη συνθήκη (2) . Πιο κάτω, σε σχόλιο που παρατίθεται στο τέλος της λύσης, αναφέρονται τα εξής:

Σχόλιο:

Στην παραπάνω λύση, ίσως να αναρωτηθείτε:

«Είναι απαραίτητο να επαληθεύσουμε ότι η 3

συνάρτηση που βρήκαμε: f (χ) = �+ χ2 πληροί 3

τις δοσμένες συνθήκες;». Η απάντηση είναι «Ναι

είναι απαραίτητο». Γιατί, αφού φθάσαμε στη

συνάρτηση αυτή χωρίς ισοδυναμίες, από πουθενά,

δεν εξασφαλίζεται ότι αυτή πληροί τις δοσμένες

συνθήκες. Θα ήταν λοιπόν δυνατόν, η συνάρτηση

που βρήκαμε, να μην πληροί τις δοσμένες συνθήκες,

οπότε τέτοια συνάρτηση που ζητάμε δεν θα υπήρχε.

Επομένως, όχι μόνο είναι απαραίτητο, αλλά αν δεν

το κάνουμε η λύση θα είναι λάθος, γιατί δεν θα

ξέρουμε αν η συνάρτηση αυτή είναι η ζητούμενη ή

όχι (βλέπε επόμενη άσκηση).

Σημειώνουμε ακόμα ότι, αν τη συνάρτηση αυτή τη

βρίσκαμε με ισοδυναμίες, τότε σίγουρα θα ήταν η

ζητούμενη (σ' αυτήν την περίπτωση, αν το

επαληθεύαμε, θα σήμαινε ότι δε γνωρίζουμε την

έννοια της ισοδυναμίας).

Μολονότι, γενικά, όταν φθάνουμε σ' ένα συμπέρασμα χωρίς ισοδυναμίες απαιτείται να εξετάσουμε το αντίστροφο, στην προκειμένη περίπτωση δεν χρειάζεται. Αυτό συμβαίνει γιατί χρησιμοποιούμε για την εύρεση του c μια αρχική συνθήκη, την f(O) = Ο . Αυτή ακριβώς η συνθήκη,

ΕΥΚΛΕΙΔΗ Β' λζ' τ.3/10

Page 13: Ευκλειδης Β 51

Το βήμα του Ευκλείδη

η οποία προέρχεται από την αρχική σχέση ( 1) για χ = Ο μας εξασφαλίζει ότι η συνάρτηση που βρήκαμε μέσω συνεπαγωγών θα επαληθεύει οπωσδήποτε την αρχική σχέση και θα αποτελέσει έτσι λύση του προβλήματος. Πράγματι, θα αποδείξουμε στη γενική περίπτωση την πρόταση : Εάν f, g δύο συναρτήσεις παραγωγίσιμες σ' ένα

διάστημα Δ, τότε ισχύει η εξής ισοδυναμίας: {f'(x) = g'(x)} f(x) = g(x) {:::>

f(κ) = g(κ)

για κάθε χ Ε Δ και κ Ε Δ

Απόδειξη :

Έστω f(x) = g(x) . Τότε, f '(x) = g'(x) για κάθε χ Ε Δ , καθώς επίσης και f(κ) = g(κ) , αφού κ Ε Δ . Αντιστρόφως: Έστω ότι f'(x) = g'(x) (1) και

f(κ) = g(κ) (2), για κάθε χ Ε Δ και κ Ε Δ . Από (1) έπεται f(x) = g(x) + c για κάθε χ Ε Δ και λόγω της (2) έπεται c = Ο , οπότε

f(x) = g(x).

Στη συνέχεια όμως δίνεται, όπως αναφέρεται

στο σχόλιο, «η επόμενη άσκηση» 12 προς

τεκμηρίωση της άποψης ότι απαιτείται και η

εξέταση του αντιστρόφου. Στη διατύπωση της

άσκησης αυτής υπάρχει κάποιο τυπογραφικό

λάθος. Απ' ότι μπορεί να συμπεράνει κανείς

διαβάζοντας τη λύση, η άσκηση έχει ως εξής:

Να βρείτε τις συνεχείς συναρτήσεις f : JR � JR

για τις οποίες: "κάθε χ Ε JR , ισχύει: χ -χ J f(t)dt - J f(t)dt = 2χ2 + 2χ +ι (1)" ο 1

(άσκηση 12, σελ. 74).

Η πορεία που ακολουθείται στη λύση είναι η εξής: Παραγωγίζοντας την ( 1) βρίσκουμε τελικά ότι f(x) = 2χ + _!_. Κατόπιν εξετάζεται το αντίστροφο,

2

εάν δηλαδή η f (χ) = 2χ + _!_ επιΑηθεύει την ( 1 ) . 2

1 Θέτοντας στην (1) όπου f(x) = 2χ +-2

διαπιστώνεται ότι η f δεν επαληθεύει την ( 1 ) . Άρα

καταλήγουμε στο συμπέρασμα ότι δεν υπάρχει συνάρτηση. Φαινομενικά, δίνεται η εντύπωση ότι το παράδειγμα της άσκησης 12 έρχεται σε αντίθεση με τον παραπάνω ισχυρισμό μας, ότι δηλαδή δεν απαιτείται η εξέταση του aντιστρόφου και η επαλήθευση. Αυτό όμως δεν είναι σωστό. Το παράδοξο οφείλεται στο γεγονός ότι η ( 1) στην πραγματικότητα είναι αδύνατη . Πράγματι, η ( 1) για χ = Ο δίνει

Ο I -J f(t)dt = 1 => J f(t)dt = 1 (2). ο

Για χ = -1 δίνει: -1 ο ο 1 3 J f(t)dt = 1=> -3 J f( t) dt = 1=> J f( t) dt= -3 (3). ο -1 -1

I -1

Για χ= 1 δίνει: 3 J f(t)dt - J f(t)dt = 5 => Ο I

I I

=> 3 J f ( t)dt + J f ( t)dt = 5 => ο -1

I Ο I

=> 3 J f(t)dt + J f(t)dt + J f( t)dt = 5 ο -1 ο

ι ο =?4J f(t)dt + Jr(t)dt = 5 , (4) .

ο -1

Από (2), (3), ( 4) έπεται 4 - .!. = 5 =?!.!. = 5 , άτοπο. 3 3 Είναι γνωστό όμως ότι εάν η πρόταση p είναι ψευδής τότε η συνεπαγωγή (p =? q) είναι πάντα αληθής, ανεξάρτητα από την τιμή αληθείας της q. Επομένως στην περίπτωση αυτή οφείλουμε να διαπιστώσουμε, είτε με τον αντίστροφο είτε με απαγωγή σε άτοπο ότι η συνάρτηση που προέκυψε με συνεπαγωγές δεν αποτελεί πραγματική λύση .

Τελικό συμπέρασμα:

lη περίπτωση : Εάν η σχέση η οποία δίνεται ότι ικανοποιεί η f (ολοκληρωτική εξίσωση) είναι πράγματι αληθής, τότε δεν χρειάζεται ο έλεγχος του aντιστρόφου, αφού για την απόδειξή του χρησιμοποιείται μια αρχική συνθήκη αυτής. 2η περίπτωση : Εάν η σχέση η οποία δίνεται ότι ικανοποιεί η f είναι ψευδής (δεν ισχύει για κάθε χ στο δοθέν διάστημα), τότε η συνάρτηση που

ΕΥΚΛΕΙΔΗ Β' λζ' τ.3/1 1

Page 14: Ευκλειδης Β 51

Το βήμα του Ευκλείδη

προκύπτει με συνεπαγωγές δεν αποτελεί λύση και οφείλουμε να το αποδείξουμε, είτε με τον έλεγχο του aντιστρόφου είτε με απαγωγή σε άτοπο. Σημειωτέον δε ότι η δεύτερη περίπτωση συνιστά ένα είδος παγίδας και για το λόγο αυτό θα ήταν πιο «έντιμΟ>) από παιδαγωγικής απόψεως να διατυπώνεται η άσκηση ως εξής: «Να αποδείξετε ότι δεν υπάρχει συνεχής συνάρτηση f η οποία να ικανοποιεί τη σχέση . . . . )) ή «Υπάρχει συνεχής συνάρτηση fη οποία ικανοποιεί τη σχέση . . . . ;)) . Παρεμπιπτόντως, θα υπενθυμίσουμε ότι στο παρελθόν έχει τεθεί παρόμοιο θέμα στις πανελλαδικές εξετάσεις, εύρεσης δηλαδή μιας συνάρτησης f που να ικανοποιεί μια ολοκληρωτική εξίσωση. Το θέμα ανήκε στην πρώτη περίπτωση, αφού εκεί δεν επιτρέπονται παγίδες, και η λύση που προέκυπτε με συνεπαγωγές θεωρούνταν πάντα πλήρης και πολύ σωστά δεν αφαιρέθηκαν ποτέ μόρια για τη μη εξέταση του aντιστρόφου.

ΚΑΙ ΟΜΩΣ ΧΡΕΙΑΖΕΤ ΑΙ ΤΟ ΑΝτΙΣΤΡΟΦΟ

Αντώνης Κυριακόπουλος

Ο συνάδελφος Αντώνης Κυριακόπουλος, μέλος της Συντακτικής Εmτροπής του περιοδικού, απαντά στο γράμμα που μας έστεtλε ο συνάδελφος κ. Μπάμπης Τουμάσης, σχετικά με τις ασκήσεις 11 (σελίδα 73) και 12 (σελίδα 74) του τεύχους 48 του Ευκλείδη ΒΌ 1) Κατ' αρχήν, η γνωστή ισοδυναμία που γράφετε

(Ολοκληρώματα, Θ. Καζαντζή, έκδοση 1994, σελίδα 363), δεν αναφέρεται στο σχολικό βιβλίο. Ανεξάρτητα όμως από αυτό, με την ισοδυναμία αυτή, όταν έχουμε να επιλύσουμε μια ολοκληρωτική εξίσωση, απλώς μερικές φορές βρίσκουμε ένα ισοδύναμο σύστημα σχέσεων, στο οποίο ορισμένες ολοκληρωτικές συνθήκες έχουν μετατραπεί σε διαφορικές. Αλλά, το δύσκολο τις περισσότερες φορές είναι να συνεχίσουμε με ισοδυναμίες μέχρι να βρούμε τη ζητούμενη συνάρτηση. Τότε, και βέβαια δεν χρειάζεται το αντίστροφο. Όμως, σε όλες σχεδόν τις περιπτώσεις, είναι ευκολώτερο να εργαζόμαστε με συνεπαγωγές (χωρίς να εξετάζουμε αν ισχύουν ή όχι τα

αντίστροφα, όπως κάνουμε πολλές φορές στις άρρητες εξισώσεις). Τότε όμως χωρίς το αντίστροφο η λύση θα είναι λάθος. Για παράδειγμα, αν επιχειρήσουμε να λύσουμε την άσκηση: «Να βρείτε τις συνεχείς συναρτήσεις f : IR � IR , για τις οποίες για κάθε χ ε IR ,

χ -χ 3 4 ισχύει 2Jf(t)dt+ J f(t)dt =�+x--o ι 3 3

(Ολοκληρωτική εξίσωση)». Θα διαπιστώσουμε ότι ο δεύτερος τρόπος (με συνεπαγωγές και το αντίστροφο) είναι ευκολώτερος (θα βρούμε: f(x) = χ2 + 1) . Ερωτώ

σ' αυτή την άσκηση μπορούμε να βρούμε μια αρχική συνθήκη για τη συνάρτηση f και ποια είναι; Στις επίμαχες δύο ασκήσεις 11 και 12 ακολουθείται ο δεύτερος τρόπος. Απλά στη δεύτερη άσκηση 12, η συνάρτηση που βρίσκουμε δεν επαληθεύει τη δοσμένη εξίσωση και συνεπώς αυτή είναι αδύνατη. Στο δε αντίστοιχο σχόλιο απλά τονίζονται όλα αυτά. 2) Σε παρόμοιο θέμα στις Πανελλαδικές Εξετά­

σεις, αν ένας μαθητής εργαζόταν με συνε­παγωγές και ο βαθμολογητής θεωρούσε πλήρη τη λύση χωρίς το αντίστροφο, απλά αδικούσε τους καλούς μαθητές. Δυστυχώς, τα τελευταία κυρίως χρόνια, όλοι οι μαθητές αδικούνται ποικιλοτρόπως στις Πανελ­λαδικές εξετάσεις (λάθος θέματα, λάθος λύσεις, λάθος οδηγίες της Κεντρικής επιτροπής κτλ.).

3) Στο τεύχος 49 του Ευκλείδη Β' και στη σελίδα 80 έχει γίνει η διόρθωση του τυπογραφικού λάθος της άσκησης 12 (σελίδα 74). Στην επιστολή σας δίνετε μια άλλη λύση της άσκησης αυτής (φθάνετε σ' ένα άλλο άτοπο). Αλήθεια, εκεί δεν υποθέτετε ότι υπάρχει μία συνάρτηση f που πληροί τη σχέση (1), έστω και αν δεν το λέτε; Διαφορετικά, σε ποια συνάρτηση βάζετε χ = Ο κτλ.; Το ίδιο γίνεται και στη λύση του περιοδικού. Υποθέτει ότι υπάρχει μια τέτοια συνάρτηση και φτάνει σε άτοπο, ότι δηλαδή η συνάρτηση που αναγκαία προκύπτει, δεν πληροί την αρχική σχέση, όπως υποθέσαμε.

ΕΥΚΛΕΙΔΗ Β' λζ' τ.3/12

Page 15: Ευκλειδης Β 51

Το βήμα του Ευκλείδη

4) Αυτά που γράφετε πριν από το τελικό συμπέρασμα δεν είναι σωστά. Γιατί, πώς θα διαmστώσουμε ότι η εξίσωση είναι αδύνατη; Φuσικά με κάποια απόδειξη. Έτσι, η λύση τελειώνει εκεί και επομένως δεν οφείλουμε να διαmστώσουμε τίποτα άλλο (αλήθεια, ο δεύτερος τρόπος που προτείνετε εκεί, πάλη απαγωγή σε άτοπο δεν είναι;). Επίσης, και αυτά που γράφετε στο τελικό συμπέρασμα δεν είναι σωστά. Εκεί διακρίνετε δύο περιπτώσεις, αν υπάρχει τέτοια συνάρτηση f και αν δεν υπάρχει. Πώς όμως θα το γνωρίζουμε αυτό εκ των προτέρων; ΑJ..λά, ας υποθέσουμε ότι κατά κάποιο μαγικό τρόπο το γνωρίζουμε. Στη δεύτερη περίπτωση δεν τελειώνει εκεί το θέμα; Τι χρειάζονται τα υπόλοιπα; Στα Μαθηματικά η έννοια της εξίσωσης είναι μία, είτε ο άγνωστος είναι αριθμός, είτε είναι συνάρτηση, είτε είναι ένα οποιοδήποτε άλλο μαθηματικό αντικείμενο. όι εξισώσεις είναι προτασιακοί τύποι. Λοιπόν, μια εξίσωση δεν είναι μια σχέση που είναι αληθής ή ψευδής (δεν

είναι μια πρόταση). Επίσης, το ίσον σε μια εξίσωση δεν είναι ένα ίσον που ισχύει, ούτε είναι ένα ίσον που πρόκειται να αποδειχθεί. Και βέβαια μια εξίσωση μπορεί να είναι αδύνατη. Αυτό όμως δεν το γνωρίζουμε εκ των προτέρων.

5) Σύμφωνα με αυτά που γράφετε είναι παγίδα, και για τον λόγο αυτό, όπως λέτε, δεν είναι «έντιμο>> από παιδαγωγικής απόψεως, να ζητήσουμε από ένα μαθητή να λύσει μια εξίσωση που είναι αδύνατη. Για παράδειγμα, να ζητήσουμε από ένα μαθητή της Α' τάξης του Λυκείου να λύσει (στο R) την εξίσωση: 2χ2 - 3χ + 4 = 0 (είναι αδύνατη στο R). ΑJ..λά τότε το βιβλίο Άλγεβρας της Α' Λυκείου (και όχι μόνο) έχει παγίδες και από παιδαγωγικής απόψεως το βιβλίο αυτό δεν είναι «έντιμο», αφού έχει παρόμοιες ασκήσεις (η παραπάνω εξίσωση είναι λυμένη στο βιβλίο αυτό, σελίδα 119). Δεν είναι λοιπόν όπως τα λέτε τα πράγματα. Στα Μαθηματικά ότι είναι σωστό είναι και «έντιμο» και αυτό είναι το πρώτο πράγμα που πρέπει να διδάσκει η Παιδαγωγική.

Ι> Εnιστημοvικά Για ΑΕΙ, ΤΕΙ, ΙΕΚ Εκnαιδευτικά

Μπορείτε vo δείτε τιc; vtεc; μοc; εκδόσειc; στο lnternet στην ιστοσελίδα:

Για Γυμνόσιο, Λύκειο, ΤΕΕ Διάφορα Λογοτεχνία, μελέτες, λευκώματα

ΠΛΗΡΕΙΙ IEIPEI ΕΚΠΑΙΔΕΥ1ΙΚΩΝ ΒΙΒΛΙΩΝ

ΑΛΓΕΒΡΑ Β' ΛΥΚΕΙΟΥ Γενικής naιδι:Ιος

ΑΝ. ΑθΑΝΑΣΙΑΔΗΣ

ΕeΑΡΜΟΣΜΕΝΛ ΜΑθΗΜΑΠΚΑ

ΚΡΠΗΡΙΛ ΑΞΙΟΛΟΓΗΣΗΣ Β' & Γ' ΛΥΚΕΙΟΥ ΚΡΙΤΗΡΙΛ ΑΞΙΟΛΟΓΗΣΗΣ ΠΡΛΚΠΚΗ ΛΡΙθΜΗΠΚΗ • θtτιιιής • JtΧΥ/κής κστ/νσης • Γeνιιής naιδtίιι<; Β' ΚΥΚΛΟΥ ΤΕΕ

ΠΟΧΑΙ1ΊΚΕΙ ΑΝΕΑΙΞΕΙΙ

Τ. ΔΑΡΑΣ ·Π. ΣΥΨΑΣ

ΠΟΚΑmΚΕΣ ΛΝΕΞΕΛΙΞΕΙΣ

ΟΙΒΛΝΟΤΗΙΙΙ ΑήιιιruξιιΕφοιιιοιιίrr

ΑΛ. ΜΖΑΡΙΔΗΣ Α. ΒΕΡΝΑΡΔΟΣ ·Π. ΜΠΟτΣΑΡΗΣ Ν. ΚΑΡΑΜΠΕfΑΚΗΣ

ΠΙ8ΛΝοτΗ1ΕΣ ΛΝΛnτΥΞΗ ΕΦΛΡΜΟΓΟΝ ΕΙΣΑΓΟΓΗ ΠΗΝ Γιο καθηyητtς ΣΕ ΠΡΟΓΡΑΜΜΛΠmΚΟ ΠΕΡΙΒΛΛΛΟΝ FORl'RAN 90195

& φοιτητtς ΑΕΙ·1ΕΙ Γ' ΛΥΚΕΙΟΥ Τεχνολ. Κατει!β.

ΕΥΚΛΕΙΔΗ Β ' λζ' τ.3/13

ΜΑΘΗΜΑτtΚΑ ··.:

ΜΑθΗΜΛΠΚΑ ΓΡΑΜΜΙΚΗ ΑΛΓΕΒΡΑ Β' ΚΥΚΛΟΥ Τ.Ε.Ε. Για τους φοπητtς ΑΕΙ-ΠΙ

Ν. ΚΜΕΝΤΕΡΙΔΗΣ

Η ΓΕΟΜΕτΡΙΑ ΤΟΥΚΥΚΛΟΥ

Ν. ΚΑΡΑΜΠΕfΑΚΗΣ

Σ. ΠΑΜΑΤΑΚΗΣ

ΕΥ. ΨΟΜΟΠΟΥΛΟΣ

ΜΑθΗΜΛΠΚΑ lo ΠΡΟΓΡΑΜΜΛΠΣΜΟΣ

ΠΟ ΜΛτΗΕΜΛΠCΑ

Page 16: Ευκλειδης Β 51

16. (Το είχε προτείνει ο συνάδελφος Ροδόλφος

Μπόpης απ'τη Δάφνη Αττικής)

Έστω f : (Ο, +οο) � IR μια παραγωγίσιμη συνάρτηση. Να δειχθεί ότι η ικανή και αναγκαία συνθήκη για να υπάρχει εφαπτομένη της Cr, που διέρχεται απ' την αρχή Ο των αξόνων είναι: «υπάρχει χσ>Ο τέτοιος ώστε x0f'(x0) = f(x0) ».

Υ (ε)

Σχ. 1

Στη συνέχεια να δειχθεί ότι: «αν η Cr έχει a­σύμπτωτη (δ) με εξίσωση y = λχ που συναντάει

τη (δ) σ' ένα σημείο Κ, τότε έχει εφαπτομένη, που διέρχεται απ' την αρχή των αξόνων 0».

Απάντηση

• Για το πρώτο Η εξίσωση της εφαπτομένης (ε) σ' ένα σημείο P0 (x0, f(x0)) τηςCrείναι

y = f'(x0)x + ( f(x0)-x0f'(x0)) .

Συνεπώς: Ο Ε (ε)�

�Ο = f'(x0) ·Ο + ( f(x0)-x0f'(x0)) �

� f(x0) = x0f'(x0) .

• Για το δεύτερο

Θεωρούμε τη συνάρτηση g(x)= f(x)

-λ /(Ο,+οο), χ

που είναι προφανώς παραγωγίσιμη και για την οποία έχουμε: lim g(x) =Ο και g(ξ) =Ο όπου

Χ-Η«>

ξ η τετμημένη του σημείου Κ. Παρατηρούμε ότι: «αν υπάρχει η > ξ με g(η) =Ο , τότε κατά το Θεώρημα Rolle,

υπάρχει χ0 Ε (ξ, η) τέτοιος ώστε: g'(x0) = ό >>.

Ο Εuκλείδnς ' Ε·-·, '1 'δ· uρο·τε·ιvει .. . u'.κ.ο.ει · :D

και ... lhόφ.avτo

Οπότε: x0f'(x0)-f(x0)=0.

Το συμπέρασμα τώρα προκύπτει αμέσως απ' το πρώτο. Έστω: g( χ) # Ο για κάθε χ Ε (ξ, +οο) . Συνεπώς

η g(x) διατηρεί σταθερό πρόσημο κατά μήκος του ανοικτού διαστήματος (ξ,+οο) . Έστω α>ξ τότε g(α) >Ο ή g(α) <Ο . (I) Έστω g( α) > Ο , οπότε g(χ) > Ο για κάθε

Χ Ε (ξ, +οο) .

(Ιι): Αν g(α) = max { g(x) / x Ε (ξ,+οο)} , τότε

σύμφωνα με το Θεώρημα Fennat, ισχύει g'(α)=Ο . Και συνεπώς αf'(α)-f(α)=Ο .

Υ

Σχ. 2

(12): Έστω ότι υπάρχει χ, Ε (ξ, +οο) τέτοιος ώστε: g(x1) > g(α) .

Θεωρούμε τη συνάρτηση: φ( χ)= g(x) - g(α)/(ξ, +οο) ,

για την οποία ισχύει: φ( χ,)> Ο . + Αν χ1 Ε (ξ, α) , τότε υπάρχει β Ε (ξ, χ,) τέτοιος

ώστε φ(β) =Ο . Αφού: φ( ξ) = -g( α) < Ο . Και το

συμπέρασμα προκύπτει αμέσως απ' το Θεώρημα Bolzano.

Υ g(χι) ...... .... Μ Β : g(a) ........ --τ--·

Σχ. 3 Η g /[β, α] ικανοποιεί τις υποθέσεις του Θεωρήματος Rolle. Άρα υπάρχει χ0 Ε (β, α) τέτοιος ώστε: g'(x0) =Ο δηλαδή χσf(χσ)-f(χσ)=Ο.

ΕΥΚΛΕΙΔΗΣ Β' λζ' τ.3/14

Page 17: Ευκλειδης Β 51

Ο Εκλείδης προτείνει •.. Ευκλείδη ... και Διόφαντο

+ Αν χ1 Ε (α, +cο) , τότε υπάρχει β Ε (χ1 , +cο) τέτοιος ώστε φ(β) = Ο

Υ Μ g(χι ) - - - - - - - - - - : Β g(α) - - - - - - - � - - 1 · - - - - - - - - -

Σχ. 4 χ

Αφού: lim φ( χ) = -g(α) και συνεπώς υπάρχει χ-Η«>

χ2 αρκετά μεγάλο (μεγαλύτερο κάποιου Κ θετικού, κατά τον ορισμό του ορίου) τέτοιος ώστε φ(χ2 ) < Ο . Το συμπέρασμα τώρα προκύπτει αμέσως απ' το Θεώρημα Bolzano.Στη συνέχεια εφαρμόζουμε το Θεώρημα Rolle. Άρα υπάρχει χ0 Ε (α, β) τέτοιος ώστε φ '(Χο) = Ο. Οπότε: x0f' ( χ0 ) - f { χ0 ) = Ο

(11) Εργαζόμαστε παρόμοια στην περίπτωση : g(α) < 0 .

• Στην ίδια, κατά βάση λογική, κινείται η διαπραγμάτευση του συναδέλφου Γεράσιμου Κεραμιδά, απ' την Κέρκυρα.

• Ο συνάδελφος Αθανάσιος Καλάκος απ' τα Πατήσια της Αθήνας μας έστειλε μια δια­πραγμάτευση που στηρίζεται στο παρακάτω:

Λήμμα :

Για μια συνεχή συνάρτηση h : [α, +cο) � IR δεχόμαστε ότι: « h( α) = Ο » και ότι «υπάρχει

β Ε (α, +cο) με h(β) > 0 ». Αν lim h(x) = Ο , τότε Χ-+-1«>

η h παίρνει μέγιστη τιμή.

Απόδειξη

Για ε = h(β) υπάρχει, κατά τον ορισμό του ορίου, Κ>β τέτοιος ώστε: « jh(x) j < h(β) για κάθε χ Ε [α, +co) με χ> Κ». Άρα για κάθε χ Ε [α, +co) με χ> Κ ισχύει: h(x) � jh(x) j < h(β) . Η h είναι συνεχής κατά μήκος του κλειστού δια­στήματος [α,Κ] . Άρα, σύμφωνα με το Θεώρημα

«Μέγιστης και Ελάχιστης τιμής», υπάρχει χ0 Ε [α, Κ] τέτοιος ώστε: h(x) � h(x0 ) , για κάθε χ Ε [α,Κ] . Ωστόσο β Ε [α, Κ] . Άρα: h(x0 ) � h(β) >Ο . Και συνεπώς χ0 > α . Συμπεραίνουμε λοιπόν ότι: h(x) � h(x0 ) , για κάθε χ Ε [α, +co) · δηλαδή : h(x0 ) =rrnx{h(x)/ x E [α,+co)} . Π ό ρισμα:

Για μια συνεχή συνάρτηση h1 : [α, +cο) � IR δεχόμαστε ότι: «hι(α) = 0» και ότι «υπάρχει β Ε (α, +co) με την ιδιότητα: h1 (β) < Ο ».

Α ν lim h1 (χ) = Ο , τότε η h1 παίρνει ελάχιστη Χ-Η«>

τιμή.

Σχετικά τώρα με το Θέμα παρατηρούμε ότι: «αν Κ {ξ, f(ξ)) είναι σημείο της (1-, απ' το οποίο διέρχεται η ασύμπτωτή της (δ) με εξίσωση y = λχ ,

τότε f(ξ) = λξ ή f(ξ) = λ )) . ξ

Θεωρούμε τη συνάρτηση g(x) = f(x) - λ /[ξ, +cο) χ και παρατηρούμε ότι: «αν υπάρχει η Ε (ξ,+cο) με g( η) = Ο , τότε η g ικανοποιεί τις υποθέσεις του Θεωρήματος Rolle στο διάστημα [ξ,η] . Συνεπώς υπάρχει χ0 Ε (ξ, η) τέτοιος ώστε g'(x0 ) = Ο ή x0f'(x0 ) - f(x0 ) = Ο .

Άρα δε βλάπτεται η γενικότητα, αν δεχτούμε � τι: «για κάθε χ Ε (ξ, +cο) ισχύει: g(x) -=F- Ο )) .

Η συνάρτηση h με h(x) = jg(x) j ικανοποιεί τις υποθέσεις του Λήμματος. Άρα υπάρχει χ0 Ε (ξ, +co) τέτοιος ώστε

h(x0 ) = max {h(x) / χ Ε [ξ, +co )} . Ωστόσο η h είναι παραγωγίσιμη . Και μάλιστα

ισχύει:

h'(x) = g(x) g'(x) = g(x) . χ · f'(x) - f(x) jg(x) j jg(x) j χ2 '

για κάθε χ Ε (ξ, +co) . Συνεπώς για την h ισχύουν οι υποθέσεις του

Θεωρήματος Fermat. · Άρα h'(x0 ) = Ο ή x0f'(x0 ) - f(x0 ) = Ο .

ΕΥΚΛΕΙΔΗΣ Β' λζ' τ.3/15

Page 18: Ευκλειδης Β 51

Ο Εκλείδης προτείνει ... Ευκλείδη ... και Διόφαντο

• Στο ίδιο, βασικά, πνεύμα κινείται και η δια­πραγμάτευση του συναδέλφου Κωνσταντίνου

Ηλία απ' το Αλιβέρι Ευβοίας.

,-....; . ,-....;

17 . (Το είχε προτείνει ο Συνάδελφος Ροδόλφος

Μπόρης απ' τη Δάφνη Αττικής)

Για μια συνάρτηση f : (Ο,+οο) � � , που εί-

ναι δύο φορές παραγωγίσιμη, δεχόμαστε ό­τι: «η f" είναι συνεχής» και ότι: «η γραφι­

κή παράσταση της Cr δεν εμπεριέχει ευθύ­

γραμμο τμήμα». Υ

f(Xo) _ _ _ _ _ _ _ _ _ Ρ

χ

Αν υπάρχει ευθεία δ � y - λχ = Ο , η οποία

είναι ταυτόχρονα aσύμπτωτη και εφαπτομέ­

νη της Cr, να δειχθεί ότι «η Cr έχει τουλάχι­

στον δύο σημεία καμπής».

Απάντηση

Έστω Κ (ξ, f(ξ) ) σημείο της Cr, στο οποίο ε­φάπτεται η (δ) . Τότε: f(ξ) = λξ και f'(ξ) = λ .

Υ

Σχ.2 Θεωρούμε τη συνάρτηση g με g(x) = f(x) - λχ κι έχουμε: g(ξ) = g'(ξ) = Ο , lim g(x) = Ο (2)

χ --++«>

και για κάθε χ Ε (0, +οο) ισχύει g"(x) = f"(x) (3) .

(1): Έστω g"(ξ) > Ο . Τότε εξαιτίας της (3) και του ό,τι η f" είναι συ­

νεχής, υπάρχουν δι με Ο < δι < ξ και δ2 με δ2>Ο τέ-τοιοι ώστε: « g"(x) > Ο , για κάθε χ Ε (ξ-δι ,ξ+δ2 ) ».

Ισχυριζόμαστε ότι: η συνθήκη g" (χ) > Ο , δε μπορεί να ισχύει για κάθε χ Ε (ξ, +οο) .

Πραγματικά αν ήταν: g" (χ) > Ο , για κάθε χ Ε (ξ, +οο) , τότε η παράγωγος g ' θα'ταν γνήσια αύξουσα κατά μήκος του διαστήματος [ξ, +οο) .

Συνεπώς: g'(x) > g'(ξ) = Ο , για κάθε χ Ε (ξ, +οο) .

Κι επομένως η g θά'τανε γνήσια αύξουσα κατά μήκος του διαστήματος [ξ, +οο) . Οπότε για κάθε χ Ε (ξ, +οο) ισχύει: g(x) > g(ξ) δηλαδή : g(x) > Ο .

Για κάποιον τώρα Χι > ξ , συγκεκριμένο έχου­με: g(χ) > g(χι ) > Ο , για κάθε Χ Ε (Χι , +οο) .

Κι επειδή : lim g(x) :2: g(xι ) , συμπεραίνουμε, Χ--Η«>

κατά τη (2), ότι: Ο :2: g( Χι ) , που είναι άτοπο. Άρα υπάρχει θετικός αριθμός θ τέτοιος ώστε το

διάστημα (ξ, ξ + &) είναι το ευρύτερο διάστημα δεξιά του ξ, κατά μήκος του οποίου ισχύει: g"(x) > O .

Τώρα: αν ήταν g"(ξ + &) < Ο , τότε για οποιον­δήποτε χ του ανοικτού διαστήματος (ξ, ξ + &) θα υπήρχε, κατά το Θεώρημα Bolzano, κάποιος ρ με χ < ρ < ξ + & τέτοιος ώστε g"(ρ) = Ο : άτοπο.

Άρα: g"(ξ + &) = Ο . Απ' τον ορισμό του θ συμπεραίνουμε ότι: «υ­

πάρχει δ>θ τέτοιος ώστε: g" (χ) � Ο , για κάθε Χ Ε (ξ + &, ξ + δ) .

Συνεπώς: η παράγωγος g ' είναι φθίνουσα κατά μήκος του ανοικτού διαστήματος (ξ + &, ξ + δ) .

Έστω τώρα Χι ,χ2 τέτοιοι ώστε: ξ + & < Χι < χ2 < ξ + δ .

Τότε: g'(χι ) :2: g'(x2 ) . Αν ήταν g'(xι ) = g'(x2 ) , τότε για κάθε

Χ Ε (Χι , χ2 ) ισχύει: g'(χ ι ) :2: g'(x) :2: g'(xz )

και λόγω της υπόθεσης ισχύει μόνο το «=». Άρα η g ' είναι σταθερή κατά μήκος του διαστήμα­τος (ξ + &, ξ + δ) .

Οπότε: g'(x) = Cι , για κάθε χ Ε (ξ + &, ξ + δ) . Και συνεπώς: g( χ) = Cι χ + c2 για κάθε

Χ Ε (ξ + &, ξ + δ) ή f(x) = (λ + cι )x + c2 , για κάθε Χ Ε (ξ + &, ξ + δ) .

Η τελευταία δεν ισχύει· αντιβαίνει στην υπόθε­ση ότι η Cr δεν εμπεριέχει ευθύγραμμο τμήμα.

ΕΥΚΛΕΙΔΗΣ Β' λζ' τ.3/16

Page 19: Ευκλειδης Β 51

Ο Εκλείδης προτείνει. .. Ευκλείδη •.. και Διόφαντο

Άρα για κάθε χ Ε (ξ + 3, ξ + δ) ισχύει: g"(x) < Ο (7) και συνεπώς, λόγω της (2), το Μ (ξ + 3, f (ξ + 3)) είναι σημείο καμπής της Cr.

Παρατηρούμε τώρα ότι: g'(ξ + 3) > Ο και g'(ξ + δ) < Ο (8). Άρα υπάρχει, σύμφωνα με το Θεώρημα Bolzano, χ0 Ε (ξ + 3, ξ + δ) τέτοιος ώστε g'(x0 ) = 0 . Έστω ότι: «για κάθε Χ Ε (χ0 , +οο) ι­σχύει g"(x) < Ο ».

Τότε η παράγωγος g ' είναι γνήσια φθίνουσα κατά μήκος του διαστήματος [χ0 , +οο) . Συνεπώς για κάθε Χ Ε (χ0 , +οο) ισχύει g'(x) < g'(x0 ) = 0 · δηλαδή : g' (χ) < Ο για κάθε χ Ε ( χ0 , +οο) . Κι επο-μένως η συνάρτηση g είναι γνήσια φθίνουσα κατά μήκος του διαστήματος [ χ0 , +οο) .

Ας πάρουμε τώρα κάποιον χ2 Ε (χ0 , ξ + δ) με g( χ2 ) > Ο . Η εφαπτομένη της Cg στο σημείο Σ ( Χυg(χ2 )) έχει εξίσωση: y=g(x2 )+ g'(� )(x-� )

και συναντάει τον οριζόντιο άξονα χ ' χ στο ση­μείο R(r, O) με r = x2g'(x;) - g(x2 ) (9) .

g (χ2 ) Εύκολα διαπιστώνεται ότι: g(r) < Ο . Και συνεπώς για κάθε χ Ε ( r, +οο) ισχύει:

g(x) < g(r) [αφού η g είναι γνήσια φθίνουσα] . Συνεπώς lim g(x) � g(r) ή Ο � g(r) : άτοπο.

Χ -Η«>

Άρα υπάρχει θετικός αριθμός 3' τέτοιος ώστε: «το ανοικτό διάστημα (χ0 , χ0 + 3') είναι το ευρύ-τερο διάστημα δεξιά του Χο, κατά μήκος του οποί­ου ισχύει g"(x) < Ο ».

Με όμοιο τρόπο, όπως πιο πάνω, αποδεικνύου­με ότι: «Το Ν( ν, f(ν)) με ν = χ0 + 3' είναι σημείο καμπής της Cr».

(11) : Έστω g"(ξ) < Ο . Τότε για τη συνάρτηση h με

h(x) = -g(x) = λχ - f(x) /(O,+oo) έχουμε: h(ξ) = Ο, h'(ξ) = Ο, h"(ξ) > Ο, lim h(x) = O και για κάθε χ Ε (Ο, +οο) ισχύει:

χ -++«>

h"(x) = -f"(x) . Συνεπώς, σύμφωνα με το (1), υπάρχουν δύο

τουλάχιστον σημεία καμπής της Ch και λόγω συμ­μετρίας δύο τουλάχιστον σημεία καμπής της Cr.

(111) : Έστω g"(ξ) = Ο .

Η συνθήκη g" (χ) = Ο δε μπορεί να ισχύει για κάθε χ Ε (ξ, +οο) . Άρα υπάρχει Χι Ε (ξ, +χ) τέ­τοιος ώστε: g"(χι ) * Ο .

(lllι): Αν είναι: g"(χι ) > Ο . Τότε υπάρχει 3ι με Ο < 3ι � Χι - ξ τέτοιο.;

ώστε «το διάστημα (χι - 3ι , χ ι ] είναι το ευρύτερο διάστημα αριστερά του Χι , κατά μήκος του οποίου ισχύει: g"(x) > 0 )) . [αφού η g" είναι συνεχής] . + Αν Χι - 3ι > ξ , τότε με απλούς συλλ.ογισμούς -

παρόμοιους με εκείνους που χρηmμοποιήσαμε για το ξ + 3 της (Ι) καταλήγουμε ότι: «το Τ ( χι - 3ι , f(χι - 3ι )) είναισημείο καμπήςτης Cp).

+ Αν χι - 3ι = ξ , τότε εργαζόμαστε ακριβώς όπως εργαστήκαμε στην (1) . (lll2) : Αν g"(χι ) < Ο . Εργαζόμαστε όπως στην περίπτωση (11) .

,..._, . ,..._,

• Ο συνάδελφος Γεράσιμος Κεραμιδάς απ' την Κέρκυρα μας έστειλε μια διαπραγμάτευση που στηρίζεται στο παρακάτω.

Λήμμα:

Για μια συνάρτηση f : (Ο, +οο) � IR , που είναι

δύο φορές παραγωγίσιμη, δεχόμαστε ότι: f"(x) > Ο , για κάθε χ. Αν η ευθεία (δ) : y = λχ + β είναι aσύμπτωτη

της Cr, τότε η Cr είναι πάνω απ' την

aσύμπτωτη.

Απόδειξη

Απ' τον ορισμό της ασύμπτωτης έχουμε: lim [ f (χ) - λχ - β] = Ο . χ-+1«>

Θεωρούμε τη συνάρτηση g με g(x) = f(x) - λχ - β /(Ο, +οο) .

Θα αποδείξουμε ότι: « g(x) > Ο, για κάθε Χ)) . Έχουμε: g'(x) = f'(x) - λ, lim g(x) = Ο και για

Χ-Η«>

κάθε χ Ε (Ο, +οο) ισχύει: g"(x) = f"(x) . Αφού : g"(x) > Ο , για κάθε χ, η παράγωγος g '

είναι γνήσια αύξουσα. Για κάποιο 3 > Ο και τυχαίο θετικό χ υπάρ­

χουν, κατά το Θεώρημα Μέσης τιμής, ξι Ε (χ - 3, χ), ξ2 Ε (χ, χ + 3) τέτοιοι ώστε:

ΕΥΚΛΕΙΔΗΣ Β ' λζ' τ.3/17

Page 20: Ευκλειδης Β 51

Ο Εκλείδης προτείνει ... Ευκλείδη ... και Διόφαντο

g'(ξ1 ) = � (g(x) - g(x - θ-)) και

g'(ξz ) = �g ( x + θ-) - g ( x) ( 1 ) . Κι επειδή: ξ, < χ < ξ2 και g1, συμπεραίνουμε ότι: g'(ξ, ) < g'(x) < g'(ξz ) ή _!_(g(x) -g(x -θ-)) < g'(x) <_!_(g(x +8-) -g(x)) (2) a. a. Κι επειδή lim {.!.(g(x) - g(x - θ-))} = Ο και

Χ -ΗΟΟ a.

lim {.!.(g(x + θ-) - g(x) )} = Ο , συμπεραίνουμε, Χ-Η«> a. λόγω των (2) και του κριτηρίου παρεμβολής, ότι:

lim g'(x) = Ο (3) χ->+«>

Θα αποδείξουμε τώρα ότι: g' (χ) < Ο , για κάθε χ. Πραγματικά: για οποιονδήποτε χ1 > Ο και

Χ2 > χ, έχουμε: g'(x) > g'(x2 ) > g'(x , ) , για κάθε χ Ε (xz , +oo) [αφού η g ' είναι γνήσια αύξουσα] (4)

Απ' την παραπάνω σχέση παίρνουμε: lim g'(x) � g'(x2 ) • δηλαδή : g'(x2 ) � Ο . Οπότε ε-

χ -> +«>

ξαιτίας της (4) συνάγεται ότι: g'(x) < Ο , για κάθε χ. Άρα η g /(0, +οο) είναι γνήσια φθίνουσα. Ας είναι τώρα τυχαίος ξ>Ο και χ, > ξ . Τότε για κάθε Χ Ε ( Χ1 , +οο) ισχύει:

g(x) < g(x, ) < g(ξ) (5)

Κι επειδή lim g(x) = Ο , συμπεραίνουμε, εξαι-χ->+«>

τίας της (5), ότι: lim g(x) � g(x 1 ) ή Ο � g(x1 ) και Χ ->+«>

συνεπώς, λόγω της (5) παίρνουμε: g(ξ) > Ο .

Πόρισμα: Για μια συνάρτηση f : (0, +οο) � IR , που είναι

δύο φορές παραγωγίσιμη δεχόμαστε ότι: f"(x) < Ο , για κάθε χ.

Αν η ευθεία (δ) : y = λχ + β είναι ασύμπτωτη

της Cr, τότε η Cr είναι κάτω απ' την ασύμπτωτη.

Σχετικά με το θέμα 17 Υ y=xf(Xo)

Ρο f(Xo) _ _ _ _ _ , _ _ _ _ '

χ

Χωρίς βλάβη της γενικότητας δεχόμαστε λ>Ο. Ας είναι Κ(ξ, f(ξ)) σημείο της Cr, στο οποίο

εφάπτεται της Cr η ασύμπτωτή της. Τότε: λ = f'(ξ) και f(ξ) = λξ = f'(ξ)ξ . (6) Ωστόσο απ' το προηγούμενο Θέμα 16 βρίσκε­

ται χ0 Ε (ξ, +οο) τέτοιος ώστε: (7)

Και συνεπώς η εφαπτομένη της Cr στο Ρ0 ( χ0 , f ( χ0 )) διέρχεται απ' την αρχή Ο των αξόνων.

Αν το Ρ0 είναι σημείο της ασύμπτωτης τότε: f(x0 ) = χ0f'(ξ) . Και συνεπώς η συνάρτηση φ με

f(x) , φ( χ) = --/[ξ, χ0 ] ικανοποιει τις υποθέσεις του χ Θεωρήματος Rolle. Άρα υπάρχει χ� Ε (ξ, χ0 ) τέ­τοιος ώστε: φ'(χ� ) = Ο ή χ� · f'(x� ) - f(x� ) = Ο . Κι επειδή η Cr δεν εμπεριέχει ευθύγραμμο τμήμα, μπορούμε να θεωρήσουμε, χωρίς βλάβη της γενι­κότητας, ότι το σημείο Ρ� (χ� , f (χ� )) δεν ανήκει στην ασύμπτωτη της Cr.

Καταλήγουμε έτσι για το Ρο ότι: «ή θα είναι πάνω απ' την ασύμπτωτη» ή «θα είναι κάτω από την ασύμπτωτη» .

(1): Έστω ότι το Ρ0 είναι πάνω από την aσύ­μπτωτη .

Δηλαδή : f(x0J > λχ0 .

Συνεπώς: f(xo ) > λ ή f'(x0 ) > f'(ξ) (8). Χ ο Η συνάρτηση f /[ξ, χ0 ] ικανοποιεί τις υποθέ-

σεις του Θεωρήματος Μέσης Τιμής. Άρα υπάρχει χ, Ε (ξ, χ0 ) τέτοιος �στε:

f'(x, ) = f(xo ) - f(ξ) Χο - ξ

ή λόγω των ( 6) και (7),

f'(x, ) = Xof' (Χ ο ) - ε, f'(ξ) Χο - ξ (9).

Όμοια η συνάρτηση f' /[ξ, χ, ] ικανοποιεί τις υ-ποθέσεις του Θ.Μ.Τ . .

Οπότε υπάρχει χ2 Ε (ξ, χ, ) τέτοιος ώστε:

f"( ) f'(x1 ) - f'(ξ) , 1 1,,, ( , Χ2 = , η οποια '"'-' rω της 9) γινεται: χ, - ξ

f"(x2 ) = χσ (f'(χο ) - f'(ξ)} > 0 (10) [λfyγω της (8)] . (χο - ξ)(χ, - ξ)

ΕΥΚΛΕΙΔΗΣ Β' λζ' τ.3/18

Page 21: Ευκλειδης Β 51

Ο Εκλείδης προτείνει ... Ευκλείδη ... και Διόφαντο

Εργαζόμαστε παρόμοια και βρίσκουμε χ3 Ε (χ1 , χ0 ) τέτοιον ώστε

f"(x3 ) = -ξ (f'(χο ) - f'(ξ)) < 0 ( Ι Ι ) (χο - Χι )(χο - ξ)

Η συνάρτηση f" /[χυχ3 ] είναι συνεχής και παίρνει ετερόσημες τιμές στα άκρα του διαστήμα­τος. Άρα, σύμφωνα με το Θεώρημα Bolzano, υπάρχει μ Ε (χ2 , χ3 ) τέτοιος ώστε: f"(μ) = Ο ( Ι 2). + Αν f"(x) > Ο για κάθε χ Ε [χυμ) και f"(x) < Ο

για κάθε χ Ε (μ, χ3 ) , τότε το Μ(μ, f(μ)) είναι σημείο καμπής της Cr.

+ Αν το διάστημα [χυ χ2 + Θ-) με 0 < Θ- < μ - χ2 είναι το ευρύτερο διάστημα, κατά μήκος του οποίου ισχύει: f" (χ) > Ο , τότε εύκολα αποδεικνύεται ότι το σημείο Μ( Xz +Θ-,f(Xz +Θ-)) είναι σημείο καμπής της Cr.

+ Αν το διάστημα (χ3 - Θ-', χ3 ] με 0 < Θ-' < χ3 - μ είναι το ευρύτερο διάστημα, κατά μήκος του οποίου ισχύει f" (χ) < Ο , τότε το σημείο Μ" ( χ3 -θ', f( χ3 - Θ-')) είναι σημείο καμπής της Cr. Αποδεικνύουμε τώρα ότι: «δεξιά του Χο υπάρχει

Αν χ,y,ω θετικοί αριθμοί με χ + y + ω = ι , να βρεθεί η ελάχιστη τιμή της παράστασης

κ = (� - ι)(� - ι)(� - ι} Απάντηση

Είναι γνωστό ότι: «για οποιουσδήποτε θετικούς αριθμούς α,β ισχύει: α + β � 2� » .

Συνεπώς για οποιουσδήποτε θετικούς αριθμούς χ,y,ω ισχύει (χ + y)(y + ω)(ω + χ) � 8χyω . (1)

Ωστόσο έχουμε χ + y = Ι - ω, y + ω = Ι - χ, ω + χ = I - y

και συνεπώς η ( Ι ) γίνεται: (1-ω)(Ι - χ)(Ι - y) � 8χyω .

Ι - ω Ι - χ I - y Άρα --·--·-- � 8 ή ω χ Υ

(� - Ι)(� - Ι) (� - Ι) � 8 .

Συνεπώς η ελαχίστη τιμή της παράστασης είναι 8 . . Ι και την παιρνει οταν: χ = y = ω = - . 3

ν τέτοιος ώστε το Ν (ν, f (ν)) είναι σημείο καμπής • Την ίδια διαπραγμάτευση κάνουν οι της Cr.

Σύμφωνα με το Λήμμα, η συνθήκη f"(x) < Ο δε μπορεί να ισχύει για κάθε χ Ε (μ, +οο) . Αφού θά'πρεπε η Cr /[μ, +οο) να' ναι κάτω απ' την aσύ-μπτωτη. [άτοπο το Ρο είναι πάνω απ' την aσύ­μπτωτη] .

Άρα υπάρχει Θ-" > Ο τέτοιος ώστε το διάστημα (μ, μ + Θ-") είναι το ευρύτερο διάστημα, κατά μή­κος του οποίου ισχύει: f" (χ) < Ο .

Άρα το σημείο Ν(ν, f(ν)) με ν = μ + Θ-" είναι σημείο καμπής της Cr.

(11) Εργαζόμαστε παρόμοια στην περίπτωση που το Ρο είναι κάτω απ' την ασύμπτωτη της Cr.

Αν όμως θεωρήσουμε τη συνάρτηση g(x) = λχ - f(χ) /(Ο, +οο) η περιπτωση (11) ανάγεται αμέσως στην (1) .

,....._, . ,....._,

συνάδελφοι Νίκος Αντωνόπουλος απ' το Ίλιο Αττικής, Γιώργος Κατσούλης, μέλος της Συντακτικής Επιτροπής, Γεράσιμος Κεραμι­

δάς απ' την Κέρκυρα, Ροδόλφος Μπόρης απ' τη Δάφνη Αττικής, Σωτήρης Σκοτίδας απ' την Καρδίτσα, Γιάννης Σταματογιάννης απ' τη Δροσιά Αττικής.

,....._, . ,....._,

• Την ίδια διαπραγμάτευση κάνουν και οι μαθητές Δημήτρης Μοιρογιάννης (Β' τάξη) και Κώστας Φράγκος (Γ τάξη) απ' την Αθήνα.

,....._, . ,....._,

• Ο συνάδελφος Αθανάσιος Καλάκος απ' τα Πατήσια της Αθήνας μας έστειλε την παρακάτω διαπραγμάτευση. Θέτουμε: χ = .!.. , y = .!.. , ω = .!.. . α β γ

ιs (Τό 'χε προτείνει ο συνάδελφος Γιώργος Συνεπώς: Αποστολόπουλος, απ' το Μεσολόγγι) .

ΕΥΚΛΕΙΔΗΣ Β ' λζ τ.3/19

Page 22: Ευκλειδης Β 51

Ο Εκλείδης προτείνεL .. Ευκλείδη ... και Διόφαντο

1 1 1 � +β +γ = 1 � αβ + βγ + γα = αβγ ( 1 )

και Κ = (α - β)(β - 1)(γ - 1) = = αβγ - ( αγ + βγ + γα) + (α + β + γ) - 1 ή λόγω της ( 1 ), Κ = α + β + γ - 1 . Απ' την ανισότητα τώρα «Αριθμητικού - Γεω-

μετρικού Μέσου» παίρνουμε:

- +- +- 2: 3 . 3 - . - .- ή ψ;βΎ 2: 3 (3) . 1 1 1 ffi1 1

α β γ α β γ Επίσης απ' την ίδια ανισότητα παίρνουμε:

(3) Κ = α + β + γ - 1 2: 3ψ;βγ - 1 2: 9 - 1 = 8 ·δηλαδή

Κ 2: 8 . Η ισότητα ισχύει μόνον όταν α = β = γ = 3 .

• Ο συνάδελφος Αντώνης Ιωαννίδης απ' την Λάρισα μας έστειλε την παρακάτω διαπραγμάτευση. Έχουμε: Κ = (1 - x)(l - y)(l - ω) = χyω

= 1 - (χ + y + ω) + ( xy + yω + ωχ) - χyω χyω

<:; xy + yω + ωχ - χyω = xy + yω + ωχ χyω χyω

Είναι ωστόσο γνωστό • ότι:

(χ + Υ + ω)(.!.. + .!_ + ..!..) 2: 9 . χ Υ ω

Οπότε: xy + yω + ωχ > 9 . χyω

=

1 ( 1 )

Συνεπώς Κ 2: 9 - 1 = 8 . Το ίσον ισχύει μόνον ό-1 ταν: χ = y = ω = - . 3

• Την ίδια διαπραγμάτευση κάνουν και οι συνά­δελφοι: Χρήστος Δεμιρτσόγλου απ' τη Δράμα,

• Μετά τις πράξεις η συνθήκη γίνεται:

( � + �) + (: + :) + (: + �) � 6 , που είναι προφανής

αληθώς [ αφού για κ, λ > Ο ισχύει: � + � � 2 και η ισό­λ κ

τητα ισχύει μόνον όταν κ � λ]

Ιωάννης Τσόπελας απ' την Αμαλιάδα Ηλείας.

• Ο συνάδελφος Κωνσταντίνος Ηλίας απ' το Α­λιβέρι Ευβοίας μας έστειλε μια διαπραγμάτευ­ση με παραγώγους: Θεωρούμε τη συνάρτηση f : (0, 1) � � με

f (χ) = 1 - Υ . (1 - χ)( χ + Υ) Υ x(l - x - y)

όπου y παράμετρος που διατρέχει το διάστημα (0, 1 ) .

'Εχουμε: f'(x) = (1 - y)(2x + y - 1) x2 (1 - x - y)2

Συγκροτούμε τον παρακάτω πίνακα μεταβολών.

χ f(x) f(x)

1-y ο 2 1

' - ο + '

·�ο�ε/ , Λ

f( 1-Y) 2

Απ' τον πίνακα αυτό συνεπάγεται αμέσως ότι η συνάρτηση παίρνει την ελάχιστη τιμή της για:

1 - y χ =--2

Δηλαδή : f(x) 2: f ( 1 - y) = (l + y)2 , για κάθε 2 y(1 - y)

Χ Ε (0, 1)

Θεωρούμε τώρα τη συνάρτηση g(y) = (1 + y)

2 /(0, 1) y(1 - y)

και παρατηρούμε ότι:

'( ) - (1 + y)(3y - 1) g Υ - yz (1 - y)z

(2) .

(3)

(4)

Συγκροτούμε συνεπώς τον παρακάτω πίνακα μεταβολών:

χ ο 1/3 1 g'(y) - ό + ' g(y) �ο�ε/ $ /_'. 8

ΕΥΚΛΕΙΔΗΣ Β' λζ' τ.3/20

Page 23: Ευκλειδης Β 51

Ο Εκλείδης προτείνει ..• Ευκλείδη ... και Διόφαντο

Απ' τον πίνακα αυτόν συνάγεται αμέσως ότι η συνάρτηση παίρνει την ελάχιστη τιμή της για

1 y = - . 3 Δηλαδή : g(y) 2 g(�) = 8 , για κάθε y E (0, 1) .

Για y = � η (2) γίνεται: f(x) 2 f(�} για κάθε χ Ε (0, 1) . Άρα f (�) = min {f(x) / x Ε (0, 1)} = 8 .

Και συνεπώς Κ 2 8 . Το ίσον ισχύει μόνον όταν 1 χ = y = ω = - . 3

• Ο συνάδελφος Ελευθέριος Πρωτόπαπας, απ' το Περιστέρι Αττικής μας έστειλε μια διαπραγμάτευση που στηρίζεται στη θεωρία ελαχιστοποίησης μιας συνάρτησης με δύο μεταβλητές.

19. (Το 'χε προτείνει ο Ουνάδελφος Χρήστος Δεμιρ­

τζόγλου απ' τη Δράμα)

Για ένα τρίγωνο ΑΒΓ δεχόμαστε ότι: «η

διάμεσος ΑΜ είναι ίση με τη πλευρά ΑΒ».

Α ν απ' το Μ φέρουμε την κάθετη στη ΒΓ

και είναι Ν το σημείο που συναντάει την

ΑΓ, να δειχθεί ότι: (ΑΝ) = ( ΑΓ) . 3

Απάντηση κ

Προεκτείνουμε τις ΒΑ και ΜΝ κι ας είναι Κ το σημείο που συναντιούνται.

Το ευθύγραμμο τμήμα ΚΜ είναι διάμεσος του τριγώνου ΚΒΓ.

Στο ορθογώνιο τρίγωνο ΚΜΒ έχουμε ΑΜ = ΑΒ.

Κι επειδή � + Β = 90° = φ + ω , συμπεραίνουμε ότι: � = φ . Άρα (ΑΚ) = (ΑΜ)- δηλαδή : (ΑΒ) = (ΑΚ).

Συνεπώς το Α είναι το μέσο του ΒΚ κι επομέ­νως το Γ Α είναι διάμεσος του τριγώνου ΚΒΓ. Άρα το Ν είναι το βαρύκεντρο του τριγώνου ΚΒΓ.

. Οπότε (ΑΝ) = (ΑΓ) . 3 ,....,.; . ,....,.;

• Ο συνάδελφος Αντώνης Ιωαννίδης, απ' τη Λάρισα απέστειλε την παρακάτω διαπραγμά­τευση . Θέτουμε χ = (ΑΝ). Οπότε: (ΝΓ) = β - χ .

Α

� Β α Μ α Γ 2 2

Εφαρμόζουμε το Πρώτο Θεώρημα Δαμέσων στο τρίγωνο ΑΒΓ με διάμεσο την ΑΜ κι έχουμε:

αz αz β2 + γ2 = 2μα2 +2 ή β2 - γ2 = 2 ( 1 )

Εφαρμόζουμε το Θεώρημα Stewart στο τρίγω­νο ΑΒΓ με ενδιάμεσο ευθύγραμμο τμήμα το ΒΝ και παίρνουμε:

αzχ + γz (β - χ) = βz (β - χ) ή ( ι ) αz αzχ = (βz - γz )(β - χ)�αzχ = -(β - χ) � 2

� 2χ = β - χ � χ =� . 3 /

• Ο συνάδελφος Γιώργος Κατσούλης, μέλος της Συντακτικής Επιτροπής, έδωσε την παρακάτω απάντηση :

Α

� � Β Μ Γ Έστω Δ το μέσο του ΝΓ.

ΕΥΚΛΕΙΔΗΣ Β' λζ' τ.3/21

Page 24: Ευκλειδης Β 51

Ο Εκλείδης προτείνει ... Ευκλείδη ... και Διόφαντο

Τότε: ΜΔ 1 1= ΒΝ και (ΜΔ) = (ΔΝ) = (ΔΓ) . 2 Δηλαδή το ΜΔ είναι διάμεσος του ορθογωνίου -- --τριγώνου ΝΜΓ. Άρα: ΔΜΓ = ΜΓ Δ = ω .

Λ Απ' το τρίγωνο ΑΜΓ παίρνουμε: Μι = φ + ω . Λ Λ -Κι επειδή Μι = Β = ΑΒΝ + ΝΒΜ = ΑΒΝ+ ω --

Οπότε: ΑΒΝ = φ . Συνεπώς τα τρίγωνα ΑΒΝ και ΑΜΔ είναι ίσα --[αφού : (ΑΒ) = (ΑΜ),ΑΒΝ = ΔΑΜ και -- -

ΒΝΑ = 2ω = ΜΔΑ ] . Άρα: (ΑΝ) = (ΜΔ). Τελικά παίρνουμε: (ΑΝ) = (ΜΔ) = (ΔΝ) = (ΔΓ) δηλαδή : (ΑΝ) = (ΑΓ) . 3

• Ο συνάδελφος Γεράσιμος Κεραμιδάς απ' την Κέρκυρα έστειλε την παρακάτω απάντηση.

Α

Γ Φέρουμε το ύψος ΑΔ στο τρίγωνο ΑΒΓ, το

οποίο όπως είναι γνωστό είναι μεσοκάθετη στο ΒΜ.

Ας είναι τώρα (ΒΔ) = χ. Τότε: (ΔΜ) = χ και (ΜΓ) = 2χ.

Στο ορθογώνιο τρίγωνο ΑΔΓ είναι ΜΝ//ΑΔ. Ά ' Θ ' Θ λ ' ' ΑΝ ΑΓ ρα, κατα το .. εωρημα .. α η, ισχυει:

ΔΜ = ΔΓ ·

Και συνεπώς: ΑΝ = ΑΓ . 3

• Παρόμοια εργάστηκαν οι συνάδελφοι: Αθανάσιος Καλάκος απ' τα Πατήσια της Αθήνας, Ροδόλφος Μπόρης απ' τη Δάφνη Αττικής, Σωτήρης Γκουντουβάς απ' την Ύδρα.

• Ο συνάδελφος Στέλιος ΠετρQλέκας απ' τον Κορυδαλλό Αττικής έστειλε την παρακάτω διαπραγμάτευση.

Προεκτείνουμε την ΑΜ κατά τμήμα ΜΔ = ΑΜ. Α

Τα τρίγωνα ΑΒΜ, ΜΔΓ είναι προφανώς ίσα. Ας είναι τώρα ΔΖ κάθετη στη ΜΓ- κι ας είναι Ε

το σημείο που η ΔΖ συναντάει την ΑΓ. Το Ζ είναι μέσο του ΜΓ, αφού το τρίγωνο

ΜΔΓ είναι ισοσκελές. Στο τρίγωνο ΑΔΕ το Μ είναι το μέσο του ΑΔ.

Κι επειδή η ΜΝ είναι παράλληλη προς τη ΔΕ, συ­μπεραίνουμε ότι: ΑΝ = ΝΕ.

Ωστόσο στο τρίγωνο ΝΜΓ, το Ζ είναι μέσο της ΜΓ και συνεπώς: ΝΕ = ΕΓ. Άρα:

(ΑΝ) = (ΝΕ) = (ΕΓ) = (ΑΓ) . 3

• Ο συνάδελφος Σωτήρης Σκοτίδας απ' την Καρδίτσα έδωσε την παρακάτω λύση. Απ' το Πρώτο Θεώρημα Διαμέσων έχουμε:

αz β2 + γ2 = 2μα 2 +- . Κι επειδή μα = γ , συμπεραί-2 αz νουμε ότι: γ2 = β2 -- . 2

'Ε , ΑΝ , στω τωρα - = κ , τοτε ΝΓ ΑΝ κ ' ΑΝ Κ ΑΓ ΑΝ + ΝΓ =

κ + 1 η =κ + 1 ·

Θέτουμε � = λ/Ο < λ < 1 κ + 1 Οπότε -1- = 1 - λ κ + 1 Και συνεπώς: ΑΝ = λβ και ΝΓ = ( 1 -λ)β.

( 1 )

Απ' το τρίγωνο ΑΝΜ και το Νόμο των Συνημι-τόνων παίρνουμε:

ΕΥΚΛΕΙΔΗΣ Β' λζ' τ.3/22

Page 25: Ευκλειδης Β 51

Ο Εκλείδης προτείνει ... Ευκλείδη ... και Διόφαντο

(ΑΜ)2 = (ΑΝ)2 + (ΜΝ)2 --2(ΑΝ)(ΜΝ)συν (ΜΝΑ). Κι επειδή ΑΜ = γ, (ΜΝ)2 = (1 - λ)2 β2 - (�)2 συν (ΜΝΛ)= συν(90° + Γ) = -ημΓ , η παραπά-

νω σχέση γίνεται μετά από απλές πράξεις: �(�-1) = ο � λ = .! . 4 1 - λ 3

Ο μαθητής Γ τάξης Κώστας Φράγκος απ' την Αθήνα μας έστειλε την παρακάτω διαπραγμάτευση.

Ε

Έχουμε: ΑΜ = ΑΒ και ΕΜ κάθετη στη ΒΓ. Άρα: ΜΕΒ = 90° - Β = 90° - ΒΜΑ = ΑΜΕ . Και συνεπώς: ΑΕ = ΑΜ = ΑΒ. Οπότε: ΒΕ = 2ΕΑ Εφαρμόζουμε τώρα το Θεώρημα Μενελάου για

τη διατέμνουσα ΕΝΜ του τριγώνου ΑΒΓ κι έχουμε: ΝΑ . ΜΓ . ΕΒ _ 1 ή ΝΑ _ _! ΝΓ ΜΒ ΕΑ - ΝΓ - 2 .

Συνεπώς: ΝΑ - 1 ή ΑΝ = _!_ δηλαδή : ΝΑ + ΝΓ 3 ΑΓ 3

ΑΝ = ΑΓ . 3

• Ο συνάδελφος Νίκος Αντωνόπουλος, απ' το Ίλιον Αττικής έστειλε την παρακάτω διαπραγμάτευση.

Α

� Β α Δ α Μ α Γ

4 4 2

Αν ΑΔ είναι το ύψος του τριγώνου ΑΒΓ, τότε από την ομοιότητα των τριγώνων ΑΔΓ, ΝΜΓ

α , ΜΝ ΜΓ 2 2 παιρνουμε: -- = - = - =- . ΑΔ ΔΓ 3α 3

4

Οπότε: (ΜΝ) = �(ΑΔ) . ( 1 ) 3 1 α < I J

Έχουμε ακόμη : Ε( . ) = - · - · (ΜΝ) = ΜΝΓ 2 2

α 2 1 ( 1 ) 1 = 4 .. 3 (ΑΔ) = 3 · 2 · α · (ΑΔ) = 3E(Aiιr) (2)

1 (2 ) και Ε( . ) + Ε( . ) = -Ε( . ) � ΜΝΓ ΑΜΝ 2 ΑΜΝ

1 1 1 Ε( λλ1Ν) = 2 Ε( Aiιr) -}Ε( Aiιr) = 6 Ε( M!r) .

Άρα: Ε( Mi-ιr) = 2Ε( λλ1Ν) � Ε( ΑΜι-J = 3Ε( λλ1Ν) � .

1 1 1 � 2(ΑΓ)d = 3 · 2(AN)d � (ΑΝ) = "3 (ΑΓ)

• Ο συνάδελφος Ελευθέριος Πρωτοπαπάς απ' το Περιστέρι Αττικής έστειλε την παρακάτω διαπραγμάτευση.

Α

�Γ Στο τρίγωνο ΑΒΓ το Μ είναι μέσο του ΒΓ και

ΜΚΙΙΑΒ, άρα το Κ είναι το μέσο του ΑΓ, οπότε ΜΚ = ΑΒ � MK =l και ΑΚ = ΚΓ = ΑΓ = Q_

2 2 2 2 ' Αφού ΑΒ = ΑΜ το τρίγωνο ΑΒΜ είναι ισοσκε-

- - -λές με ΑΒΜ = ΒΜΑ = ΚΜΓ . Εξ' άλλου ισχύει:

ΛΜΝ = 90° - ΒΜΑ., ΝΜΚ = 90° - ΚΜΓ .

Συνεπώς στο τρίγωνο ΑΜΚ το ΜΝ είναι διχο­τόμος, άρα θα ισχύει ότι:

β ΑΚ · ΑΜ - · γ

ΑΝ = � ΑΝ = -2- � ΑΜ + ΜΚ + γ γ -2

ΕΥΚΛΕΙΔΗΣ Β ' λζ' τ.3/23

Page 26: Ευκλειδης Β 51

Ο Εκλείδης προτείνει ... Ευκλείδη ... και Διόφαντο

βγ l β ΑΓ <:::::> ΑΝ =- <:::::> ΑΝ = - <:::::> ΑΝ =- . 3γ 3 3 2

• Ο συνάδελφος Ι ωάννης Τσόπελας απ' την Αμαλιάδα Ηλείας έστειλε την παρακάτω διαπραγμάτευση:

Α

-Έστω ΑΒΜ = ΑΜΒ = ω ( 1 ) Θεωρούμε το τμήμα ΒΝ και το ΜΤ 1 1 ΒΝ . . Αφού η ΜΝ είναι μεσοκάθετη στο ΒΓ, συμπε--- --

ραίνουμε ότι ΝΒΓ = ΝΓΒ = φ (2) και λόγω της παραλληλίας των ΜΤ και ΒΝ συμπεραίνουμε ότι: -- -

ΝΒΓ = ΤΜΓ = φ (3) Συγκρίνουμε τα τρίγωνα ΑΒΝ και ΑΜΤ για τα

οποία: 1) ΑΒ = ΑΜ (από υπόθεση). - --2) ΑΒΝ = ΑΒΜ - ΝΒΓ = ω- φ και ---- -- ---

ΑΜΒ = ΜΑΓ+ ΝΓΒ οπότε ΜΑΓ = ω- φ . - ---Άρα ΑΒΝ = ΜΑΓ = ω- φ .

---... ο " ,.... ο 3) ΒΑΓ = 1 80 - Β - Γ = 1 80 - ω - φ και ΑΜτ = 1 8Ο0 - ΒΜΛ - fΜΓ = 1 80° - ω - φ . Άρα τα τρίγωνα ΑΒΝ και ΑΜΤ είναι ίσα μετα-

ξύ τους και συνεπώς ΑΝ = ΜΤ (4). Στο τρίγωνο ΒΝΓ το Μ είναι μέσο του ΒΓ και

ΜΤ//ΒΝ. Οπότε το Τ είναι το μέσο του ΝΓ και ΒΝ = 2 · ΜΤ ή ΝΓ = 2 · ΜΤ (5).

Από τις σχέσεις ( 4) και ( 5) συνάγεται ότι ΝΓ =2ΑΝ οπότε ΑΓ = ΝΓ +ΑΝ δηλαδή ΑΓ = 3 · ΑΝ .

20. (Το 'χε προτείνει ο συνάδελφος Χρήστος

Δεμερτζόγλου απ ' τη Δράμα).

Δίνεται τρίγωνο ΑΒΓ (Β > Γ J . Προεκτείνου­

με τη Γ Α προς το Α κατά τμήμα ΑΔ = ΑΒ. Ας

είναι Μ τυχαίο σημείο του ΒΔ. Φέρνουμε την

κάθετη απ' το Μ στη ΒΔ κι ας είναι Ν, Θ τα

σημεία που τέμνει τις ΑΒ, ΑΓ αντίστοιχα. Α ν Ε

σημείο της ΑΓ τέτοω ώστε EBr =�(Β-Γ J , να αποδειχθεί ότι: ΜΝ + ΜΘ = ΒΕ .

Απάντηση

Δ

-Ας είναι θ το μέτρο της γωνίας ΔΒΓ και ω το

μέτρο της γωνίας ΔΒΑ. . Τότε: ,9 = ω + Β . Κι επει-Λ Λ

δή Α = 2ω [η γωνία Α είναι εξωτερική του ισο-

σκελούς τριγώνου ΒΑΔ] παίρνουμε ,9 = Α + Β . Κι 2 Λ Λ Β- Γ <ΥJ -

επομένως: ,9 = 90° +-- = 90° + ΕΒΓ ( 1 ) . 2

Άρα: ,9 - EBf = 90° , δηλαδή : ΒΔ _L ΒΕ (2) . Φέρνουμε τώρα την ΑΚ κάθετη στη ΒΔ και την

ΑΛ κάθετη στη ΜΘ και παρατηρούμε: ΜΝ + ΜΘ = (ΜΑ - ΝΑ) + (ΜΑ+ ΛΘ) .

Ωστόσο το τρίγωνο ΝΑΘ είναι ισοσκελές Λ Λ Λ Λ Λ Λ

[αφού: Α1 = Θ1 ,Ν1 = Α2 και Α1 = Α2 (η ΑΚ είναι μεσοκάθετη και διχοτόμος της κορυφής)] .

Άρα: ΝΑ = ΛΘ. (4) Απ' την (3), λόγω της (4), έχουμε:

ΜΝ + ΜΘ = 2(ΜΛ) = 2(ΑΚ) = ΒΕ .

• Βασικά την ίδια λύση έδωσαν οι συνάδελφοι Αντώνης Ιωαννίδης απ' τη Λάρισα, Γεράσι­

μος Κρεμίδας απ' την Κέρκυρα, Στυλιανός

Πετρολέκας, απ' τον Κορυδαλλό Αττικής,

ΕΥΚΛΕΙΔΗΣ Β' λζ' τ.3/24

Page 27: Ευκλειδης Β 51

Ο Εκλείδης προτείνει ... Ευκλείδη ... και Διόφαντο

Ο συνάδελφος Σωτήρης Γκουντουβάς απ' την • Στο ίδιο πνεύμα, αλλά με χρήση Ύδρα έστειλε την παρακάτω διαπραγμάτευση : Τριγωνομετρίας, διαπραγματεύεται το θέμα ο

συνάδελφος Αθανάσιος Καλάκος, απ' τα Δ Πατήσια της Αθήνας.

Στο ΒΕ παίρνουμε τμήμα ΒΖ = ΜΘ. Απ' την υπόθεση έχουμε:

ω = ffiE = � (Β- Γ) . Λ Λ

Συνεπώς: Γ = Β- 2ω . Λ

( 1 ) Η Ει ως εξωτερική στο τρίγωνο ΓΒΕ επαλη-

� � ω � θεύει την ισότητα: Ε, = Γ + ω = Β - ω .

Λ Λ Κι επειδή Β2 = Β- ω , συμπεραίνουμε ότι:

Λ Λ Β2 = Ει . Οπότε ΒΑ = ΑΕ (2) .

Στο τρίγωνο ΔΒΕ ισχύει: ΒΑ = ΑΕ = ΑΔ. Οπ& τε: Δiffi = 90° .

Το τετράπλευρο ΜΒΖΘ είναι ορθογώνιο. Άρα: ΜΒ = ΘΖ.

Λ Λ Έχουμε ακόμη : Δ = Θι (αφού ΔΒ//ΘΖ) και

Λ Λ Λ Λ Δ = Βι . Άρα: Βι = Θι .

Τα τρίγωνα ΜΒΝ και ΖΘΕ είναι ίσα [αφού ΝΜΒ = θzΕ = 90° ,ΜΒ = ΘΖ,Βι = e, ] .

Άρα: ΜΝ = ΖΕ. Τελικά έχουμε: ΜΝ + ΜΘ = ΖΕ + ΒΖ = ΒΕ .

• Την ίδια, στην ουσία, διαπραγμάτευση κάνουν και οι συνάδελφοι Κωνσταντίνος Ηλίας απ' το Αλιβέρι Ευβοίας, Ελευθέριος Πρωτοπαπάς

απ' το Περιστέρι Αττικής.

• Ο συνάδελφος Γιώργος Κατσούλης, μέλος της Συντακτικής Επιτροπής, έδωσε την παρακάτω απάντηση :

Δ

Λ Λ - Α ΑΒ = ΑΔ <:::> Δ = ΔΒΑ = ω =- ( 1 ) 2

Λ Λ Ν = Θ = φ [αφού: ω + φ = 90° ] . Κι επειδή :

Γ

Λ Λ - Λ 1 ( Λ Λ ) ο Α Β2 = Β- ΓΒΕ = Β-2 Β- Γ = 90 -2 = φ

συμπεραίνουμε ότι: «η ΜΝ είναι παράλληλη προς τη ΒΕ».

Ωστόσο: ΑΒ = ΑΔ = ΑΕ. Άρα: Δiffi = 90° . ΜΘ ΔΘ ΔΘ 2ΜΘ ΔΘ Συνεπώς: - =- = --<:::> -- =- (2) ΒΕ ΔΕ 2ΑΕ ΒΕ ΑΕ

ΘΝ ΑΘ και - = -ΒΕ ΑΕ 2ΜΘ + ΘΝ ΔΘ + ΑΘ ΑΔ Άρα: = = - = 1 .

ΒΕ ΑΕ ΑΕ

Οπότε: 2ΜΘ + ΘΝ = ΒΕ

(3)

ή ΜΘ + (ΜΘ + ΘΝ) = ΒΕ ή ΜΘ + ΜΝ = ΒΕ .

• Ο συνάδελφος Σωτήρης Σκοτίδας, απ' την Καρδίτσα έστειλε την παρακάτω διαπραγμά­τευση :

ΕΥΚΛΕΙΔΗΣ Β ' λζ' τ.3/25

Page 28: Ευκλειδης Β 51

Ο Εκλείδης προτείνει ... Ευκλείδη ... και Διόφαντο

8-f φ=-2

Απ' το τρίγωνο ΒΓΕ παίρνουμε: Λ Λ Λ Λ Λ Λ Β+ Γ --- Λ Β+ Γ Ει = Γ+ φ = -2-,ΑΒΕ = Β- φ = -2- .

Άρα: ΑΒ = ΑΕ Κι επειδή ΑΒ = ΑΔ, συμπεραίνουμε ότι στο

τρίγωνο ΕΒΔ η διάμεσος ΒΑ είναι ίση με το μισό της πλευράς ΔΕ.

Άρα το τρίγωνο ΕΒΔ είναι ορθογώνιο στο Β · δηλαδή η ΒΕ είναι κάθετη στη ΒΔ. Και συνεπώς ΒΕ//ΜΝ.

Τα τρίγωνα ΜΒΝ και ΔΒΕ είναι όμοια. [Ορθο-Λ Λ γώνια και Βι = Δι ] .

Σ , ΜΝ ΜΒ υνεπως: -- = -- . ΒΕ ΒΔ ( 1 )

Επίσης τα τρίγωνα: ΔΜΘ και ΔΒΕ είναι όμοια. , ΜΘ ΜΔ Συνεπως: - = - (2)

ΒΕ ΒΔ <ι > (ΜΒ ΜΔ ) Άρα: ΜΝ + ΜΘ = -+- ΒΕ = 1 · ΒΕ . <2> ΒΔ ΒΔ

• Στο ίδιο κλίμα κινείται και η απάντηση του μαθητή Β ' τάξης Δη μήτρη Μοιρογιάννη απ' την Αθήνα.

Ο συνάδελφος Γιάννης Σταματογιάννης, απ' τη Δροσιά Αττικής έστειλε την παρακάτω δια-

Λ Λ 1 ( Λ Λ ) Έχουμε: Βι = Β- φ = 2 Β+ Γ ( 1 )

Λ Λ Άρα: Βι = Ει . Οπότε: ΑΒ = ΑΕ. Συνεπώς: ΕΓ = ΑΓ- ΑΕ = ΑΓ -ΑΒ . (3) Ας είναι τώρα Η το ίχνος του ύψους του τριγώ-

νου ΑΒΕ, που άγεται απ' την κορυφή Α. Η ΑΗ εί-Λ ναι διχοτόμος της γωνίας Α . [το τρίγωνο ΑΒΕ εί-

ναι ισοσκελές] . Λ Λ Λ Λ Λ Λ Έχουμε: Αι +Α2 = Δι + Β2 ή 2Α2 = 2Δι . Κι ε-

πομένως η ΑΗ είναι παράλληλη προς τη ΒΔ· δη­λαδή ΔΒΕ = 90° .

Α ν τώρα η ΑΗ τέμνει τη ΜΝ στο Ζ, τότε ---ΑΖΘ = 90° . Και συνεπώς το τετράπλευρο ΒΗΖΜ είναι ορθογώνιο. Άρα: ΜΖ = ΒΗ = ΒΕ

(4) 2 Συνεπώς:

(4) ΜΝ+ΜΘ=ΜΖ+ΖΝ+ΜΘ=ΒΗ+ ΖΝ+ΜΘ (5). Το τρίγωνο ΘΑΝ είναι ισοσκελές, αφού η διχο-

Λ τόμος της γωνίας Α είναι και ύψος άρα μεσοκά-θετη. Δηλαδή ΘΖ = ΖΝ.

(5) Έτσι έχουμε: ΜΝ + ΜΘ = ΒΗ + ΘΖ + ΜΘ =

(6) = ΒΗ + ΜΖ = 2ΒΗ = ΒΕ .

• Στο ίδιο πνεύμα κινείται και η διαπραγμάτευση του συναδέλφου Ροδόλφου Μπόρη απ' τη Δάφνη Αττικής.

Ο συνάδελφος Γιάννης Τσόπελας απ' την Αμαλιάδα · Ηλείας μας έστειλε την παρακάτω διαπραγμάτευση :

Δ

Γ

Θεωρούμε το σημείο Θ ' πάνω στην προέκταση της ΜΘ προς το Μ ώστε ΜΘ' = ΜΘ.

Έστω Κ το σημείο τομής των ΔΘ ' και ΒΕ. Ας ----ονομάσομε την γωνία ΒΑΓ = 2φ , τότε ---ΑΔΒ = ΑΒΔ = φ διότι το τρίγωνο ΑΒΔ είναι ισο-Λ Λ 1 ( Λ Λ ) και Ει = Γ+ φ = 2" Β+ Γ . (2) σκελές. Το τρίγωνο ΘΔΘ' έχει την ΔΜ ύψος και

ΕΥΚΛΕΙΔΗΣ Β' λζ' τ.3/26

Page 29: Ευκλειδης Β 51

Ο Εκλείδης προτείνει ... Ευκλείδη ... και Διόφαντο

διάμεσο. Άρα το τρίγωνο ΘΔΘ' είναι ισοσκελές με ΘΔ = Θ' Δ Άρα η ΔΜ είναι διχοτόμος της γωνίας Δ . Συνεπώς ΜΔθ' = φ

και ΑΒΕ=Β-ΕΒr=Β-Ξ(Β- Γ ) = Ξ( Β+ Γ} Α.+ Β+ Γ = ι 8ο0 => Β+ Γ = ι 8ο - 2φ =>

=> Ξ( Β+ Γ) = 90° - φ (2)

(ι), (2) => ΆΒΕ = 90° - φ = -ΜΝΒ = ΑΝΘ . Άρα ABE=ANe=90° - φ (3) Άρα ΒΕ//ΝΘ διότι έχουνε τις εντός, εκτός και

επί τα αυτά μέρη γωνίες ίσες. ( 4) - -

Επίσης ΚΔΑ = ΒΑΓ = 2φ οπότε ΔΚ// ΑΒ διό-τι έχουνε τις εντός, εκτός και επί τα αυτά μέρη γω-νίες ίσες. (5).

Από τις (4), (5) συμπεραίνουμε ότι το τετρά-πλευρο ΚΒΝΘ' είναι παραλληλόγραμμο

Οπότε: Θ'Ν = ΚΒ. Αρα Θ'Μ +ΜΝ = ΚΒ::::> ΜΘ + ΜΝ =ΚΒ (6)

ΚΕ //ΘΘ' και ΔΜ ..l ΘΘ' . Άρα ΔΒ ..l ΚΕ . Συνεπώς στο τρίγωνο ΔΚΕ το ΔΒ είναι ύψος

και διχοτόμος οπότε η ΔΒ είναι διάμεσος Συνεπώς ΚΒ = ΒΕ (7). Από τις (6), (7) παίρνουμε ΒΕ = ΜΘ +ΜΝ .

,....., . ,.....,

21 . (Το 'χε προτείνει ο συνάδελφος Γιάννης

Στρατήγης απ 'την Τρίπολη)

Να αποδειχθεί ότι: «για κάθε ν Ε Ν με ν>l

ισχύει: ;(ι -;) � ( ι -; )• <� ».

+ Για το πρώτο: Εύκολα διαπιστώνουμε ότι: «για οποιονδήποτε

θ . . κ - ι κ ( ι ) ετικο κ ισχυει: -- < -- » . κ κ + ι Θέτουμε: κ = 2,3 , . . . ,(ν- ι ) και παίρνουμε

ι 2 3 ν - 2 ν - ι - < - < - < . . . < -- < -- (2) 2 3 4 ν - ι ν ι ν - 1 2 ν - ι ν - 2 ν - ι Οπότε: - < -- ,- < -- , . . . . ,-- < --2 ν 3 ν ν - ι ν όταν: ν>2.

ι 2 ν - 2 ν - 1 ν - ι ν - ι Άρα: 2 · 3 . . . . ν - ι < -ν- ·-ν

-. . . -ν- ή

ι ( ν - ι )ν-2 -- < --ν - ι ν

ν-2 παραγ.

ι ν - ι ν - ι ν - ι -( )2 ( )ν-2

( '

Συνεπώς: ν - ι · --;- < -ν- · --;-) , ν - ι ( ν - ι )ν η -- < --

ν2 ν

Δηλαδή �(ι -�) < (ι - � )•

Για ν = 2 ισχύει η ισότητα. + Για το δεύτερο :

Απ' την ( 1 ) παίρνουμε ν - ι ν ν + ι ν + (ν - ι) 2ν - ι -- < -- < -- < . . . < =--. ν ν - ι ν + 2 ν + ν 2ν

· Και συνεπώς ν- 1 ν- ι ν- ι ν ν+ ι 2ν- ι ν ι

- · - --<--· -- --=-=-ν ν · · · ν ν+ ι ν+ 2 · · · · · 2ν 2ν 2

ν παραγ.

. . λλ λό ( ν - ι )ν ι . (ι ι )ν ι η με α α για: -ν- < 2 η -� < 2 .

,....., . ,.....,

• Ο συνάδελφος Γερμάσιμος Κεραμιδάς απ' την Κέρκυρα απόδειξε το πρώτο μέρος με τη βοήθεια της ανισότητας Bemoulli . Πραγματικά:

( 1 )ν-Ι ι Η αποδεικτέα γίνεται: ι - � > � Ωστόσο απ' την ανισότητα Bemoulli παίρνο�

( ι )ν-Ι ι ι με: ι - - � ι - (ν - 1) · - = - . ν ν ν Το ίσον ισχύει μόνον όταν ν = 2. Για το δεύτερο μέρος χρησιμοποιεί το παρακάτω. Λήμμα:

Αν μια ακολουθία (α.) είναι αύξουσα κι έχει όριο f , τότε: « α. � f για κάθε ν Ε Ν* ».

Στη συνέχεια, αφού αποδείξει ότι:

1im (ι-.!.)' =� , συμπεραίνει ότι: 1im (ι -.!.)γ = χ-+«<> χ e • ..._, ν e

ΕΥΚΛΕΙΔΗΣ Β' λζ' τ.3/27

Page 30: Ευκλειδης Β 51

Ο Εκλείδης προτείνει ... Ευκλείδη ... και Διόφαντο

Κατόπιν δείχνει ότι: «η ακολουθία (αν) με

αν = (ι -�)ν είναι γνήσια αύξουσα». 'Ε , , (ι ι )ν ι τσι φτανει στην ανισοτητα: -� < ; , για

κάθε ν Ε Ν με ν> 1 . κ δ ' ι ι ' ' ζ ' ι επει η : - < -=-- , συναγεται α μεσ ως το ητου-e 2

μενο.

• Ο συνάδελφος Αθανάσιος Καλάκος απ' τα Άνω Πατήσια Αθήνας μας έστειλε μια διαπραγμάτευση · όπου με τη βοήθεια της ανισότητας Bemoulli αποδεικνύει και τα δύο σκέλη. Το δεύτερο σκέλος της αποδεικτέας γίνεται:

2 < (-ν-)ν = (ι + _ι _)ν και απ' την ανισότητα ν - ι ν - ι

Bemoulli έχουμε: (ι +-ι-)ν > ι + ν-ι- > 2 για ν - ι ν - 1 κάθε ν Ε Ν με ν> ι .

• Την ίδια διαπραγμάτευση κάνει και ο συνάδελφος Ροδόλφος Μπόρης απ' τη Δάφνη Αττικής.

Ο συνάδελφος Αντώνης Ιωαννίδης απ' την Λάρισα αποδεικνύει το πρώτο σκέλος με τη Μέθο­δο της Τελείας Επαγωγής.

+ Για ν = 3 η αποδεικτέα γίνεται: .!.. · � < (�)3 ή 3 3 3 3<4.

+ Δεχόμαστε ότι: για κάποιον κ με κ � 4 ισχύει:

( ι )κ-Ι ι ι - κ > κ

Θα δείξουμε ότι: (ι - _ι_)κ > _ι_ . κ + ι κ + ι

Πραγματικά:

( ι )κ ( ι )κ-Ι ( ι ) ι - κ + ι = ι - κ + ι

. ι - κ + ι >

( ι ) .

> (ι -.!.)κ-Ι . � > .!. ._κ_ =_ι_ . κ κ + ι κ κ + ι κ + ι + Για το δεύτερο σκέλος θεωρεί τη συνάρτηση

f(x) = (ι -� γ /(1, +οο) και αποδεικνύει ότι

είναι γνήσια αύξουσα. Πραγματικά:

για κάθε χ Ε (ι, +οο) .

Άρα: f ' ( x ) = ( ι - �)χ g(x) Όπου: g(x) = .en ( ι -.!.) +-ι- . χ χ - ι 'Εχουμε: g'(x) -ι < 0 , για κάθε χ Ε (ι, +οο) . χ(χ- ι) Οπότε η g(x) είναι γνήσια φθίνουσα. Ωστόσο lim g(x) = O και

Χ -Η«>

(χ - ι ).en 1 - - + ι ( ι ) lim g(x) = lim χ = +οο x�l+ χ--+1+ Χ

διότι: lim [cx - ι).eη ( ι -.!.)] = Ο . χ�Ι· χ Συνεπώς το σύνολο τιμών της συνάρτηση g(x)

είναι το ανοικτό διάστημα: (Ο, +οο) · δηλαδή : g( χ) > Ο , για κάθε χ Ε (ι, +οο) .

Απ' τη (2) τώρα παίρνουμε: « f'(x) > Ο , για κάθε χ Ε (1, +οο) »· δηλαδή η f(x) είναι γνήσια αύξουσα.

Για κάποιον τώρα χ Ε (1, +οο) και για κάθε t με t>x έχουμε: Ο < f(x) < f(t) . Και επειδή

Ι. f( ) ι . ' ' ιm t = - , συμπεραινουμε, συμφωνα με την ι-��- e

* Έχουμε:

lim [t ln (ι - !)] = lim ln ( l - �) lim -6 · -iz =

t�+oo t ι�+οο 1 t�+oo 1 t t2

= - lim _t _ = - 1 Άρα: lim f (x ) = e-1 = ..!_ ι�+οο t - 1 !->-+«> e

ΕΥΚΛΕΙΔΗΣ Β ' λζ' τ.3/28

Page 31: Ευκλειδης Β 51

Ο Εκλείδης προτείνει .•. Ευκλείδη •.. και Διόφαντο

ι ι διότητα «όριο και διάταξη» ότι: f(x) � - < - . Δη­e 2

λαδή : f(χ) < ..!.. , για κάθε χ Ε (ι, -tοο) . 2

Ο μαθητής Β ' τάξης Δημήτριος Μοιρογιάννης απ' την Αθήνα διαπραγματεύεται το πρώτο σκέλος της αποδεικτέας με την Μέθοδο της Τελείας Επα-

δ , , ι (ι ι )ν , γωγής, απο εικνυοντας οτι: « � < -� , για κα-

θε ν Ε Ν με ν :2:: 4 ». Οπότε για κάθε ν Ε Ν με ν :2:: 4 ισχύει:

�(ι - �) < � < (ι -�) ν , δηλαδή

� (ι -�) < (ι -�) ν για κάθε ν Ε Ν με ν :2:: 3 .

Αφού για ν = 3 είναι ισοδύναμη με την 3 < 4 (που είναι αληθής)

Για ν = 2 ισχύει η ισότητα. Για το δεύτερο σκέλος αναπτύσσει κατά

Newton το διώνυμο: (ι + _ι_)ν ν - ι Οπότε: (ι +

-

ι )ν = ι + (ν)_

ι + (ν) · ι 2 + . . . + ν - ι ι ν - ι 2 (ν - ι)

+ . + . > 2 . ( ν ) ι (ν) ι ν - ι (ν - ιy-' ν (ν - ιγ

' ( ν )ν ' ( ν - ι )ν ι δ λ δ ' Συνεπως: -- > 2 η -- < - · η α η ν - ι ν 2

(ι -�)ν <� .

Ο μαθητής Γ τάξης Κώστας Φράγκος απ' την Αθήνα μας έστειλε μια διαπραγμάτευση του θέμα­τος όπου το δεύτερο σκέλος το αποδεικνύει με τη βοήθεια της ανισότητας: « ex :2:: χ + ι , η οποία ισχύ­ει για κάθε χ Ε JR ».

Έτσι για κάθε χ με χ> ι έχουμε:

e-� > ι + �ι ή e-' > (ι - �)χ

Και συνεπώς για χ = ν, ν Ε Ν με ν> ι ισχύει: (ι -.!.)ν < .!. < ..!.. •

ν e 2 Το πρώτο σκέλος το αποδεικνύει με τη Μέθοδο

της Τελείας Επαγωγής.

22. (Τό 'χε προτείνει ο συνάδελφος Γιάννης

Στρατήγης απ ' την Τρίπολη).

Για οποιοδήποτε τρίγωνο ΑΒΓ ισχύει: α + β + γ ;:: � . υ: + υ� υ� + υ; υ; + υ: τ

Απάντηση Ε

Απ' το Α φέρουμε ευθεία ε, παράλληλη προς τη ΒΓ. Α ν Ε είναι το συμμετρικό του Β ως προς την ε,

τότε: ΒΖ = ΖΕ = υα ,ΑΕ = ΑΒ = γ ( ι ) . Τώρα στο ορθογώνιο τρίγωνο ΕΒΓ έχουμε:

(ΒΕ)2 = (ΓΕ)2 - α2 • (2) (Ι ) Κι επειδή : ΓΕ � Γ Α + ΑΕ = β + γ , συμπεραί-

νουμε ότι: (ΒΕ)2 � (β + γ)2 - α2 = (β + γ + α)(β + γ - α) . Άρα: (2υα )2 � 2τ(2τ - 2α) ή υ; � τ(τ - α) (3) Εργαζόμαστε παρόμοια για το υβ και παίρνου-

με: υ� � τ( τ - β) (4). (3) γ ι Άρα: υ2 + υβ2 � τγ και συνεπώς: > -α (4) υ2 + υ2 - τ . α β

α ι β ι Όμοια βρίσκουμε: > - > - και υ� + υ� - τ ' υ� + υ� - τ συνεπώς προσθέτοντας κατά μέλη καταλήγουμε στην αποδεικτέα.

• Ο συνάδελφος Χρήστος �εμφτ�ό· ,.. i . ι:• ι a..-τ · τη Δράμα αποδεικνύει κατ' αρ;ι:ά.; το

ΕΥΚΛΕΙΔΗΣ Β' λζ' τ.3/29

Page 32: Ευκλειδης Β 51

Ο Εκλείδης προτείνει ... Ευκλείδη ... και Διόφαντο

Λήμμα: «σε κάθε τρίγωνο ΑΒΓ ισχύει: 4(τ - β)( τ - γ) :::; α2 ».

Απόδειξη :

4(τ - β)(τ - γ) = (2τ - 2β)(2τ - 2γ) = = (α - β + γ)(α + β - γ) = [α - (β - γ)) [α + (β - γ)] = α2 - (β - γ)2 ::; α2 •

Έχουμε τώρα για το θέμα που μας ενδιαφέρει: 2 4 υα = -2 τ(τ - α)(τ - β)(τ - γ) . (Απ' τον τύπο του α

Ήρωνα) Οπότε, λόγω του Λήμματος, παίρνουμε:

2 2 < ' γ > 1 υ α + υβ - τγ η 2 2 - - ο υ α + υβ τ

Από εδώ και μετά η διαδικασία προχωράει ό­πως προηγούμενα.

,....,. . ,....,.

• Με τον ίδιο βαmκά τρόπο διαπραγματεύονται το θέμα οι συνάδελφοι: Αντώνης Ιωαννίδης απ' την Λάρισα, Γεράσιμος Κεραμιδάς απ' την Κέρκυρα, Σωτήρης Σκοτίδας απ' την Καρδίτσα.

23. (Το 'χε προτείνει ο συνάδελφος Ελευθέριος

Πρωτοπαπάς απ' το Περιστέρι Αττικής). π 7π

Αν z ε C με Argz = & / - ::; & ::; - και 2 4

l2z - sι = 5 , να υπολογίσετε Arg(2z4 - 5z3 ) . Απάντηση

Έστω ρ = lzl . Τότε z = ρ(συν& + iημ&) . ( 1 ) Οπότε: 2z - 5 = (2ρσυν& - 5) + i(2ρημ&) . (2) Συνεπώς: l 2z - 51 = 5 <=>

<::::> (2ρσυνJ - 5)2 + (2ρημJ)2 = 25 <::::>

' 3π 7π Οποτε: - < & :::; - και Ο<ρ<5 . 2 4 Αναζητούμε το πρωτεύον όρισμα του:

w = 2z4 - 5z3 = z\2z - 5)

Έχουμε: (2 ) 2z - 5 = (2 · 5συν2& - 5) + i(2 · 5συν&ημ&) = (3)

(4)

(5)

= 5 [ (2συν2 & - 1 J + i(2ημ&συν&) ο (6)

Κι επομένως: 2z - 5 = 5( συν& + iημ2&) . Ωστόσο : z3 = ρ\ συν3& + iημ3&) . (7)

( 6 ) Άρα: w = 5ρ3 ( συν5& + iημ5&) =

( 7 )

= 5ρ3 [ συν(5&+ 2κπ) + iημ(5&+ 2κπ)] / κ ε Ζ (8)

' 3π 7π Εχουμε: - < & :::; - . 2 4

' 1 5π 35π Και συνεπως: - < 5& :::; -2 4

' 6 3π 5 (\ 6 1 1π ' η π + - < \7 :::; π +- η 2 4 3π 1 1π 3π - < 5& - 6π ::; - = 2π +- . 2 4 4

' 3π 3π 8π Εστω: - < 5& - 6π < 2π <::::> - < & < - . 2 2 5

Τότε: Argw = 5&- 6π . [απ' το τύπο (8) για κ =- 3] .

Έστω 3π 3π 2π ::; 5& - 6π :::; 2π +- <=> ο :::; 5& - 8π ::;-4 4

τότε: Argw = 5& - 8π . [απ' το τύπο (8) για κ= -4]

Τελικά παίρνουμε: {Sθ - 6π,

Argw = 5θ - 8π,

3π 8π για - < θ < -2 5

8π 7π για - < θ ::; -

5 4

Άρα: συνθ>Ο.

<::::> συν& = Ε. 5 (3) • Με τον ίδιο, κατά βάση, τρόπο διαπραγματεύο­νται το θέμα οι συνάδελφοι Αθανάσιος Καλά-

ΕΥΚΛΕΙΔΗΣ Β' λζ' τ.3/30

Page 33: Ευκλειδης Β 51

Ο Εκλείδης προτείνει .•• Ευκλείδη .•. και Διόφαντο

κος απ' τα Άνω Πατήσια Αθήνας, Γιάννης Έστω M(z) σημείο του τόξου αυτού. Φέρουμε Σταματογιάννης απ' τη Δροσιά Αττικής. την ΟΜ και λαμβάνουμε σημείο Α, συμμετρικό

"" • "" του Ο ως προς το Μ. Προφανώς στο Α αντιστοιχεί ο μιγαδικός αριθμός 2z. Απ' το Α φέρουμε την κά-

• Στα πλαίσια των. παραπάνω ιδεών κινείται η θετη στην κατακόρυφη y 'y. Κι ας είναι: (ΑΒ) = 5 . διαπραγμάτευση του συνάδελφου Αντώνη Τότε στο Β αντιστοιχεί ο μιγαδικός αριθμός 2z-5 .

Ιωαννίδη απ' τη Λάρισα.

Ο συνάδελφος Γιώργος Κατσαούνης απ' την Πρέβεζα έστειλε την παρακάτω διαπραγμάτευση .

Υ � \ Κ( � ,0)

χ

A(2z)

Έστω z = χ + iy I χ, y ε IR . Τότε: j2z - 5 j = 5 ς:::> χ2 + y2 = 5χ ( 1 ).

Είναι προφανές ότι: χ * Ο διότι διαφορετικά απ' την ( 1 ) θα παίρναμε y = Ο. Οπότε θά 'χαμε: z = Ο κι αυτό αντιβαίνει την υπόθεση ότι: «ο μιγαδικός α-

θ ' ' ' ' θ π (\ 7π ρι μος z εχει πρωτευον ορισμα με - :::; -cr :::; - ». 2 4 Απ' την ( 1 ) παίρνουμε χ>Ο (αφού : χ2 + y2 > Ο ) . Ας είναι τώρα jz j = ρ . Τότε: z = ρ( συνg + iημg) .

Οπότε: χ = ρσυνg ή συνg = � > Ο . Ά . 3π

(\ < 7π ρα. - < -cr - - . 2 4

ρ

Έτσι ο γεωμετρικός τόπος των σημείων M(z) --

του μιγαδικού επιπέδου είναι το τόξο ΟΣ του κύ-

κλου κέντρου Κ (% , Ο) και ακτίνας r = % , μη συ-μπεριλαμβανομένου του κέντρου Ο.

Το τρίγωνο ΟΒΑ είναι ισοσκελές. [αφού (ΟΒ) -= 5 = (ΑΒ)] . Άρα: ΒΑΟ = ΑΟΒ = ω ,

Τώρα απ' το ορθογώνιο τρίγωνο ΟΓΑ παί� π -νουμε: ω = - -ΑΟΓ . (2) 2

Κι επειδή : Aor = g - 3π έχουμε: 2

Άρα: Arg(2z - 5) = g - ω = g - (2π - g) = 2g - 2π (3)

' 3π 7π 3π αφου - < g ::ς - ς::::> π < 2g - 2π :::;- . 2 4 2 Έχουμε τώρα:

Arg(2z4 - 5z3 ) = Arg [ z3 (2z - 5) J . Οπότε: Arg(2z4 - 5z3 ) = Argz3 + Arg(2z - 5) + 2κπ όπου κ ε Ζ (κατάλληλος) . = 3g + (2g - 2π) + 2κπ = 5g + 2μπ / μ ε Ζ (κατάλληλος) . (4) Από εδώ και μετά η διαδικασία προχωράει α­

κριβώς όπως πιο πάνω. ,-....; . ,-....;

Ο συνάδελφος Γιάννης Δ. Στρατής, μέλος της Συντακτικής Επιτροπής, έδωσε την παρακάτω α­πάντηση :

'Εχουμε: j2z - 5 j = 5 ς:::> Jz -%1 =% ( 1 ) και συνε-

πώς ο γεωμετρικός τόπος των σημείων M(z) του .\Ιι­γαδικού Επιπέδου, που επαληθεύουν την ( 1 ). εiναι ο

' κλο c , Α( 5 ο) · 5 κυ ς 2 με κεντρο l , και ακτινα r = Ξ .

ΕΥΚΛΕΙΔΗΣ Β' λζ' τ.3/31

Page 34: Ευκλειδης Β 51

Ο Εκλείδης προτείνει ... Ευκλείδη ... και Διόφαντο

Im

Απ' το σχήμα συνάγεται αμέσως ότι το πέρας ....

Μ της διανυσματικής ακτίνας ΟΜ , που αντιστοι-.-..

χεί στο μιγαδικό αριθμό z, διαγράφει το τόξο ΟΑ μη συμπεριλαμβανομένου του άκρου Ο.

Άρα: 3π < $- � 7π (2). 2 4 --

Έστω τώρα φ το μέτρο της γωνίας ΑΟΜ . Τότε $- = 2π - φ ή φ = 2π - $- (3). Απ' το Μ φέρουμε ευθύγραμμο τμήμα ΜΝ παράλληλο και ίσο προς

.... το ΑΟ (ομόρροπο προς το διάνυσμα ΑΟ ). Το τε-τράπλευρο ΑΟΝΜ είναι ρόμβος. Συνεπώς η δια-

-γώνιος ΟΜ διχοτομεί τη γωνία ΑΟΝ · δηλαδή το

-μέτρο της γωνίας ΑΟΝ είναι 2φ.

Στο σημείο Ν αντιστοιχεί ο μιγαδικός αριθμός: 5 κ ' ' ' 5 ' z - 2 . ι επομενως ενα ορισμα του z - 2 ειναι:

(3 ) 2π - 2φ = 2π - 2(2π - $-) = 2$- - 2π . Επομένως z -2. = 2.[[ συν(2$- - 2π)] + iημ(2$- - 2π) J = 2 2 = 2.( συν2$- + iημ2$-) . 2 Άρα: 2z - 5 = 5(συν2$- + iημ2$-) . Τώρα αν lz l = ρ , τότε z = ρ( συν$- + iημ$-) . Οπότε: z3 = ρ3 (συν3$- + iημ3$-) . Κι επειδή : 2z4 - 5z3 = z\2z - 5) συμπεραίνου­

(5 ) με ότι: 2z4 - 5z3 = z3 = 5ρ3 ( συν5$- + iημ5$-) .

(6 ) Δηλαδή : 2z4 -5i =5ρ3 [συν(5a+2κπ)+iημ(5a+2kπ)] !κ εΖ . Από δω και μετά η διαδικασία προχωράει όπως

πιο πάνω.

Ο μαθητής Γ ' τάξης Κώστας Φράγκος απ' την Αθήνα απέδειξε κατ' αρχάς ότι:

συν$- =l.:l 5

Απ' όπου συμπεραίνουμε ότι: 3π < $- � 7π •

Ας είναι τώρα 2 4

Arg(2z - 5) = φ / Ο � φ < 2π . Τότε:

( 1 ) .

(2).

(3)

2z - 5 = l2z - 5 l (συνφ + iημφ) = 5(συνφ + iημφ) .(4)

Κι επειδή : z = lz l (συν$- + iημ$-) συμπεραίνουμε ( Ι )

ότι: 2 lz l (συνθ + iημθ) - 5 = 5(συνφ + iημφ)<=> <:::::> 2 · 5συνθ( συνθ + iημθ) - 5 = 5( συνθ + iημθ) <:::::> 2συν2 $- - 1 = συνφ και 2συν$-ημ$- = ημφ . Δηλαδή : συν2$- = συνφ και ημ2$- = ημφ <:::::> 2$- - φ = 2κπ ή φ = 2$- - 2κπ / κ ε Ζ . ..,. Αν ήταν κ � Ο , τότε φ>3π: άτοπο.

...._ Α ' 2 ' π ' .,... ν ηταν κ :?: , τοτε φ < -- : ατοπο. 2 Άρα: κ = 1 . Επομένως: φ = 2$- - 2π .

Οπότε: 2z - 5 = 5 [ συν(2$- - 2π) + iημ(2$- - 2π)] = = 5( συν2$- + iημ2$-) .

Από εδώ και μετά η διαπραγμάτευση προχωρά­ει όπως πιο πάνω.

* Εύκολα αποδεικνύεται ότι: Ι z l > Ο και συνεπώς

συνθ > Ο ΕΥΚΛΕΙΔΗΣ Β' λζ' τ.3/32

Page 35: Ευκλειδης Β 51

Η Homo Mathematicus είναι μια στήλη στο περιοδικό μας, με σκοπό την ανταλλαγή απόψεων και την ανάπτυξη προβλημαη­σμού πάνω στα εξής θέματα: I ) Τι είναι τα Μαθηματικά, 2) Πρέπει ή όχι να διδάσκονται, 3) Ποιοι είναι οι κλάδοι των Μαθημα­τικών και ποιο το αντικείμενο του καθενός, 4) Ποιες είναι οι εφαρμογές τους, 5) Ποιες επιστήμες ή κλάδοι επιστημών απαιτούν καλή γνώση των Μαθηματικών για να μπορέσει κάποιος να τους σπουδάσει. Για τους συνεργάτες της στήλης : παράκληση ! τα κείμενα της στήλης αυτής, ως προς το περιεχόμενό τους και ως προς το επ� πεδό τους, θα πρέπει να είναι συμβιβαστά με τα ενδιαφέροντα και το επίπεδο κατανόησης από μέρους των παιδιών.

Ι. "που και με ποιο τρόπο εφαρμό(ονται τα Λιfαθιzματικά,· " Θέμα: Μαθηματικά και Γλωσσολογία

Η μαθηματική γλωσσολογία είναι ένας κλάδος των Μαθηματικών, που αναπτύσσει ένα τυπικό "μηχανισμό" για την περιγραφή της δομής των φυσικών γλωσσών και ορισμένων τυπικών γλω σ­σών. Η μαθηματική γλωσσολογία παρουσιάστηκε στη δεκαετία του 1 950 σαν αποτέλεσμα της επιτ α­κτικής ανάγκης για την αποσαφήνιση βασικών ε ν­νοιών της γλωσσολογίας.

Η μαθηματική γλωσσολογία χρησιμοποιεί, κυ­ρίως την άλγεβρα, την θεωρία των αλγορίθμων και την θεωρία των αυτομάτων. Μολονότι δεν α­ποτελεί μέρος της γλωσσολογίας, ωστόσο έχει α­ναπτυχθεί σε στενή σχέση μαζί της. Μερικές φ cr ρές, ένας γλωσσολογικός κλάδος έρευνας, που χρησιμοποιεί μαθηματικά, ονομάζεται μαθηματική γλωσσολογία.

Η μαθηματική περιγραφή της γλώσσας βασίζ ε­ται στην έννοια της γλώσσας, κατά F. de Saussur, ως μηχανισμού, του οποίου η λειτουργικότητα " α­ποκαλύπτεται" (διαφαίνεται) από τον τρόπο ομιλίας αυτών, που την χρησιμοποιούν. Η ομιλία οδηγεί σε "ορθά κείμενα", δηλαδή ακολουθίες "προφορικών

ΙΙ . ''Α υτό το ξέρατε; ,

μονάδων" (speech units), που υπόκεινται σε καθcr ρισμένους νόμους, πολλοί από τους οποίους μπ cr ρούν να περιγραφούν μαθηματικώς. Η μελέτη των μεθόδων της μαθηματικής περιγραφής ορθών κε ι­μένων (κυρίως, προτάσεων) είναι κλάδος της μ α­θηματικής γλωσσολογίας και ονομάζεται θεωρία περιγραφικών μεθόδων συντακτικών δομών (τheory of Descήptiνe for Syntactic Structures ) .

Ένας άλλος κλάδος της μαθηματικής γλωσσ cr λογίας και, μάλιστα, κεντρικός γι' αυτή είναι η θεωρία των τυπικών γραμματικών, της cr ποίας ο κύριος εισηγητής είναι ο Ν. Chomsky. Ο Chomsky εξετάζει μεθόδους περιγραφής των ν cr μοτελειακών κανονικοτήτων, που χαρακτηρίζουν, όχι μόνο απομονωμένα κείμενα, αλλά το σύνολο των ορθών κειμένων σε μια διαδεδομένη γλώσσα.

Η μαθηματική γλωσσολογία ασχολείται επίσης και με την μελέτη αναλυτικών μοντέλων της γλώσσας. Η εφαρμογή της μαθηματικής γλωσσcr λογίας σε πραγματικές ( φυσικές) γλώσσες είναι μέρος της μελέτης της γλωσσολογίας. [πηγή : Με­γάλη Σοβιετική Εγκυκλοπαίδεια]

Ποιος έγραψε την παρακάτω επιστολή, στην οποία Cauchy είναι θεοπάλαβος . . . Ό,τι κάνει είναι έξοχο, α λ­εκφράζει τη γνώμη του για μια σειρά από μεγάλα μ α- λά και εξαιρετικά μπερδεμένο. Στην αρχή δεν καταν ο­θηματικά ονόματα του πρώτου μισού του l 9 °u αιώνα; ούσα ουσιαστικά τίποτα από όσα υποστήριζε. Τώρα Παραθέτουμε περικοπές από την επιστολή αυτή . αρχίζω να τον κατανοώ κάπως . . . Ο Cauchy είναι ο μ ό-

«Παρίσι, 24 Οκτωβρίου 1 826 νος που ασχολείται με τα καθαρά Μαθηματικά. Οι Για να σου πω την αλήθεια, η πιο πολύβουη πρωτε ύ- Poisson, Fouήer, Ampere, κλπ. είναι απασχολημένοι με

ουσα της ηπειρωτικής Ευρώπης είχε ως τώρα πάνω μου τον μαγνητισμό και άλλα θέματα Φυσικής. Ο κ. Laplace την επίδραση μιας ερήμου. Δεν γνωρίζω ουσιαστικά δεν γράφει τίποτε τώρα, όπως πιστεύω. Η τελευταία του κανέναν. Είναι άλλωστε η πιο όμορφη εποχή και όλοι εργασία ήταν ένα συμπλήρωμα στη Θεωρία του των βρίσκονται στην ύπαιθρο . . . Μέχρι τώρα γνωρίστηκα με Πιθανοτήτων. Τον συναντώ αρκετά συχνά στο Ινστ ι­τον κ. Legendre, τον κ. Cauchy και τον κ. Hachette και τούτο. Είναι πολύ εύθυμος τύπος. Ο κ. Poisson είναι μερικούς άλλους, λιγότερο γνωστούς αλλά πολύ ικ α- πολύ φιλικός · είναι ένας άνθρωπος που ξέρει να συμπε­νούς μαθηματικούς: τον κ. Saige, εκδότη του Bulletin ριφέρεται με μεγάλη αξιοπρέπεια. Το ίδιο θα μπορούσα des Siences και τον κ. Lejeune-Dirichlet, έναν Πρώσο να πω και για τον κ. Fouήer. Ο Lacroix είναι αρκετά που ήρθε να με δει πιστεύοντας ότι είμαι συμπατριώτης ηλικιωμένος. Ο κ. Hachette πρόκειται να με γνωρίσει σε του. Είναι ένας μαθηματικός μεγάλης οξυδέρκειας. Με μερικούς απ' αυτούς τους ανθρώπους. τον κ. Legendre έχει αποδείξει ότι είναι αδύνατη η λύση Έχω μόλις τελειώσει μια εκτενή μελέτη για μιαν ορ ι-της εξίσωσης x5+y5=z5 σε ακέραιους αριθμούς, αλλά σμένη κατηγορία υπερβατικών συναρτήσεων την οποία και πολλά άλλα έξοχα πράγματα. Ο Legendre είναι ε- θα παρουσιάσω στο Ινστιτούτο, που θα συνέλθει την ξαιρετικά ευγενής, αλλά δυστυχώς πολύ γέρος. Ο επόμενη Δευτέρα.

ΕΥΚΛΕΙΔΗΣ Β' λζ' τ.3/33

Page 36: Ευκλειδης Β 51

ΗΟΜΟ MATHEMAτiCUS

Έδειξα την εργασία μου στον κ. Cauchy, αλλά μόλις καταδέχθηκε να της ρίξει μια ματιά. Και τολμώ να πω, χωρίς καμιά διάθεση καυχησιολογίας, ότι είναι μια πο-

λύ καλή εργασία. Είμαι περίεργος να δω τι θα πουν στο Ινστιτούτο . . . . »

1 1 1 . "Οι συνεργάτεr;: τικ στιί).η;:: γράφουν-ερωτοι)ν"

Α Ό Μαθη ματική Λογοτεχνία Η στήλη μας, αλιεύοντας στο Διαδίκτυο, είχε την αγαθή τύχη να "πέσει" πάνω σε ένα δημοσίευμα της εφημερ�

δας «ΤΑ ΝΕΑ» (9/7/2003), ενός σημαντικού ανθρώπου, του Τεύκρου Μιχαηλίδη (μαθηματικός και μεταφραστής επιστημονικών βιβλίων). Εμείς τον ευχαριστούμε, για την ευκαιρία που μας δίνει να απολαύσουμε ένα κείμενο, όπου καταγράφεται ζωντανά μια άλλη πλευρά της Μαθηματικής Επιστήμης.

<< Η ιωθη μερινότητα των Μ αθηματικών», του Τεύκρου Μιχαηλίδη «Όταν το 1 993 κυκλοφόρησε το μυθιστόρημα

του Απόστολου Δοξιάδη «0 θείος Πέτρος και η εικασία του Γκόλντμπαχ» ουδείς υποπτεύθηκε ότι δεν είχαμε απλώς ένα νέο μυθιστόρημα, αλλά -στην ουσία - ένα νέο λογοτεχνικό είδος. Μάλιστα, το ίδιο το μυθιστόρημα πέρασε για αρκετό καιρό σχεδόν απαρατήρητο ώσπου να κάνει - το 2000 -τη δεύτερη, πλήρως αναθεωρημένη έκδοσή του και να γίνει μέσα σε λίγες ημέρες μπεστ σέλερ, διεθνώς. Σήμερα κυκλοφορεί σε 26 γλώσσες και αποτελεί, αν όχι το πρώτο χρονολογικά, το γνω­στότερο πάντως δείγμα μαθηματικής λογοτεχνίας.

Τα Μαθηματικά, εφιαλτική ανάμνηση των μα­θητικών χρόνων για τους περισσότερους -ακόμα και για πολλούς από αυτούς που στη μετέπειτα καριέρα τους διατήρησαν στενή επαφή μαζί τους -δεν προσφέρονται ως θέμα για τη μυθοπλασία. Οι έννοιές τους, λιτές, εξαιρετικά αφηρημένες και κυρίως μη επιδεχόμενες πολλαπλές ερμηνείες, δεύτερες αναγνώσεις, κρυμμένα νοήματα - το ψω­μοτύρι δηλαδή του λογοτέχνη - υποταγμένες στη σκληρή διχοτομία «αληθές - ψευδές» δεν συγκέ­ντρωσαν ποτέ την προτίμηση των υπηρετών του «ηδυσμένου λόγου)) . Όταν λέμε «ποτ6) εννοούμε φυσικά μέχρι την τελευταία πενταετία.

Στις μέρες μας κυκλοφορούν τουλάχιστον εκατό τίτλοι βιβλίων που διεκδικούν - δίκαια ή άδικα -τον τίτλο της μαθηματικής λογοτεχνίας, της λογο­τεχνίας δηλαδή στην οποία το κύριο θέμα, η πλο­κή, οι συναισθηματικές συγκρούσεις αναφέρονται στα Μαθηματικά. Αυτή η έξαρση αποτελεί ένα θέμα προς διερεύνηση, τόσο από πολιτιστική όσο και από κοινωνική και ιστορική σκοπιά.

Για όλους τους προαναφερθέντες λόγους παρα­κολουθήσαμε με ιδιαίτερο ενδιαφέρον τη δεύτερη μαθηματικο-λογοτεχνική δημιουργία του Απόστο­λου Δοξιάδη, το θεατρικό έργο «lncompleteness)) («Μη πληρότητω)). Το έργο αναφέρεται στις τε­λευταίες ημέρες του κορυφαίου μαθηματικού Kurt Goedel ( 1 906 - 1 978), ο οποίος χάρη στις συντα-

ρακτικές ανακαλύψεις του στον τομέα της Μαθη­ματικής Λογικής, χαρακτηρίστηκε ως ο «νέος Α­ριστοτέληφ.

Στο Δεύτερο Διεθνές Συνέδριο Μαθηματικών, που έγινε τον Αύγουστο του 1 900 στο Παρίσι, ο κορυφαίος μαθηματικός Daνid Hilbert ( 1 862-1 943) παρουσίασε 23 προβλήματα που, κατά τη γνώμη του, θα αποτελούσαν το επίκεντρο της μα­θηματικής δραστηριότητας κατά τον 20ό αιώ­να.Από αυτά, τα δύο πρώτα αφορούσαν στη θεμε­λίωση των μαθηματικών και τη μαθηματική λογι­κή . Συγκεκριμένα το δεύτερο ζητούσε να αποδει­χθεί ότι η δομή του συνόλου των φυσικών αριθ­μών είναι πλήρης και χωρίς αντιφάσεις.

Στο αίτημα αυτό του Hilbert, ο Goedel απάντησε το 1 93 1 με το θεώρημα της μη πληρότητας (που έδωσε και τον τίτλο του και στο έργο του Δοξιά­δη) . Με απλά λόγια, το θεώρημα του Goedel λέει ότι είναι αδύνατο να αποδειχθεί ταυτόχρονα η πληρότητα και η μη αντιφατικότητα του συνόλου των φυσικών αριθμών, αφού σε μια μαθηματική θεωρία θα υπάρχουν πάντα προτάσεις για τις οποί­ες δεν θά μπορεί να αποδειχθεί ούτε ότι είναι αλη­θείς ούτε ότι είναι ψευδείς.

Με την ανακάλυψή του ο Goedel προκάλεσε συ­γκλονιστικές αναταράξεις στον χώρο των μαθημα­τικών, αφού ανέτρεψε ολόκληρη την αντίληψη και τον σχεδιασμό του Hilbert για τα Μαθηματικά. Ο Goedel συνέχισε τη μαθηματική του σταδιοδρομία ανακαλύπτοντας και άλλα πολύ σημαντικά θεω­ρήματα στον τομέα της Μαθηματικής Λογικής. Πέθανε στις 14 Ιανουαρίου 1 978, ύστερα από πα­ραμονή 1 7 ημερών στο νοσοκομείο, επειδή αρνή­θηκε να πάρει οποιαδήποτε τροφψ)

Ση μείωση Στο επόμενο τεύχος ευελπιστούμε να έχουμε τη χαρά να δημοσιεύσουμε ένα άρθρο του Απόστολου Δοξιάδη στο οποίο θα φαίνεται η άπο­ψή του για τις μαθηματικές σπουδές.

Β Ό ' ' Απόψεις" Ο σχολικός σύμβουλος Νίκος Βαδιβούλης (είναι ένας σημαντικός φίλος της στήλης και, προ πάντων, σκεφτόμενος άνθι»­

πος) μας έστειλε δύο κείμενά του με αντίστοιχους τίτλους «Υψώνω-υπερασπίζομαι» και «Αναζητώντας τη γλώσσα της φύσης». Πραγματικά, βρεθήκαμε μπροστά σε μεγάλο δίλημμα ποια από τις δύο εργασίες να δημοσιεύσουμε. Καταλήξαμε στην πρώτη

ΕΥΚΛΕΙΔΗΣ Β' λζ' τ.3/34

Page 37: Ευκλειδης Β 51

ΗΟΜΟ MATHEMAτiCUS

μόνο και μόνο γιατί ήταν μικρή και χώραγε στον διαθέσιμο χώρο της στήλης. Υπενθυμίζουμε ότι, οι όποιες θέσεις των συ-γγρα­φέων, απηχούν τις προσωπικές τους θέσεις και σαν τέτοιες τίθενται στην κρίση των αναγνωστών.

«υψώνω - υπερασπίζομαι», του Νίκου Βαδιβούλη (Πρέβεζα) «0 Σταγειρίτης φιλόσοφος Αριστοτέλης είχε μια

αντιπάθεια προς τους σοφιστές, αφού σε πολλές περιπτώσεις αυτοί έπειθαν τους ανθρώπους το κ α­ταμεσήμερο ότι ενέσκηψε πυκνό σκοτάδι. Όμως κανείς δεν αμφισβητεί ότι η πρακτική διεξαγωγής δημοσίων διαλεκτικών μονομαχιών είχε θετικά αποτελέσματα, αφού με ακανόνιστη επιχειρημ α­τολογία ο κάθε ένας των μονομάχων προσπαθούσε να καταρρίψει αυτό που ο έτερος υποστήριζε. Ο Αριστοτέλης έθεσε .κανόνες στη λογική επιχειρ η­ματολογία και για το λόγο αυτό θεωρείται ο θεμ ε­λιωτής της μαθηματικής λογικής. Πίστευε ότι οι σκοποί της λογικής θεώρησης του καθημερινού γίγνεσθαι διαφέρουν από αυτούς της ρητορικής, γιατί η μεν ρητορική τεχνική θέλει να πείσει, η δε Λογική επιστήμη διδάσκει την έγκυρη γνώση μέ­σω της απόδειξης, που είναι το μεγαλύτερο μέχρι σήμερα ανθρώπινο επίτευγμα και ελληνική επ ι­νόηση.

Σήμερα η δίτιμη μαθηματική λογική δέχεται μια συνεπαγωγή ως ψευδή τότε και μόνο τότε, όταν η υπόθεση είναι αληθής και το συμπέρασμα ψευδές. Είναι γνωστή η ιστορία, ο Ράσελ και ο Πάπας, κα­τά την οποία ο φιλόσοφος Ράσελ, με τη βοήθεια συνεπαγωγής που του δόθηκε, αποδεικνύει ότι μια ψευδής πρόταση συνεπάγεται οποιαδήποτε πρότα­ση. Σαν παράδειγμα λοιπόν ας ασχοληθούμε με την επόμενη συνεπαγωγή : Αν υψώνω τη σημαία τότε την υπερασπίζομαι. Όπως αντιλαμβάνεστε τα ρήματα υψώνω, υπερασπίζομαι, καθορίζουν την αλήθεια ή όχι της προηγούμενης συνεπαγωγής. Έτσι όταν το υψώνω εκφράζει το πραγματικό και

το υπερασπίζομαι είναι μη παραδεκτό, τότε η προηγούμενη συνεπαγωγή είναι, μόνο σ' αυτή την περίπτωση, ψευδής άρα και υποκριτική όταν πραγματοποιηθεί.

Σήμερα τα περισσότερα κράτη υιοθετούν την πολιτισμική διαφοροποίηση και αναγνωρίζουν ότι οι πολύ-πολιτισμικοί και πολυεθνικοί πληθυσμοί αποτελούν δημοκρατική εγγύηση για την κοινων ι­κή ολοκλήρωση . Είναι ιστορικά βεβαιωμένο ότι δεν απορρίπτονται βιαίως κουλτούρες, γλώσσες, θρησκείες, παραδόσεις μέσω του κοινωνικού σ ω­φρονισμού, αφού τα νάματα σοφίας που εγκλείουν οι άνθρωποι δεν ξεριζώνονται ούτε με την εκ νέου ανάπλασή τους. Απομένει λοιπόν οι μετέχοντες της ελληνικής παιδείας να αισθάνονται υπερήφανοι όταν υψώνουν τα ελληνικά σύμβολα αλλά και να τα τιμούν, έστω και αν δεν θα τα υπερασπισθούν σε ειδικές περιπτώσεις.

Ο θόρυβος στοχεύει στην απαξίωση της πολιτ ι­κής ζωής του τόπου, οπότε πρέπει να απέχουμε των σοφιστικών τεχνικών, γιατί εκείνο που παρ έ­χει ωφελήματα είναι η σωστή παιδαγωγία μέσω της οποίας διαπλάθουμε κριτικά ιστάμενους πολ ί­τες, ώστε να μπορούν να διακρίνουν την έρπουσα φημολογία από το καλό κ' αγαθό. Αν οι άνθρωποι έκαναν πράξη το αρχαίο λόγιο «ο συ μισείς ετέρω μη ποιήσεις» και είχαν κατακτήσει τον τρόπο του σκέπτεσθαι τότε μέσω των πολύ-πρισματικών ε­ντρυφημάτων θα εδραίωναν την αρμονία στην ο ι­κογένεια, τη δημιουργικότητα στην κοινωνία, την τάξη στην πολιτεία, τη φιλία μεταξύ των ανθρ ώ­πων και την ειρήνη στον κόσμσ».

Γ. Μια άλλη λύση στο «Γεωμετρικό Ναπολεόντειο Πρόβλημα» Από τον εκλεκτό συνάδελφο-μαθηματικό Θανάση Δ. Γκίκα πήραμε μια νέα λύση στο πρόβλημα της

εύρεσης της ακτίνας και του κέντρου ενός δοσμένου κύκλου, χρησιμοποιώντας μόνο χάρακα και διαβή­τη, που δημοσιεύσαμε στο τεύχος 49 στη στήλη Homo mathematicus. Τον ευχαριστούμε.

Ο καλός συνάδελφος μας λέει: «Αν ο Γ. Κερασαρίδης ήταν φοιτητής το '59-'63 , ήμασταν τότε συμφο � τητές». Του απαντούμε: Ναι, σωστά θυμάται, και τον χαιρετούμε θερμά.

Δ '. Ένα πολύ-πολύ παράξενο φαινόμενο Τον τελευταίο καιρό κάποιος συνάδελφος στέλλει ανώνυμες προϊόντα γνήσιου "σχολαστικισμού", β) δηλώνουμε πως θεω­επιστολές προς τη συντακτική επιτροπή του περιοδικού (οι ρούμε απρέπεια το να ασκείς την όποια κριτική κρυμμένος υπεύθυνοι της στήλης έλαβαν γνώση μόνο της τελευταίας πίσω από την ανωνυμία. Να ρωτήσουμε τον "καλό" μας ανω­επιστολής), μέσα από τις οποίες ασκεί κριτική (στηλιτεύει θα νυμογράφο, γιατί φορά την κουκούλα κάθε φορά που εμφα\1. έλεγα) στα όποια λάθη ή ανεπάρκειες του περιοδικού. Για τη ζεται στο φως της ημέρας; Ποιον ή τι φοβάται; Κρίμα, γιατί Homo mathematicus η όποια κριτική είναι ευπρόσδεκτη και "δείχνει" να "ξέρει γράμματα" . Αν μας απαντήσει επώνυμα, δημοσιεύσιμη. Για το λόγο αυτό στο επόμενο τεύχος θα τοπο- εμείς υποσχόμαστε πως θα δημοσιεύσουμε την όποια απάντrr θετηθούμε αναλυτικά για τα σημεία που μας αφορούν. Προκα- σή του, διαφορετικά δεν θα υποθάλψουμε κουκουλοφόρους . . . ταβολικά: α) θεωρούμε, τις παρατηρήσεις που μας αφορούν,

Ιlα. "Αυτό το ξέοατεj " [η απάντηση] Την επιστολή αυτή έγραψε ο Niels Henrik Abel ( 1 802-1 829). Με νεώτερο σημείωμα, θα επανέλθουμε στο έργο αυτού του γ}.

γαντα της μαθηματικής επιστήμης [πηγή:Ε.Τ. BELL "Οι μαθηματικοί" , τ. 11, σσ.42-43 , Πανεπιστημιακές Εκδόσεις Κρήτης]

ΕΥΚΛΕΙΔΗΣ Β' λζ' τ.3/35

Page 38: Ευκλειδης Β 51

«Ολυμπιακές Προσεγγίσεις>> Σχέση ισοδυναμίας

1 . Εστω σύνολο Ε :;t 0 . Έστω ένα σύνολο δει­κτών Ι (πιθανόν Ι = Ν ή Ι ς Ν ). Κάθε αντι­στοιχία του Ι εντός του Ε ονομάζεται οικογέ­νεια στοιχείων του Ε με δείκτες από το Ι και συμβολίζεται Εϊ, i Ε Ι ή απλά Ε;. Προφανώς κ οικογένειες με κ Ε Ν* είναι οι Ει Ε2, . . . , Εκ.

2. Έστω σύνολο Ε :;t 0 .Έστω σύνολο δεικτών I. Έστω Ε;, i Ε Ι οικογένειες με στοιχεία του Ε. Αν UE; = Ε και ταυτόχρονα για κάθε i, j Ε Ι

iεl

με i :;t J,E; n Ej = 0 (δηλαδή ανά δύο τα E; , i Ε Ι είναι ξένα μεταξύ τους) λέμε ότι τα Ε; ,i Ε Ι ορίζουν μία διαμέρίση επί του Ε.

3 . Έστω σύνολο Ε :;t 0 . Διμελής σχέση σ εντός του Ε ονομάζεται κάθε υποσύνολο του καρτε­σιανού γινομένου Ε χ Ε. Άρα το πεδίο ορισμού μίας διμελούς σχέσης σ( χ, y) ή χσy είναι υπο­σύνολο του Ε χ Ε.

4. Έστω σύνολο Ε :;t 0 . Μία διμελής σχέση σ στα στοιχεία του Ε που ορίζεται από ένα νόμο χσy ή σ(χ, y) (ο οποίος καθίσταται πρόταση � ταν τα χ, y αντικατασταθούν από συγκεκριμέ­να στοιχεία του Ε) και για τον οποίο αληθεύ­ουν ταυτόχρονα οι προτάσεις: (α) για κάθε χ Ε JR (χσχ) (αυτοπαθής ή α­

νακλαστική) (β) · για κάθε (χ, y) E ExE (χσy => yσχ) (συμ­

μετρική) (γ) για κάθε χ Ε Ε, yEE, zEE (χσy και yσz

=> χσz) (μεταβατική) ονομάζεται σχέση ισοδυναμίας.

5. Παραδείγματα 1 . Η ισότητα είναι σχέση ισοδυναμίας (το

αντίστροφο δεν ισχύει πάντα) 2 . Έστω το σύνολο των τριγώνων ενός επι­

πέδου. Ορίζουμε τη σχέση σ ως εξής: χσy � "χ ισοεμβαδικό y" .

Τότε η σ είναι σχέση ισοδυναμίας. 3 . Έστω Ζ το σύνο'UJ των ακεραίων αριθμών.

Έστω ν ένας σταθερός ακέραιος αριθμός. Ορίζουμε τη σχέσης α = β(mοdν) αν και μόνο αν α - β = κ · ν, κ Ε Ζ . Η σχέση αυ­τή είναι σχέση ισοδυναμίας και μάλιστα ο­νομάζεται αριθμητική ισοδυναμία.

Παρατήρηση : Έστω σύνολο Ε, σ μία σχέση ισοδυναμίας εντός του Ε και Χο στοιχείο του Ε. Ό­λα τα στοιχεία του Ε, που είναι ισοδύναμα προς το χ0 ως προς την σ αποτελούν ένα υποσύνολο του Ε

Σωτήρης Λουρίδας που το ονομάζουμε τάξη ισοδυναμίας ή κλάση ι­σοδυναμίας. 6. Βασικό Θεώρημα

Σε κάθε σχέση ισοδυναμίας σ μέσα σε ένα σύνολο Ε αντιστοιχεί μία διαμέριση και αντί­στροφα, σε κάθε διαμέριση του Ε αντιστοιχεί μία σχέση ισοδυναμίας.

Απόδειξη : Θεωρούμε ότι έχει ορισθεί μέσα στο Ε μία σχέση ισοδυναμίας σ. Έστω Εχο το σύνο'UJ των στοιχείων του Ε που είναι ισοδύναμα προς ένα στοι­χείο Χο του Ε. Υποθέτουμε την ύπαρξη και ενός άλ­'λλJυ στοιχείου χΌ του Ε και Εχ·ο το σύνο'UJ των στοι­χείων του Ε που είναι ισοδύναμα στο χΌ.

Τότε Χ Ε Εχο => χ σ Χο. Αν χ σ χΌ τότε χ'ο σ χ => Εχο ς Εχ·ο· Κατά τον ίδιο τρόπο έχουμε ΕΧ'ο ς Εχο · Άρα Εχο = Εχ·ο· Αν χ0%Χ� και Εχο n Εχ·ο :;t 0 θα υπάρχει

X E Exo n Ex·o τότε χ σ Χο και χ σ χΌ => Χο σ χΌ, ά­τοπο. Έτσι λοιπόν στη σχέση ισοδυναμίας σ αντι­στοιχεί η διαμέριση Έη όπου το η αντιπροσωπεύει τα στοιχεία του Ε που δεν είναι ισοδύναμα ,μεταξύ τους ως προς τη σχέση σ.

Α ντίστροφα: Έστω Ε;, i Ε Ι μία διαμέριση του Ε. Ορίζουμε μία σχέση σ ως εξής: χσy � χ και y βρίσκονται στο ίδιο Ε; , i Ε I. Προφανώς η χσy εί­ναι ανακλαστική (χσχ), συμμετρική αφού για κάθε χ, y EE; (χσy => yσχ), μεταβατική αφού για κάθε x,y,z E E; χσy και yσz => χ, z E E; => χσz). Άρα έ­χουμε σχέση ισοδυναμίας τα δε στοιχεία της δια­μέρισης είναι οι τάξεις ισοδυναμίας.

Παρατ.ήρηση : Θεωρούμε αΕΕn τότε το En εί­ναι το σύνολο των στοιχείων του Ε που είναι ισο­δύναμα προς το α ως προς την σχέση ισοδυναμίας που ορίσαμε. Δηλαδή το τυχαίο στοιχείο χ ε Ea έχει την «ίδια συμπεριφορά)) με το α ως προς τη σχέση ισοδυναμίας που ορίσαμε. Αυτό έχει μεγάλη σημασία διότι ομαδοποιώντας τα στοιχεία ενός συνόλου Ε με βάση τη σχέση ισοδυναμίας τα με­λετάμε μέσω των αντιπροσώπων τους.

Παράδειγμα 1 Έστω κυρτό πολύγωνο ν-πλευρών με περί-

μετρο ν..fi . Να αποδειχθεί ότι υπάρχει ένα του­λάχιστον μη κένο σύνολο τριγώνων με κορυφές τρείς από τις κορυφές του πολυγώνου, εμβαδού μικροτέρου ή ίσο τουl.

Λύση : Έστω Τ το σύνολο όλων των τριγώνων με κορυφές τρείς από εκείνες του πολυγώνου. Η σχέση χσy= (χ ισοεμβαδικό με y), όπου x,yET εί-

ΕΥΚΛΕΙΔΗ Β' λζ' τ.3/36

Page 39: Ευκλειδης Β 51

-------------- «Ολυμmακές προσεyγίσεις» -------------

ναι σχέση ισοδυναμίας, αφού ισχ6ουν οι ιδιότητες χσχ (ανακλαστική), χσy= yσχ (συμμετρική), χσy και yσz => χσz (μεταβατική). Αυτή η σχέση δη­μιουργεί μια διαμέριση. Έτσι χωρίζεται το Τ σε κλάσεις ισοδυναμίας. Άρα αν τουλάχιστον ένα τρί­γωνο με κορυφές τρείς από τις κορυφές του πολυ­γώνου έχει εμβαδόν μικρότερο ή ίσο του 1 , τότε η κλάση στην οποία ανήκει είναι το ζητούμενο � νολο. Έστω λοιπόν, ότι όλα τα τρίγωνα έχουν εμ­βαδόν μεγαλύτερο του 1 .

Δ Για τυχαίο τρίγωνο ΑΒΓ έχουμε:

ΒΔ<ΑΒ άρα ΒΔ · ΑΓ < ΑΒ · ΑΓ ή Μ· ΑΓ ΑΒ· ΑΓ , (ABn ΑΒ· ΑΓ , 1 ΑΒ · ΑΓ -- <-- η 'A J < η < ---2 2 2 2

ή J2 < 2 -./ΑΒ · ΑΓ ή 2J2 < ΑΒ + ΑΓ 2 ( 1 ) (αφού για 2νΓαβ � α + β .

α, β Ε IR , με α,β>Ο, έχουμε

Α

Β Από την ( 1 ) προκύπτει ότι:

Α1Α2 + Α2Α3 > 2J2 Α2Α3 + Α3Α4 > 2J2

ΑvΑι + Α1Α2 > 2J2 Με πρόθεση κατά μέλη έχουμε:

Γ

2(A1Az +AzA3 + . . . +�Α1 ) > 2νJ2 ή 2νJ2 > 2νJ2 άτοπο. Άρα θα υπάρχει ένα τουλάχιστον τρίγωνο με κορυφές τρείς από εκείνες του ν-γώνου με εμ­βαδόν μικρότερο ή ίσο του 1 το οποίο δημιουργεί συγκεκριμένη κλάση.

Παράδειγμα 2 : Να αποδειχθεί ότι κάθε κλάση υπολοίπων

modu1o ι3 είναι ένωση 3 κλάσεων υπολοίπων modulo 39.

Λύση : Γνωρίζουμε (από την θεωρία αριθμών) ότι το σύνολο Ζ των ακραίων διαμερίζεται σε κλάσεις (ή τάξεις) ισοδυναμίας με βάση την σχέση ισοδυναμίας α = βmod(n)(� η / α - β) . Παρατη-ρούμε ότι 1 3/39 αφού 39 = 3 · 1 3 . Από α = β mod(39) έχουμε α = β mod(1 3) .

Άρα κάθε κλάση ισοδυναμίας modulo 39 είναι υποσύνολο μιας κλάσης ισοδυναμίας modulo 13 . Άρα η τυχαία κλάση ισοδυναμίας modulo 1 3 είναι

ένωση πεπερασμένου πλήθους κλάσεων modulo 39. Θεωρούμε την κλάση χ = Χ0 mod(l 3) τότε ot

ακέραιοι Χ0 , Χ0 + 1 3, Χ0 + 26 ανήκουν στην κλάση Χ0 mod(1 3) . Αυτοί δεν είναι ισοδύναμα στοtχεία ως προς την σχέση ισοδυναμίας mod 39. Αυτό ε­πειδή Χ0 + 1 3 · i = (χ0 + 1 3 · i 1)mοd39(με i = 0, 1, 2) ή 39 /(i - i 1) · 1 3 � 3 / i - i 1 (αφού 1 3 · 3 = 39 ) ή i = i 1 mod 3 ή i = i 1 ( αφού Ο � i και Ο � i 1 � 2 ) ή i = i 1 ή Χ0 + ί · m1 = Χ0 + i 1" m1 •

Έστω τώρα α ένας τυχαίος ακέραιος της κλά­σης Χ0 mod(1 3) τότε α = Χ0 + 1 3 · k με k Ε Ζ .

Άρα α = Χ0 + 1 3k = α + (3π + υ) · 1 3 = = Χ0 + 3π · 1 3 + 1 3υ = Χ0 + 1 3 · υ + 39π

ή α = (χο + 3υ)mod39 με Ο � υ � 3 . Οπότε κάθε κλάση υπολοίπων modu1o 1 3 εί-

ναι η ένωση 3 κλάσεων ισοδυναμίας mod39. Τρία προβλήματα Πρόβλημα ι Θεωρούμε συνάρτηση f : IR � IR με την ι:..

διότητα [r<x + ι) y - r(x2 + 2χ) = ι (ι) rια κάθε χ Ε IR . Να αποδείξετε ότι το πεδίο τιμών της δεν είναι υποσύνολο των ρητών αριθμών.

Λύση : Η συνάρτηση f : IR � IR με f(x) = x έχει την ιδιότητα ( 1 ) αφού

[ f(x + 1) ]2 = (χ + 1)2 = χ2 + 2χ + 1 και f(x2 + 2χ) = χ2 + 2χ οπότε [f(x + 1)]2 - f(x2 + 2χ) = 1 . Άρα υπάρχει μια τουλάχιστον συνάρτηση f

με την ιδιότητα ( 1 ) και f : IR � IR . Γενικά λοιπόν, έχουμε την εξής λύση :

Θεωρούμε χ2 + 2χ = χ + 1 � χ2 + χ - 1 = 0 με

Δ = (+1)2 - 4 · 1(- 1) = 5 οπότε χ1,2 = - 1�.J5 . Άρα {-1 -.JS -1 + .J5} για χ0 Ε , έχουμε: 2 2

Υ ο = f(x0 + 1) = f(x� + 2χ0 ) . Από την ισχ6 της σχέσης ( l ) έχουμε y� - y0 = 1 � y� - y0 - 1 = 0 με

2 { 1 - .JS 1 + .J5} Δ = (-1) - 4 · 1 · (- 1) = 5 => y0 Ε -2-, -2-που δεν είναι υποσύνολο των ρητών αριθμών Q.

Παρατήρηση : Έστω η συνάρτηση f(x) = x /IR τότε f(x2 + 1) * f(x) για κάθε χ Ε IR διότι αν f(x2 + l) = f(x)=> x2 + l = x=>x2 - χ+ 1 = 0 με Δ = -3 < Ο . Άρα δεν υπάρχει χ Ε IR ώστε f(x2 + 1) = f(x) . Συνεπώς θεωρείται αναγκαία η

ΕΥΚΛΕΙΔΗ Β ' λζ' τ.3/37

Page 40: Ευκλειδης Β 51

-------------- «Ολυμmακές προσεyyίσεις»

θεώρηση χ 2 + 2χ = χ + 1 ώστε να υπάρχει το χ0 με την ιδιότητα οι αντίστοιχες τιμές f(x� + 2χ0 ), f(x0 + 1) να είναι ίσες και να μπο­ρούν να αντιπροσωπευθούν από μια τιμή Υ ο = f(x0 + 1) = f(x� + 2χ0 ) (παρατηρούμε ότι οι

, 1 - JS 1 + J5 δ , , τι μες -2-, -2- εν ανηκουν σε προφανεις

τιμές για την ισχύ της ισότητας χ� + 2χ0 = χ0 + 1 ) .

Πρόβλημα 2 Δίνεται ευθ. Τμήμα ΑΒ. Θεωρούμε το σύ­

νολο των σημείων Μ?Β με την ιδιότητα Λ

ΑΜΒ = ω όπου ω δοθείσα γωνία ώστε:

Ο < ω < π . Για το τυχαίο σημείο Μ του σημειο-2

συνόλου αυτού θεωρούμε Τ την προβολή του μέσου Ρ του ΜΒ πάνω στην ΜΑ. Να αποδείξε-

, λό ΜΒ ·ΜΤ , θ , τε οτι ο γος ΒΤ2 παραμενει στα ερος α-

νεξάρτητα από την θέση του σημείου Μ ώστε ΑΜΒ = ω με M :;t: B .

Λύση : Γνωρίζουμε ότι το σύνολο των σημεί­ων Μ είναι τόξο κύκλου (αν και ο μη προσδιορι­σμός αυτός δεν επηρεάζει την λύση του προβλή-

Δ ματος) Τα ορθογώνιο τρίγωνο ΤΜΡ παραμένει

Δ όμοιο προς τον εαυτό του αφού η γωνία ΤΜΡ = ω

θ ' Ά λ ' ΜΡ ' θ ' Ε στα ερη . ρα ο ογος ΜΤ ειναι στα ερος. πο-

2ΜΡ μένως και είναι σταθερός άρα και ο 2ΜΤ ΜΡ = λ (λ σταθερός) . ΜΤ

Δ Το τρίγωνο λοιπόν ΜΤΒ παραμένει όμοιο

προς τον εαυτό του αφού ΤΜΒ = ω δεδομένη και ΜΒ λ θ , Ά λ ' ΜΒ ΜΤ , - = στα ερο. ρα οι ογοι -, - ειναι ΜΤ ΒΤ ΒΤ σταθεροί δηλαδή ΜΒ = k1 , ΜΤ = k2 όπου kι ,kz ΒΤ ΒΤ

θ , θ , , ΜΒ · ΜΤ k k θ στα εροι αρι μοι οποτε ΒΤ2 = 1 • 2 στα ε-ρό σε όλη την κίνηση του Μ.

Παρατήρηση 1 : Το πρόβλημα γενικεύεται με σημείο Ρ της ΜΒ ώστε κ · ΜΡ = λ · ΡΒ όπου κ,λ σταθεροί θετικοί αριθμοί. Επίσης με ΡΤ να σχημα­τίζει δεδομένη γωνία με την ευθεία ΜΑ.

Παρατήρηση 2: Λύνεται και τριγωνομετρικά. Πρόβλημα 3 Δίνεται ισοσκελές τρίγωνο ΑΒΓ (ΑΒ=ΑΓ)

και σημείο Δ της ΒΓ, μεταξύ των Β και Γ, ώστε ΔΓ=9ΒΔ. Για το μοναδικό σημείο Σ της ΑΔ �

Λ

στε ΒΣΓ = 90° + Α να αποδειχθεί ότι: ΣΓ=3ΣΒ. 2

Λύση : i) Αν θεωρήσουμε κύκλο (Κ, R) που να εφάπτεται στις ΑΒ, ΑΓ στα σημεία Β και Γ τότε το σημείο τομής της με την ΑΔ είναι το σημείο Σ που είναι μονοσήμαντα τα ορισμένα αφού κάθε σημείο Σ ' * Σ .

Α

κ

της ΑΔ δίνει πολύ εύκολα ΒΣ τ > ΒΣΓ ή ΒΣΤ < ΒΣΓ , Β1 = fΊ , Β2 = fΊ οπότε

� � � � Α � Α � Α Β1 + Β2 = Γι + Γ2 = Γ = Β = 90° - - => ΒΣΓ = 90° +-2 2

ii) (ΒΔΣ) ΒΔ · ΒΣ (ΒΣΑ) ΒΣ· ΒΑ ΑΒ= ΑΓ (ΓΑΣ) ΓΣ· ΓΑ' (ΓΔΣ) ΓΣ· ΓΔ ' .

Με πολλαπλασιασμό κατά μέλη των δύο πρώτων σχέσεων έχουμε:

(ΒΔΣ) · (ΒΣΑ) (Γ ΑΣ) · (Γ ΔΣ)

Έχουμε επίσης

= ΒΣ2 · ΒΔ ( 1 ) . ΓΣ2 · ΓΔ (ΒΔΣ) ΒΔ

-- = - και (ΓΔΣ) ΔΓ

(ΒΣΑ) ΒΒ ' ΒΔ = - = - οπότε με πολλαπλασιασμό (ΓΣΑ) ΓΓ ΔΓ

(ΒΔΣ) · (ΒΣΑ) ΒΔ2 κατά μέλη έχουμε: = (2) (ΓΑΣ) · (ΓΔΣ) ΓΔ2 Από τις σχέσεις (1) και (2) έχουμε:

ΒΣ2 · ΒΔ ΒΔ2 ΒΣ2 ΒΔ ΒΣ2 1 --- = -- � -- =- � -- = -ΓΣ2 · ΓΔ ΓΔ2 ΓΣ2 ΓΔ ΓΣ2 9

� ΒΣ = .!. � ΓΣ = 3ΒΣ ΓΣ 3

ΕΥΚΛΕΙΔΗ Β' λζ' τ.3/38

Page 41: Ευκλειδης Β 51

'τάξη τ u

rpαμμικ6 ιJutnιfμστα c{ιcτώcτcων α- �σιμού

Α. ΓΡΑΜΜΙΚΉ ΕΞΙΣΩΣΗ ΜΕ ΔΥΟ ΑΓΝΩΣΤΟΥΣ

Γραμμική εξίσωση με δύο αγνώστους ονομάζεται κάθε εξίσωση της μορφής: lα · χ + β · ψ = γ l (ι), με α,β,γ ε !R π.χ. 2 . χ - ψ = ι , χ = 2, ψ = 3 , κλπ. Λύσεις της εξίσωσης αυτής (αν υπάρχουν) είναι κάθε ζεύγος αριθμών (χ,

ψ) που την

επαληθεύει. Παράδειγμα: Η εξίσωση : 2χ - ψ = ι έχει άπειρες λύσεις π. χ. τα ζεύγη : (1, 1), (2, 3), . . . Όλα τα ζεύγη αυτά απει­κονιζόμενα στο καρτεσιανό επίπεδο παρίστανται με τα σημεία της ευθείας: ε: ψ = 2χ - ι δηλαδή την γραφική παράσταση της συνάρτησης f(x) = 2χ - ι με χ Ε IR .

Υ

χ

Ας δούμε όμως τις περιπτώσεις των γραμμικών εξισώσεων με τη σειρά:

1 η περίπτωση :

Α ν α * Ο ή β * Ο , δηλαδή αν οι συντελεστές α,β δεν είναι συχρόνως μηδέν, τότε:

Κώστα Βακαλόπουλου

D Αν β * Ο τότε η εξίσωση ( 1) γίνεται:

β · ψ � -αχ + γ <ο> Ι ψ � -t · χ +tl και txf;ι

λύσεις όλα τα ζεύγη των αριθμών (χ, ψ

) που είναι συντεταγμένες των σημείων της ευθείας ε,

α ./ με συντελεστή διεύθυνσης: λ = -- και που

β Υ

'I

./ τέμνει τον άξονα ψψ'

στο σημείο Β( o,t) . Λέμε τότε ότι η ευθεία ε έχει εξίσωση την:

α · χ+β ·ψ= Ύ ή ότι η εξίσωση α · χ + β · ψ = 'f παριστάνει την ευθεία ε.

Προσοχή ! Αν β * Ο και α = Ο τότε η εξίσωση ( 1 ) γίνε-

ται: Ι ψ

= tl και παριστάνει εuθεiα παράλληλη στον

άξονα χχ ' . Υ

Β ε

χ

ΕΥΚΛΕΙΔΗΣ Β' λζ' τ.3/39

Page 42: Ευκλειδης Β 51

Μαθηματικά για την Α' Λυκείου

Ο Α ν β = Ο τότε α * Ο και η εξίσωση ( 1) γίνεται:

Ι χ = �� και παριστάνει ευθεία παράλληλη στον άξονα ψψΌ

Υ ε

χ

Α

2η περίπτωση :

Αν α = Ο και β = Ο τότε η εξίσωση (1) γίνεται: Ο · χ + Ο · ψ = γ , (2) οπότε: ./ Αν γ = Ο τότε η εξίσωση (2) γίνεται

Ο . χ + Ο · ψ = Ο επαληθεύεται από οποιοδήποτε ζεύγος αριθμών και παριστάνουν όλο το επίπεδο.

./ Αν γ * Ο τότε η εξίσωση (2) γίνεται Ο · χ + Ο · ψ = γ και δεν επαληθεύεται από κανένα ζεύγος αριθμών είναι δηλαδή αδύνατη και παριστάνει το κενό σύνολο.

ΣΥΜΠΕΡ ΑΣΜΑ ΤΑ Μια γραμμική εξίσωση α · χ + β · ψ = Ο , (με δυο αγνώστους) παριστάνει ευθεία μόνο αν οι συντε­λεστές α και β δεν μηδενίζονται συγχρόνως δηλα­δή αν α * ο ή β * ο ή ισοδύναμα: lαl + ιβ ι * ο .

Παράδειγμα:

Η εξίσωση κ · χ + (κ + 1) · ψ = 3 αριστάνει ευθεία για κάθε κ Ε JR αφού τα κ, κ + 1 δεν μηδενίζονται συγχρόνως για καμία πραγματική τιμή του κ.

Παρατήρηση : Ο καλύτερος τρόπος για να αποδώσουμε το ,σύνολο λύσεων μιας γραμμικής εξίσωσης: α · χ + β · ψ = Ο με α * Ο ή β * Ο είναι με τις συντεταγμένες των ση�ν της ευθείας που παριστάνει.

Β. ΣΥΣΤΉΜΑΤΑ ΓΡΑΜΜΙΚΩΝ ΕΞΙΣ� ΣΕΩΝ ΜΕ ΔΥΟ ΑΓΝΩΣΤΟΥΣ

Σύστημα δύο γραμμικών εξισώσεων με δύο α­γνώστους ον�μάζουμε δυο γραμμικές εξισώσεις μαζί, των οποίων ζητάμε, αν υπάρχουν, τις κοινές , , {αι · χ + βιψ = γι λυσεις π.χ. το συστημα (Σ) : β

με α2 · χ + 2ψ = γ2 αι , β ι , γι , α2, β2, γ2 Ε 1R . Επίλυση ενός γραμμικού συστήματος είναι η δια­δικασία εύρεσης, αν υπάρχουν, των λύσεών του.

Αξιόλογες επισημάνσεις: Α ν οι εξισώσεις του συστήματος παριστάνουν ευ­θείες τότε αν :

./ Οι ευθείες τέμνονται, το σύστημα έχει μια μόνο λύση, τις συντεταγμένες του κοινού σημείου των δύο ευθειών .

./ Οι ευθείες είναι παράλληλες, το σύστημα είναι αδύνατο και δεν έχει λύση .

./ Οι ευθείες ταυτίζονται, το σύστημα έχει άπειρες λύσεις τις συντεταγμένες των σημείων της μιας εκ των δύο ευθειών.

ΓΡΑΦΙΚΗ ΕΠΙΛΥΣΗ ΓΡΑΜΜΙΚΟΥ ΣΥΣΤΗΜΑΤΟΣ

Σύμφωνα με τα παραπάνω ένας τρόπος επίλυσης ενός γραμμικού συστήματος του οποίου οι εξισώ­σεις παριστάνουν ευθείες είvαι ο γραφικός δηλαδή η παράσταση σ' ένα καρτεσιανό σύστημα συντε­ταγμένων, των ευθειών που παριστάνουν οι εξι­σώσεις του και στη συνέχεια,

./ είτε ο προσδιορισμός του κοινού τους σημείου

./ είτε η διαπίστωση ότι οι ευθείες είναι παράλληλες δηλαδή ότι το σύστημα είναι αδύνατο

./ είτε ακόμη ότι οι ευθείες ταυτίζονται δηλαδή ότι το σύστημα έχει άπειρες λύσεις.

Παράδειγμα {2x - y = 1 Να λυθεί (γραφικά) το σύστημα: 2 x + y =

ΕΥΚΛΕΙΔΗΣ Β' λζ' τ.3/40

Page 43: Ευκλειδης Β 51

Μαθηματικά για την Α' Λυκείου

Λύση

Σύμφωνα με τις γραφικές παραστάσεις των γραμ­μικών εξισώσεων του συστήματος (σχήμα), λύση του συστήματος είναι οι συντεταγμένες του κοινού τους σημείου δηλαδή : (χ, ψ) = ( 1 , 1 ) .

Υ

χ

ΑΛΓΕΒΡΙΚΕΣ ΜΕΘΟΔΟΙ ΕΠΙΛ ΥΣΗΣ ΓΡΑΜΜΙΚΟΥ ΣΥΣΤΗΜΑΤΟΣ

Επειδή η γραφική επίλυση ενός γραμμικού συστήμα­τος έχει το μειονέκτημα της έλλειψης ακρίβειας στον προσδιορισμό των λύσεων υπενθυμίζουμε τις αλγε­βρικές μεθόδους επίλυσης γραμμικών συστημάτων: • Μέθοδος αντικατάστασης • Μέθοδος αντιθέτων συντελεστών και • Μέθοδος οριζουσών

ΜΕΡΟΣ Α' : • ΜΕΘΟΔΟΣ ΑΝτΙΚΑΤΑΣΤΑΣΗΣ

• ΜΕΘΟΔΟΣ ΑΝτΙΘΕΤΩΝ ΣΥΝΤΕΛΕΣΤΩΝ

Στις αλγεβρικές μεθόδους επίλυσης ενός γραμμι­κού συστήματος, το μετατρέπουμε διαρκώς σε άλ­λο ισοδύναμό του, δηλαδή σε σύστημα με τις ίδιες ακριβώς λύσεις με το αρχικό. Αυτό συνήθως επι­τυγχάνεται με δύο τρόπους: D Λύνουμε τη μια εξίσωση ως προς τον έναν

άγνωστο και τον aντικαθιστούμε στην άλλη

ΠΑΡΆΔΕΙΓΜΑ 1 {2x - y = l Να λυθεί το σύστημα:

x + y = 2

Λύση

(Μέθοδος αντικατάστασης) {2x - y = 1 <=> {2x - y = 1 <=> {2χ - (2 - χ) = 1 <::::> x + y = 2 y = 2 - x y = 2 - x {3χ = 3 {χ = 1 {χ = 1

y = 2 - x <::::> y = 2 - 1 <::::> y = 1 Άρα: Το σύστημα έχει μια μοναδική λύση την (χ, y) = ( 1 , 1 ) .

ΠΑΡΆΔΕΙΓΜΑ 2 {2x - y = l Να λυθεί το σύστημα:

x + y = 2

Λύση

(Μέθοδος αντίθετων συντελεστών) {2χ - y = 1 <::::> {( 2χ - y) + (χ + y) = 1 + 2

<::::> x + y = 2 x + y = 2 {3χ = 3 {χ = 1 {χ = 1 <::::> x + y = 2 <::::> 1 + y = 2

<::::> y = 1

Άρα: Το σύστημα έχει μια μοναδική λύση την (χ, y) = ( 1 , 1 ) .

Παρατήρηση : Οι αριθμοί λ1 και λ2 επιλέγονται έτσι ώστε να

προκύψουν αντίθετοι συντελεστές σ' έναν από τους

αγνώστους. Στο προηγούμενο παράδειγμα ήταν

λ1 = 1 και λ2 = 1 .

ΠΑΡΆΔΕΙΓΜΑ 3 {3x + 2y = 36 D Αντικαθιστούμε μια από τις εξισώσεις του Να λυθεί το σύστημα: ·

. 2x + 4y = 40 συστήματος π.χ. την ε1 με το γραμμικό της συν­

δυασμό με την άλλη ε2, δηλαδή με την εξίσωση: λ1 • ε1 + λ2 • ε2 , λ1 =ι:. Ο .

(Η εξίσωση : λ1 • ε1 + λ2 • ε2 είναι εκείνη που προ-κύπτει προσθέτοντας κατά μέλη τις εξισώσεις ει και ε2, αφού προηγουμένως πολλαπλασιάσουμε και τα δύο μέλη της ει με τον αριθμό λι και της ε2 με τον αριθμό λ2)

{3x + 2y = 36 <::::> 2x + 4y = 40

Λύση

{2 · {3χ + 2y) + ( -3) · {2χ + 4y) = 2 · 36 + ( -3) · 40 <::::> 2x + 4y = 40 {-8y = -48 {y = 6 {y = 6 2x + 4y = 40 <::::> 2χ + 4 · 6 = 40 <::::> χ = 8

ΕΥΚΛΕΙΔΗΣ Β' λζ' τ.3/41

Page 44: Ευκλειδης Β 51

Μαθηματικά -για την Α' Λυκείου

Άρα: Το σύστημα έχει μια μοναδική λύση την (χ, y) = (8, 6) .

ΑΔ ΥΝΑ ΤΟ ΣΥΣΤΗΜΑ Όπως έχουμε αναλύσει πριν, ένα σύστημα του ο­ποίου οι εξισώσεις παριστάνουν ευθείες είναι αδύ­νατο όταν οι ευθείες είναι παράλληλες. Τι γίνεται όμως όταν το λύνουμε αλγεβρικά;

ΠΑΡΆΔΕΙΓΜΑ 4 {6x - 2y = 11 Να λυθεί το σύστημα: 9x - 3y = 15

Λύση 1η μέθοδος: (Αντικατάστασης)

<=> . . . <=> <=> {6x - 2y = 1 1 {y = 6χ; 1 1

9x - 3y = 1 5 9χ _ 3 6χ; 1 1 = 1 5 {6χ - 2y = 1 1 {6χ - 2y = 1 1 <=> 1 8χ - 1 8χ + 33 = 30 <=> Οχ = -3 (αδύνατη)

Άρα: Το σύστημα είναι ΑΔ ΥΝΑ ΤΟ. 2η μέθοδος: (Αντιθέτων συντελεστών) {6χ - 2y = 1 1 9x - 3y = 1 5

<=> {3 ( 6χ - 2y ) + (-2) (9χ - 3y ) = 3 · 1 1 + ( -2) · 1 5 9χ - 3y = 1 5

<=> {Οχ + O y = 3 (αδύνατη) 9χ - 3y = 1 5

Άρα: Το σύστημα είναι ΑΔΥΝΑΤΟ.

Παρατήρηση : Επιλύνοντας γραφικά το σύστημα παρατηρούμε ό­τι η πρώτη εξίσωση παριστάνει την ευθεία:

Υ

χ

1 1 ει : ψ = 3χ -- και η δεύτερη την ευθεία : 2 ε2 : ψ = 3χ - 5 που είναι παράλληλες οπότε το σύστημα είναι Αδύνατο.

ΑΟΡΙΣΤΟ ΣΥΣΤΗΜΑ (Η ΣΥΣΤΗΜΑ ΜΕ ΑΠΕΙΡΕΣ Λ ΥΣΕΙΣ)

Όπως έχουμε αναλύσει πριν, ένα σύστημα του ο­ποίου οι εξισώσεις παριστάνουν ευθείες έχει άπει­ρες λύσεις όταν οι ευθείες ταυτίζονται. Τι γίνεται όμως όταν το λύνουμε αλγεβρικά;

ΠΑΡΆΔΕΙΓΜΑ 5 , , {x - 3y = 9 Να λυθει το συστημα: -2x + 6y = -18 Λύση 1η μέθοδος: (Αντικατάστασης) {χ - 3y = 9 {χ = 3y + 9

-2x + 6y = -1 8 <=> -2x + 6y = -1 8

<=> {x = 3y + 9 <=> <=> -2 (3y + 9)+ 6y = - 1 8 {x = 3y + 9 <=> Oy = Ο (άπειρες λύσεις)

Άρα: Το σύστημα έχει άπειρες λύσεις της μορ­φής: (χ, y) = (3y + 9, y) ,όπου y Ε JR . 2η μέθοδος: (Αντιθέτων συντελεστών) {x - 3y = 9 -2x + 6y = -1 8

<=> {2 (χ - 3y ) + ( -2χ + 6y ) = 2 . 9 + ( - 1 8) x - 3y = 9

<=> {Οχ + Oy( άπειρες λύσεις) x - 3y = 9

(χ - 3y = 9 <=> χ = 3y + 9) Άρα: Το σύστημα έχει άπειρες λύσεις της μορ­φής: (χ, y) = (3y + 9, y) ,όπου y Ε JR .

Παρατήρηση : Επιλύνοντας γραφικά το σύστημα παρατηρούμε ότι η πρώτη εξίσωση παριστάνει την ευθεία:

1 ει : ψ = -χ - 3 και η δεύτερη την ευθεία: 3 1 ε2 : ψ = -χ - 3 που ταυτίζονται οπότε το σύστημα 3

ΕΥΚΛΕΙΔΗΣ Β' λζ' τ.3/42

Page 45: Ευκλειδης Β 51

Μαθηματικά για την Α' Λυκείου

f:χει άπειρες λύσεις της μορφής: (χ, y) = (3y + 9, y) , 1 όπου y Ε JR ή της μορφής: (χ, y) = (χ,-χ - 3), 3

χ E lR

ΑΣΚΗΣΕΙΣ Α. Σωστό - Λάθος

1. Το σημείο (2, 3) ανήκει στην ευθεία χ = 3 . Σ Λ

2. Η εξίσωση παριστάνει ευθεία για κάθε λ Ε JR

Σ Λ

3 . Υπάρχουν τιμές των α, β για τις οποίες το σύστη­

μα: {αχ - ψ = 0 δf:χεται πάντα άπειρες λύσεις. βχ + ψ = Ο

Β. Πολλαπλής επιλογής

Σ Λ

1 . Η γραμμική εξίσωση που επαληθεύεται με κάθε ζεύγος της μορφής: (χ, ψ) = (2κ + 3, κ - 1) (κ Ε JR) είναι: Α. χ - 2ψ = 1 , Δ. χ - y = 3,

Β. χ - ψ = 2, Ε. 2ψ - χ = 1

Γ. Ανάπτυξης (Συμπληρωματικές)

Γ. χ - 2ψ = 5

1 . Για ποιες τιμές των α, β οι ευθείες με εξισώσεις αντίστοιχα: ε1 : (α + β)χ + (α - β)ψ = 4β - 6α, ε2 : 2 (α - β)χ - (α + β)ψ = 2 - 2β τέμνονται στο σημείο Μ(2, -3) .

2. Να λυθούν τα συστήματα: { l x + 2ψ - 6l = 5 {0, 5χ - 1, 5ψ = 1 α) ' β) 2χ + 3ψ = 1 8 0, 4χ - 0, 2ψ = 0, 4

3 . Σε μια έκθεση αυτοκινήτων υπάρχουν συνολι­κά 1 7 αυτοκίνητα τρίθυρα και πεντάθυρα. Α ν όλα μαζί έχουν 67 πόρτες πόσα αυτοκίνητα με πέντε πόρτες και πόσα με τρεις πόρτες υπάρ­χουν στην έκθεση;

4. Σε ένα διψήφιο αριθμό το ψηφίο των δεκάδων είναι αριθμός μεγαλύτερος κατά 2 του ψηφίου των μονάδων. Αν διαιρέσουμε τον διψήφιο α­ριθμό αυτό με το άθροισμα των ψηφίων δεκά­δων και μονάδων βρίσκουμε πηλίκο 6 και υπό­λοιπο 3 . Να βρεθεί ο διψήφιος αριθμός.

(Υπόδειξη : Θυμίζουμε ότι αν χ και ψ τα ψηφία των δεκάδων και μονάδων αντίστοιχα ο αριθμό.;

ισούται με: l Ox + ψ)

ΜΕΡΟΣ Β ' : ΜΕΘΟΔΟΣ ΟΡΙΖΟΥΣΩΝ

, , {αχ + βy = γ Εστω το συστημα: α'χ + β'y = γ'

Αν D = ι:, :. ι = αβ' - βα',

Dx = l γ γ' :.ι = γβ' - βγ' και

Dψ = ι:. �I = αγ' - γα' τότε:

• Αν ID * οΙ τότε το σύστημα έχει μοναδική

λύση την: (x,y) = (� , �) (Παράδ. 1 )

• Αν ID = οΙ τότε το σύστημα είναι Αδύνατο ή έχει Άπειρες λύσεις (Αόριστο)

Επισημαίνουμε ότι: ./ Α ν D = Ο και D χ * Ο ή D Ψ * Ο τότε το

σύστημα είναι Αδύνατο (Παράδ.2) ./ Α ν D = D χ = D Ψ = Ο υπάρχει ένας τουλάχιστον

συντελεστής αγνώστου, διάφορος του μηδενός το σύστημα έχει άπειρες λύσεις. (Παράδ.3)

./ Αν D = Dx = DΨ = Ο και όλοι οι συντελεστές των αγνώστων είναι μηδέν τότε αν γ = γ' = Ο τότε το σύστημα έχει άπειρες λύσεις οποιοδήποτε ζεύγος αριθμών (χ, y) (Παραδ.5) , ενώ αν γ * Ο ή γ' * Ο το σύστημα είναι αδύνατο (Παραδ.4)

Προσέξτε όμως! Στην περίπτωση που ισχύει D = Ο μπορούμε να λύνουμε το σύστημα με έναν από τους αλ γεβρι­κούς τρόπους που προαναφέραμε και να διαπιστ� νουμε έτσι αν είναι αδύνατο ή έχει άπειρες λύσεις.

ΠΑΡ ΑΔΕΙΓΜΑ 1

, , {2x - y = 1 Να λυθει το συστημα: x + y = 2

ΕΥΚΛΕΙΔΗΣ Β' λζ' τ.3/43

Page 46: Ευκλειδης Β 51

Μαθηματικά για την Α' Λυκείου

D = l� D = 1 1 χ 2

D = 12 Υ 1

Λύση -1 1 = 2 · 1 - 1 · ( - 1) = 3 :;t: ο 1 ' - 1 1 = 1 · 1 - 2 · (- 1) = 3 1 '

�I = 2 . 2 - 1 · 1 = 3

Άρα: Το σύστημα έχει άπειρες λύσεις, της μορ­φής (χ, Υ) = (χ,± · χ - 3) όπου χ Ε � .

ΠΑΡΑΔΕΙΓΜΑ 4

{λχ + y = λ2 ' Να λυθεί το σύστημα: , , για καθε χ + λy = 1

πραγματική τιμή του λ. Άρα: Το σύστημα έχει μοναδική λύση την: Λύση

(χ, ψ) = (% ,%) = (1, 1) ΠΑΡΑΔΕΙΓΜΑ 2

{6x - 2y = 11 Να λυθεί το σύστημα: 9x - 3y = 15

Λύση

D = l6 -2 1 = 6 · (-3) - 9 � (-2) = 0 9 -3 '

D = 1 1 1 1 5 1 = 1 1 · (-3) - 1 5 · (-2) = 3 :;t: O χ -2 -3

Άρα: Το σύστημα είναι Αδύνατο. Άλλωστε οι ευθείες που παριστάνουν οι εξισώσεις

' 3 1 1 3 5 του συστηματος ει : ψ = · χ --, ε2 : ψ = · χ -2

είναι παράλληλες. π ' ' ' 6 -2 1 1 αρατηρειστε επισης οτι: "9 =

_ 3 :;t: l5 .

ΠΑΡΑΔΕΙΓΜΑ 3 {x - 3y = 9 Να λυθεί το σύστημα: -2x + 6y = -18

Λύση

D = 1 1 -3 1 = 1 · 6 - (-2) · (-3) = Ο -2 6 '

D = I 9 -3 1 = 9 . 6 - ( -1 8) . (-3) = ο χ - 1 8 6 '

D = 1 1 -2 1 = 1 · (-1 8) - 9 · (-2) = 0 Υ 9 - 1 8

Επειδή υπάρχει τουλάχιστον ένας συντελεστής μη μηδενικός το σύστημα έχει άπειρες λύσεις. Άλλωστε οι ευθείες που παριστάνουν οι εξισώσεις

1 1 του συστήματος ει : ψ = - · χ - 3, ε2 : ψ = - · χ - 3 3 3 ταυτίζονται. π ' ' ' 1 -3 9 αρατηρειστε επισης οτι:

_ 2 = 6 =

_1 8

D = I � �I = λ2 - 1 = (λ - 1Ηλ + 1) ,

1 = λ2 · λ - 1 · 1 = Κ - 1 = (λ- 1) (Κ + λ+ 1) λ '

λ λ2 D = = λ · 1 - 1 · λ2 = -λ(λ - 1) Υ 1 1

D = Ο � (λ - 1 )(λ + 1 ) = Ο � λ = 1 ή λ = -1 Διακρίνουμε τις παρακάτω περιπτώσεις : • Αν λ :;t: 1 και λ :;t: -1 τότε το σύστημα έχει

δ ' λ ' ( ) ( λ2 + λ + 1 -λ ) μονα ικη υση την: χ, y = ,--λ+ 1 λ + 1 {χ + y = 1 • Αν λ = 1 το σύστημα γίνεται: x + y = 1

Οπότε έχει άπειρες λύσεις της μορφής: (χ, y) = (χ, 1 - χ), Χ Ε � . Α ν λ = -1 το σύστημα είναι αδύνατο αφού

Dx = -2 :;t: O (Επαληθεύτε γραφικά τα συμπεράσματά σας)

ΟΜΟΓΕΝΉ ΓΡΑΜΜΙΚΆ ΣΥΣΤΉΜΑΤΑ {αχ + βψ = Ο Έστω το σύστημα: , , α χ + β ψ = Ο Το σύστημα αυτό λέγεται ομογενές (γ = γ ' = Ο) Προφανώς έχει πάντα λύση τη μηδενική (χ, y) = (0, 0) . • Α ν D :;t: Ο τότε έχει μία μόνο λύση την

μηδενική και • Α ν D = Ο τότε έχει άπειρες λύσεις

συμπεριλαμβανομένης και της μηδενικής. (Στα ομογενή συστήματα ισχύει: Dx =Dψ = 0)

Παράδειγμα:

Να λυθεί το ομογενές σύστημα με δύο εξισώσεις και δύο αγνώστους και με ορίζουσες που ικανοποιούν τις συνθήκες: Dx + Dψ + D = 1 .

Λύση Επειδή Dx = Dψ = Ο θα ισχύει: D = 1 :;t: Ο άρα το σύ­στημα έχει μόνο μία λύση την μηδενική: (χ, y) = (0, 0).

ΕΥΚΛΕΙΔΗΣ Β' λζ' τ.3/44

Page 47: Ευκλειδης Β 51

Μαθηματικά για την Α' Λυκείου

ΑΣΚΗΣΕΙΣ Α. Σωστό - Λάθος

(1) Α ν ID - 21 + I2D - 41 = Ο τότε το σύστημα έχει μοναδική λύση Σ Λ

(2) Αν IDI + jDΨ + 2 j = O τότε το σύστημα έχει ά-πειρες λύσεις Σ Λ

(3) Αν D2 + (1 - Dx )2 = 0, το σύστημα είναι αδύ-νατο Σ Λ

(4)Αν D2 + D� + (2 + DΨ )2 = Ο το (Σ) είναι αδύ-νmο Σ Λ

(5) Αν IDx l + jDΨ j + IDI = Ο τότε το (Σ) έχει πάντα άπειρες λύσεις Σ Λ

2 2. Αν D :;t: O και Dx = )D και DΨ = 2Dx τότε η λύση του αντιστοίχου συστήματος είναι:

Α(2 2) Β(� �) Γ(� �) Δ(� �) Ε(-� -�) ' ' 3 ' 2 ' 3 ' 3 ' 3 ' 3 ' 2 ' 3 {4χ + 2κψ = 1 3 . Αν το σύστημα: , , κ ε IR. είναι 2χ + 3ψ = 4 {χ + ψ = -1 αδύνατο, τότε το σύστημα: είναι: κχ + 3y = 3 Α. Αδύνατο, Β. Έχει άπειρες λύσεις Γ. Έχει μοναδική λύση την (x,y) = (-2, 1 ), Δ. Δεν μπορούμε να απαντήσουμε.

4. Για ποια τιμή του λ το σύστημα: {2χ + λψ = 7 έχει άπειρες λύσεις. 6χ - 1 5ψ = 2 1

5. Για ποιες τιμές του λ το σύστημα: {χ + ψ = 7 είναι αδύνατο. 2χ + 2ψ = λ - 1

6. Υπάρχουν τιμές του λ για τις οποίες το σύστη-{χ + λψ = 4 μα: έχει μοναδική λύση; -2χ + 5ψ = 1 7. Να λυθεί το σύστημα: {λχ + (λ + 1)ψ = λ , λ ε !R. . (λ + 1) χ + (λ + 3)ψ = 2 8. Για ποιες τιμές του λ το (ομογενές) σύστημα: {λχ - 3λψ = 0

( ) έχει και άλλες λύσεις εκτός λ - 2 χ - λψ = 0

της μηδενικής. 9. Σ' ένα σύστημα δυο γραμμικών εξισώσεων με

αγνώστους χ, ψ ισχύει: D2 + D� + D� = 6D - 2DΨ - 1 0 .

Να λυθεί το σύστημα.

Μ , ,

ια πεp1ηγηαη ιπο χωpο των

παpαλληλογpάμμων - τpαnεζiων του Βασίλη Καρκάνη

Το άρθρο αυτό μέσα από ερωτήσεις διαφόρων τύπων (Σωστό - Λάθος, ΠοΑλαπλής επιλογής και Πλήρους Ανάπτυξης) προσπαθεί να φέρει το μαθητή, σε επαφή με τις σημαντιΚές έννοιες του κεφαλαίου αυτού. Σημειώνουμε για πληρέστερη ενημέρωση τα αντίστοιχα άρθρα στα τεύχη 22, 30 και 43 του Ευκλείδη Β '. Για όσους δεν διαθέτουν τα τεύχη αυτά, υπάρχει το CD για όλα τα τεύχη του Ευκλείδη που μπορεί να προ­μηθευτούν από τα γραφεία της Ε.Μ.Ε. Α. Ερωτήσεις Σωστού - Λάθους Τις παρακάτω ερωτήσεις να χαρακτηρίσετε ως σω­στές (Σ) ή λάθος (Λ).

1. Οι διαγώνιες ενός παραλληλόγραμμου διχοτο­μούν τις γωνίες του.

2. Αν τα ευθύγραμμα τμήματα ΑΓ, ΒΔ τέμνονται και έχουν κοινό μέσο, το τετράπλευρο ΑΒΓ Δ είναι παραλληλόγραμμο.

3. Αν στο παραλληλόγραμμο ΑΒΓΔ είναι ΑΓ<ΒΔ Λ Λ

τότε θα είναι και Α < Β . 4. Αν στο παραλληλόγραμμο ΑΒΓΔ είναι

Λ Λ Λ Β+ Γ+ Δ = 270° τότε αυτό είναι ορθογώνιο.

5. Αν ένας ρόμβος έχει δύο απέναντι γωνίες πα­ραπληρωματικές τότε είναι τετράγωνο.

ΕΥΚΛΕΙΔΗΣ Β' λζ' τ.3/45

Page 48: Ευκλειδης Β 51

Μαθηματικά για την Α' Λυκείου

6. Στο τετράγωνο και στον ρόμβο όλες οι γωνίες είναι ίσες.

7. Η περίμετρος ενός τριγώνου, είναι διπλάσια α­πό την περίμετρο του τριγώνου, που ορίζουν τα μέσα των πλευρών του.

8. Το ίχνος της διαμέσου ενός ορθογωνίου τριγ� νου που αντιστοιχεί στην υποτείνουσα, ισαπέ­χει από τις τρεις κορυφές του τριγώνου.

Λ 9. Σε ορθογώνιο τρίγωνο ΑΒΓ με Α = 90° και

Λ Γ = 30° η διάμεσος που αντιστοιχεί στην υπο-τείνουσα ορίζει δύο ισόπλευρα τρίγωνα.

lΟ.Υπάρχει τρίγωνο στο οποίο το βαρύκεντρο και το ορθόκεντρο να είναι το ίδιο σημείο.

1 1 . Υπάρχει τρίγωνο στο οποίο το ορθόκεντρο να συμπίπτει με μία από τις κορυφές του.

12 .Αν στη διάμεσο ΑΜ τριγώνου ΑΒΓ πάρουμε τα σημεία Λ, Κ ώστε ΑΛ = ΛΚ = ΚΜ τότε το σημείο Κ είναι το βαρύκεντρο του τριγώνου.

13 .Αν δύο διαδοχικές γωνίες ενός τετραπλεύρου εί­ναι παραπληρωματικές τότε αυτό είναι τραπέζιο.

14 .Το ευθύγραμμο τμήμα που ενώνει τα μέσα των βάσεων τραπεζίου λέγεται διάμεσος του τραπε­ζίου.

ΙS .Αν ένα τραπέζιο έχει τρεις πλευρές ίσες τότε είναι ισοσκελές.

Β. Ερωτήσεις πολλαπλής επιλογής. Στις παρακάτω ερωτήσεις να επιλέξετε τη σωστή α­πάντηση από τις προτεινόμενες.

1 6.Αν στο παραλληλόγραμμο ΑΒΓΔ είναι ΑΒ = 3λ+ 1 7, ΓΔ = 2λ+20 τότε η τιμή του πραγματι­κού αριθμού λ είναι: i) λ = 2 iii) λ = 4

ii) λ = 3 iν) λ = 5 .

1 7.Αν στο παραλληλόγραμμο ΑΒΓ Δ Λ Λ Λ

είναι Α = 1 05° , Β = χ, Γ = χ + ψ τότε οι τιμές χ, ψ είναι: i) χ = 1 00°

ψ = 80° iii) χ = 75°

ψ = 30°

ii) χ = 35° ψ = 75°

iν) χ = 65° ψ = 75°

Λ 18.Στο παραλληλόγραμμο ΑΒΓΔ είναι Δ = 70°

και Ε σημείο της ΑΒ ώστε ΒΓ = ΓΕ. Η γωνία Λ

ΒΓΕ ισούται με: i) 30° iii) 50°

19.Στο ορθογώνιο ΑΒΓΔ είναι ΒΔ = 1 2. Αν Ο εί­ναι το σημείο τομής των διαγωνίων τότε η ΟΓ ισούται με:

i) 6 iii) 24

ii) 7 iν) 12

Λ 20.Αν στον ρόμβο ΑΒΓΔ είναι ΓΑΒ = 25° τότε η

Λ γωνία ΑΒΓ ισούται με: i) 1 55° iii) 1 20°

ii) 140° iν) 1 30°

2 1 .Α ν στο τετράγωνο ΑΒΓ Δ είναι ΑΒ = 2λ-6, ΒΓ = 3λ - 1 2 τότε η περίμετρος του ισούται με: i) 20 ii) 3λ +6 iii) 22 iν) 26

22. Τα μέσα των πλευρών πλευρου σχηματίζουν:

οποιουδήποτε τετρά-

i) ορθογώνιο ii) παραλληλόγραμμο iii) ρόμβο

23.Στο διπλανό τρίγωνο Λ Λ Λ

iν) τετράγωνο ΑΒΓ είναι

Α = 90° , Β = Γ και Δ, Ε, Ζ μέσα των πλευρών ΒΓ, ΑΒ, ΑΓ αντίστοιχα. Το πλήθος των ορθο­γώνιων και ισοσκελών τριγώνων είναι:

Α

Ε ι z ' ι ' ' ι ' ' ' ' ι ' ' ι , ' ,

Β Δ Γ i) 5 ii) 7 iii) 8 iν) 6

Λ 24.Στο ορθογώνιο τρίγωνο ΑΒΓ είναι Α = 90° ,

Λ Ο Λ Β = 60 , ΑΔ = υα , ΑΜ = μα . Η γωνία ΜΑΔ ισούται με: i) 33° iii) 42°

ii) 66° iν) 44°

25.Αν το ορθόκεντρο είναι σημείο που βρίσκεται στο εξωτερικό ενός τριγώνου, τότε το τρίγωνο είναι: i) Αμβλυγώνιο iii) Οξυγώνιο

ii) Ορθογώνιο iν) Ισοσκελές

26.Α ν ΑΜ = � και Κ το βαρύκεντρο του τριγώνου ΑΒΓ ώστε ΚΜ = 2 τότε το ΚΑ ισούται με: i) 4 ii) 3 iii) 1 iν) � 3

27.Αν ΑΜ = � και Κ το βαρύκεντρο τριγώνου ΑΒΓ τότε ο λόγος ΚΜ ισούται με: ΚΑ ") 2 Ι -

3 ii) 2

ΕΥΚΛΕΙΔΗΣ Β: λζ' τ.2/46

Page 49: Ευκλειδης Β 51

Μαθηματικά για την Α' Λυκείου

" " ") 1 111 -3 iν) 2

28.Αν ο λόγος της μικρής προς τη μεγάλη βάση ενός τραπεζίου ισούται με .!_ τότε ο λόγος της 3 διαμέσου προς τη μικρή βάση ισούται με: Q ! m 1 2 iii) 2 iν) 3

29.Αν στο ισοσκελές τραπέζιο ΑΒΓΔ με ΑΒ//ΓΔ, Λ Λ

είναι Α = χ και Β = 225° - 4χ τότε η γωνία Λ Γ ισούται με: i) 1 20° iii) 45°

ii) 75° iν) 1 35°

30.Στο παρακάτω ισοσκελές τραπέζιο το ύψος του ισούται με: ") 7 ι -

2 iii) 2λ

Παρ ατή ρηση :

ii) λ iν) 3λ

λ

\ 3λ

Οι απαντήσεις των προηγούμενων ερωτήσεων είναι: ( 1 , Λ), (2, Σ), (3 , Λ), (4, Σ), (5 , Σ), (6, Λ), (7, Σ), (8, Σ), (9, Λ), ( 1 0, Σ), ( 1 1 , Σ), ( 1 2, Σ), ( 1 3 , Σ), ( 14, Λ), ( 1 5 , Σ) ( 1 6, ii), ( 1 7, iii) , ( 1 8, iν), ( 1 9, i), (20, iν), (2 1 , ίί), (22, ii), (23 , ii), (24, iii), (25 , i), (26, i), (27, iν), (28 , iii), (29, iν), (30, ii) .

Γ. Λυμένες Ασκή σεις.

Άσκηση l η Λ

Δίνεται το τρίγωνο ΑΒΓ με Α < 90° και Δ, Ε, Ζ

τα μέσα των πλευρών ΒΓ, Γ Α, ΑΒ αντίστοιχα.

Στο εξωτερικό του τριγώνου φέρνουμε τα ΑΒ

τμήματα ΖΗ .l ΑΒ με ΖΗ = -, ΕΘ .l ΑΓ με 2 ΕΘ =

ΑΓ . Να δείξετε ότι: 2

Λ Λ α) ΖΔΕ = Α

β) ΔΕ = ΖΗ και ΔΖ = ΕΘ Λ Λ

γ) ΔΖΗ = ΔΕΘ

δ) το τρίγωνο ΔΗΘ είναι ορθογώνιο και ισο­

σκελές.

Έχουμε: ZH _l_ AB

ΖΗ = ΑΒ 2 (1) ΕΘ .l ΑΓ

ΕΘ = ΑΓ 2

Λύση

Η

α) Εφόσον τα Δ, Ζ είναι μέσα των δύο πλευρών του τριγώνου ΑΒΓ θα είναι ΔΖ 1 1 ΑΓ και ΔΖ = ΑΓ = ΑΕ . 2 Δηλαδή ΔΖ 1 1 ΑΕ και ΔΖ = ΑΕ οπότε το ΑΖΔΕ

Λ Λ είναι παραλληλόγραμμο άρα ΖΔΕ = Α .

ΑΒ (ι) β) Λόγω του (α) είναι: ΔΕ = ΑΖ = -=ΖΗ . 2

ΑΓ 0) Επίσης ΔΖ = ΑΕ = -=ΕΘ . 2

Λ Λ γ) Λόγω του (α) είναι Α = Ζι (εντός εκτός επί τα

Λ Λ αυτά) και Α = Ει (εντός εκτός επί τα αυτά) Έτσι: ΔΖΗ= 90° + Ζι = 90° +Α) Λ Λ

οπότε ΔΖΗ= ΔΕΘ . Λ Ο Λ Ο Λ

ΔΕΘ= 90 + Ει = 90 +Α Δ Δ

δ) Λόγω των (β), (γ) είναι ΔΖΗ = ΕΔΘ οπότε Λ Λ

ΔΗ = ΔΘ (2) και ΔΖΗ = ΔΕΘ (3) . Στο τρίγωνο ΕΔΘ είναι:

Λ Λ Λ ΕΔΘ+ ΕΘΔ+ ΔΕΘ = 1 80° ή

Λ Λ Λ ΕΔΘ+ ΕΘΔ+ 90° + Ει = 1 80° ή

Λ Λ Λ ΕΔΘ+ ΕΘΔ+ Α = 90° . Η τελευταία ισότητα λόγω του (α) και της (3) γράφεται:

Λ Λ Λ Λ ΕΔΘ+ ΖΔΕ+ ΖΔΗ = 90° ή ΘΔΗ = 90° (4) . Λόγω των (2),(4) το τρίγωνο ΔΗΘ είναι ορθογώνιο και ισοσκελές.

Άσκηση 2η

Δίνεται το ορθογώνιο τρίγωνο ΑΒΓ με Α = 90°

ΕΥΚΛΕΙΔΗΣ Β ' λζ' τ.2/47

Page 50: Ευκλειδης Β 51

Μαθηματικά για την Α' Λυκείου

και f = 30° . 'Εστω Δ μέσο της ΒΓ, Ε μέσο της

ΑΒ, Ζ μέσο της ΒΔ και Ι μέσο της ΑΔ. Α ν

προεκτείνουμε την ΕΖ κατά τμήμα ΖΗ = ΕΖ

τότε να δείξετε ότι:

α) το τετράπλευρο ΒΕΔΗ είναι ορθογώνιο πα­

ραλληλόγραμμο,

β) το τετράπλευρο ΑΕΖΔ είναι ισοσκελές τρα­

πέζιο,

γ) τα τμήματα ΑΗ, ΕΔ διχοτομούνται στο ση­μείο Θ,

δ) το τετράπλευρο ΕΖΔΙ είναι ρόμβος με κέντρο

το Θ.

Λύση α) Το ΒΕΔΗ είναι παραλληλόγραμμο γιατί οι

διαγώνιες του διχοτομούνται. Ακόμα τα Ε, Δ είναι μέσα των δύο πλευρών του τρίγωνου ΑΒΓ οπότε ΔΕ 1 1 ΑΓ άρα ΔΕ ..l ΑΒ .

Α

Η Λ

Δηλαδή ΔΕΒ = 90° οπότε το ΒΕΔΗ είναι ορθογώνιο.

β) Στο τρίγωνο ΒΑΔ τα Ε, Ζ είναι μέσα των δύο πλευρών του οπότε ΖΕ 1 1 ΑΔ άρα το ΑΕΖΔ είναι τραπέζιο. Το τρίγωνο ΑΒΔ είναι · ισόπλευρο (γιατί;)

οπότε θα είναι: ΑΕ = ΑΒ = ΒΔ = ΖΔ δηλαδή 2 2 ΑΕ = ΖΔ άρα το τραπέζιο ΑΕΖΔ είναι ισοσκελές. (Αλλιώς;)

γ) Το ΑΕΗΔ είναι παραλληλόγραμμο (γιατί;)

οπότε οι διαγώνιες του ΑΗ, ΕΔ θα διχοτομούνται σε σημείο Θ.

δ) Το ΕΖΔΙ είναι παραλληλόγραμμο (γιατί;) με κέντρο το μέσο Θ της διαγωνίου του ΕΔ. Επίσης ΕΙ = ΒΔ = ΑΔ = ΕΖ (γιατί;) δηλαδή 2 2 ΕΙ = ΕΖ οπότε το ΕΖΔΙ είναι ρόμβος

Άσκηση 3η

ΑΒ>ΒΓ, ΑΓ = 2ΒΓ. Αν η κάθετος στην ΒΔ στο

Ο τέμνει την ΑΔ στο Ε και την ΑΒ στο Ζ να

δείξετε ότι:

α) το τρίγωνο ΒΔΕ είναι ισόπλευρο,

β) το ΑΕΒΓ είναι παραλληλόγραμμο

γ) το ΑΕΒΟ είναι ισοσκελές τραπέζιο,

δ) ΔΖ ..l ΒΕ .

Λύση

Έχουμε ΑΒ>ΒΓ και ΑΓ = 2ΒΓ ( 1 ) Λ Λ

α) Λόγω της ( 1 ) θα είναι Αι = 30° οπότε Γι = 60° Λ

άρα και Δι = 60° (2) (γιατί;)

ι..:_ _____ _.:..,ι Γ

Στο τρίγωνο ΒΔΕ η ΕΟ είναι ύψος και διάμεσος οπότε αυτό είναι ισοσκελές και λόγω της (2) θα είναι τελικά ισόπλευρο.

(\) β) Επίσης ΕΔ = ΒΔ = ΑΓ = 2ΒΓ δηλαδή

ΕΔ = 2ΒΓ ή ΑΕ + ΑΔ = 2ΒΓ ή ΑΕ = ΒΓ. Όμως ΑΕ 1 1 ΒΓ οπότε το ΑΕΒΓ είναι παραλληλόγραμμο.

γ) Λόγω των (α), (β) είναι: ΑΟ 1 1 ΒΕ και ΑΕΒ = ΕΒΟ = 60° οπότε το ΑΕΒΟ είναι ισοσκελές τραπέζιο.

δ) Λόγω του (α) στο ισόπλευρο τρίγωνο ΒΔΕ τα ΕΟ, ΒΑ ύψη οπότε το Ζ είναι το ορθόκεντρο του τριγώνου. Άρα ΔΖ ..l ΒΕ ως φορέας του τρίτου ύψους.

Άσκηση 4η

Δίνεται το παραλληλόγραμμο ΑΒΓ Δ.

Προεκτείνουμε τις πλευρές του ΑΒ, ΔΓ κατά

ΒΕ = ΑΒ, ΓΖ = ΔΓ αντίστοιχα. Φέρνουμε τις

ΒΖ, ΓΕ που τέμνονται στο Ι. Α ν Ο είναι το

κέντρο του παραλληλόγραμμου ΑΒΓ Δ και οι

ΟΕ, ΟΖ τέμνουν τη ΒΓ στα σημεία Η, Θ

αντίστοιχα, να δείξτε ότι: Δίνεται το ορθογώνιο ΑΒΓ Δ με κέντρο Ο και

ΕΥΚΛΕΙΔΗΣ Β' λζ' τ.2/48

Page 51: Ευκλειδης Β 51

Μαθηματικά για την Α' Λυκείου

α) το ΟΒΙΓ είναι παραλληλόγραμμο,

β) ΙΕ = ΟΒ, ΙΖ = ΟΑ,

Γ) ΒΗ = ΗΘ = ΘΓ.

Λύση

α) Είναι ΒΕ 1 1 ΓΖ και ΒΕ = ΓΖ (γιατί; ) οπότε το ΒΕΖΓ είναι παραλληλόγραμμο, άρα οι διαγώνιες του θα διχοτομούνται, δηλαδή το Ι είναι μέσο της ΓΕ.

Α '

'

Δ

' , 0'.,.�"":. - -

... ... .... ' :: - - -Ε

- - - - ::. ';

z Έτσι στο τρίγωνο ΑΓΕ τα Ο, Β είναι μέσα των δύο πλευρών του οπότε: ΟΒ 1 1 ΓΕ και

ΟΒ = ΓΕ ή ΟΒ /I ΓΙ και ΟΒ = ΓΙ οπότε το 2

ΟΒΙΓ είναι παραλληλόγραμμο. β) Λόγω του (α) είναι: ΙΕ = ΙΓ = ΟΒ και ΙΖ = ΙΒ =

ΟΓ = ΟΑ. γ) Στο τρίγωνο ΑΓΕ οι ΓΒ, ΕΟ είναι διάμεσοι

οπότε το Η είναι το βαρύκεντρο άρα ΗΒ = .!_ΒΓ ( 1 ) . 3 Στο τρίγωνο ΒΔΖ οι ΒΓ, ΖΟ είναι διάμεσοι οπότε το Θ είναι το βαρύκεντρο άρα:

1 ΘΓ = '3ΒΓ (2).

Λόγω των ( 1 ),(2) θα είναι και ΗΘ = .!_ ΒΓ άρα 3 ΒΗ = ΗΘ = ΘΓ.

Άσκηση Sη Δίνεται τετράγωνο ΑΒΓ Δ με κέντρο το Ο και Ε

τυχαίο σημείο του ΔΟ. Από το Β φέρνουμε

κάθετη στην ΑΕ που τέμνει την ΑΟ στο Ζ. Να

δείξετε ότι:

α) ΒΖ = ΑΕ β) ΒΖ = ΕΓ γ) ΒΕ = ΓΖ.

δ) το ΒΖΕΓ είναι ισοσκελές τραπέζιο.

Λύση

α) Τα ορθογώνια τρίγωνα ΟΒΖ και ΟΑΕ έχουν: ΟΒ = ΟΑ (το ΑΒΓ Δ τετράγωνο) ) Λ Λ �α Β1 = Α1 (οξείες με πλευρές κάθετες)

Δ Δ ΟΒΖ = ΟΑΕ οπότε: ΒΖ = ΑΕ.

Δ

Λ , 'Ο' ,

' '

' ' '

Β

' '

Γ β) Στο τρίγωνο ΕΑΓ η ΕΟ διάμεσός και ύψος

οπότε αυτό είναι ισοσκελές με ΑΕ = ΕΓ και λόγω του (α) θα είναι τελικά ΒΖ = ΕΓ.

Δ Δ γ) Εφόσον ΟΒΖ = ΟΑΕ θα είναι ΟΖ = ΟΕ.

Ακόμη ΟΓ = ΟΒ (γιατί;) και με πρόσθεση κατά μέλη : ΟΖ+ ΟΓ = ΟΕ + ΟΒ ή ΓΖ = ΒΕ.

δ) Τα τρίγωνα ΟΒΓ και ΟΖΕ είναι ορθογώνια Λ Λ

ισοσκελή (γιατί;) οπότε Bz = 45° = Ε1 άρα σύμφωνα με το κριτήριο παραλληλίας θα είναι: ΒΓ Ι Ι ΖΕ δηλαδή το ΒΖΕΓ τραπέζιο και λόγω του (β) θα είναι ισοσκελές.

Άσκηση 6η

Από την κορυφή Α τριγώνου ΑΒΓ φέρνουμε

ευθεία ε που τέμνει την πλευρά ΒΓ σε

εσωτερικό σημείο διαφορετικό από το μέσον της Μ. Α ν ΒΔ, ΓΕ l. ε να δείξετε ότι ΜΔ = ΜΕ.

Λύση

Έχουμε ΒΔ, ΓΕ l. ε οπότε ΒΔ//ΓΕ άρα το ΒΔΓΕ είναι τραπέζιο.

ε

Το Μ είναι μέσο της διαγωνίου ΒΓ, οπότε <Ι\' � μέσο της διαγωνίου ΔΕ το ευθύγραμμο τμήμα � που ε­νώνει τα μέσα των διαγωνίων του τpαπε.:;ίου θα ro m παράλληλο στις βάσεις του. Έτσι � _ ε οπότε

ΕΥΚΛΕΙΔΗΣ Β' λζ' τ.2/49

Page 52: Ευκλειδης Β 51

Μαθηματικά για την Α' Λυκείου

mo τρί·fωνο .ΜΔΕ η ΜΝ είναι συγχρόνως ύψος και διάμεσο.; άρα αυτό είναι ισοσκελές με ΜΔ = ΜΕ.

λ σ κ η ση 7η .1ί,·εται το ισοσκελές τραπέζιο ΑΒΓΔ με

Γ = Δ = 45° . Έστω ΕΖ η διάμεσος του και ΑΗ

το ύψος του. Από το Ζ φέρνουμε παράλληλη

στην ΑΔ που τέμνει την Γ Δ στο Θ. Να δείξετε

ότι :

α) Το ΗΘΖΕ είναι ισοσκελές τραπέζιο.

β) ΘΖΓ = 90° .

γ) ΕΖ + ΑΗ = ΓΔ .

Λύση

α) Η ΕΖ είναι διάμεσος του τραπεζίου οπότε ΕΖ Ι I ΓΔ άρα ΕΖ Ι I ΗΘ δηλαδή το ΗΘΖΕ είναι τραπέζιο. Ακόμη στο ορθογώνιο τρίγωνο ΑΗΔ η ΗΕ είναι διάμεσος άρα ΗΕ = ΑΔ = ΕΔ =ΘΖ 2 (λόγω παραλληλογράμμου ΕΖΘΔ) οπότε το τραπέζιο είναι ισοσκελές.

Α Β

Δ Λ Λ Ο β) Η Θι = Δ = 45 (ως εντός εκτός επί τα αυτά) .

Λ Λ Ο Έτσι στο τρίγωνο ΘΖΓ η Θι = Γ = 45 οπότε Λ

ΘΖΓ = 90° . γ) Το τρίγωνο ΑΗΔ είναι ορθογώνιο και

Λ ισοσκελές (εφόσον Δ = 45° ) με ΑΗ = ΔΗ ( 1 ) . Επίσης λόγω του (β) τα τρίγωνα ΔΕΗ, ΓΖΘ είναι ίσα (γιατί;) και μάλιστα ορθογώνια και ισοσκελή . Φέρνουμε ΕΚ, ΖΛ .l ΓΔ τότε: ΕΖ = ΚΑ (2) , ΔΚ = ΚΗ = ΛΓ (3) (γιατί;).

(2),(3) Έτσι ΓΔ = ΚΛ + ΚΔ + ΛΓ = .

( I) = ΕΖ + ΚΔ + ΚΗ = ΕΖ+ ΔΗ= ΕΖ+ ΑΗ

Μία παρατήρηση για δημοσιευμένη άσκηση της Α' Λυκείου

Στο τεύχος 50/2003 του «Β ' Ευκλείδη» δημοσιεύεται άρθρο με ασκήσεις για τους μαθητές της Α' Λυκείου με τίτλο «Σχέσεις μεταξύ των συντελεστών και των ριζών του '!'υ βαθμού» του κ. Α. Μπουρνή . Επειδή το άρθρο αφορά μαθητές της Α' Λυκείου θεωρώ απαραίτητο να επισημάνω τα εξής: Στην άσκηση αυτή ζητείται να αποδειχτεί ότι α+β+γ = Ο με δεδομένο (εκτός των άλλων) ότι αβγ += 0 . Στον τρόπο επίλυσης της άσκησης ο συγγραφέας, παίρνοντας υπόψη του και τις σχέσεις αβγ += Ο, ρ1 + ρ2 = α, ρ2 + ρ3 = β, ρ3 + ρ1 = γ , καταλήγει . στη σχέση ρ1ρ2ρ3 = ±αβγ . Διακρίνο-ντας στη συνέχεια δύο περιπτώσεις οδηγείται, στην lη περίπτωση στο συμπέρασμα ρ1 = α, ρ2 = β, ρ3 = γ , οπότε α+β+γ = Ο και στη 2η περίπτωση στο συμπέρασμα ρι = -α, ρ2 = -β, ρ3 = -γ , οπότε πάλι α+β+γ = Ο. Είναι όμως προφανές ότι η l η περίπτωση πρέπει να απορριφθεί, γιατί οδηγεί σε άτοπο. Αφού αν ήταν ρ1 = α, ρ2 = β, ρ3 = γ τότε η σχέση ρ1 + ρ2 = α οδηγεί στο α + β = α {::} β = Ο που είναι άτοπο. Παρακαλώ να δημοσιευτεί στο επόμενο τεύχος του περιοδικού η ανωτέρω παρατήρηση .

ΕΥΚΛΕΙΔΗΣ Β' λζ' τ.2/50

Β. ΣΑΚΕΛΛΑΡΗΣ

Μέλος της Συντακτικής Επιτροπής

Page 53: Ευκλειδης Β 51

α · ηματικα για την Β ' τάξη του Λυκείου

Εκθετική και ΛογαpιΒμι«ή Σuν,όp·rηαη, Μ. Παπαδόπουλος

Το παρακάτω άρθρο ασχολείται με τη μονοτονία εκθετικής και λογαριθμικής συνάρτησης. Επίσης

διαπραγματεύεται εκθετικές και λογαριθμικές εξισώσεις. Η ύλη που υπάρχει στο σχολικό βιβλίο

θεωρείται γνωστή.

ΑΣΚΗΣΗ 1

Ν α λυθεί η εξίσωση 7 ( 1 1 + 6.J2)' = 3 -.J2 . ΛΥΣΗ

Η εξίσωση ορίζεται στο IR 7 (1 1 + 6h)x = 3 -h � � 7 (32 + 2 · 3 ·h + (h)2 )x = 3 -h � � 7[(3 +h)2T = 3 -h �

<c>Ψ3 +F2J'T = (3 -�!ι;;F2J � � 7(3 +h)2x = 39+

-1 � � 7(3 +.J2( = 7(3 +.J2Γ � � 7(3 +h( = 7 (3 +h( �

1 � 2χ = -1� χ = -- . 2 ΑΣΚΗΣΗ 2

Να βρεθεί ο θ ε IR: ώστε οι τιμές y, z που προ­

κύπτουν από τη λύση του συστήματος {yfogz + zfogy = 20 r:::: να είναι ρίζες της εξίσωσης RogνyL. = 1 Rog ( Rog(x2 + xfogθ + 110)) = Ο .

ΛΥΣΗ

Πρέπει z>O και y>O.

{Yfogz + zfogy = 20 {(1 0togy γogz + ( 10togz )'ogy

C � ι � Rogν YL = 1 Rog(yz)2 = 1 {1 otogz·fogy + 1 otogy·fogz � 1 = 20� 2Rog(yz) = 1 {2 · 1 otogz·fogy = 20 � � Rog(yz) = 2 {1 otogz·togy = 1 Ο {Rogz · Rogy = 1 � � � fogy + Rogz = 2 Rogz + Rogy = 2 � Rogy = Rogz = 1 � y = z = 10 Η δοθείσα εξίσωση ισοδύναμα γράφεται Rog ( Rog(x2 + xfogθ + 1 1 0)) = Ο� � Rog(x2 + xfogθ + 1 1 0) = 1 � � χ2 + xfogθ + 1 1 0 = 10 � � χ2 + xfogθ + 100 = 0 ( 1 ) Δεδομένου ότι y = z = 1 0 είναι ο ι ρίζες της ( 1 ) . Θα ισχύει 1 00 + 1 0R og Θ + 1 00 = Ο � � RogΘ = -20 � Θ = 10-20 ΑΣΚΗΣΗ 3

Να βρεθεί το πεδίο ορισμού της συνάρτησης f(x) = Jx( Rog2 (x + 2)- Rog(x + 2)) . Λύση

Για να ορίζεται η f πρέπει χ + 2 > 0 � χ > -2 και χ { fog2 (x + 2) - Rog(x + 2)) ;?: Ο � � xfog(x + 2) · ( fog(x + 2) - 1) ;?: Ο

( 1 )

( � )

ΕΥΚΛΕΙΔΗΣ Β' λζ ' τ.3/51

Page 54: Ευκλειδης Β 51

Μαθηματικά Β · Λυκείου

Έστω Rog(x + 2) � Ο � χ + 2 � 1 � χ � -1 και Rog(x + 2) -1 � Ο� Rog(x + 2) � 1 � � χ + 2 � 1 0 � χ � 8 Έτσι για το πρόσημο του γινομένου χ · Rog(x + 2) · (fog(x + 2) - 1] με χ > - 2 έχουμε τον παρακάτω πίνακα.

χ -2 -1 ο 8 +ao - - + + χ

log(x+2) - + + +

log(x+2)-1 - - - I +

log(x+ 2)[log(x+2)-1 ] - ψ + ψ - ψ +

Τότε για χ > - 2 η (2) αληθεύει όταν -1 � χ � Ο ή χ � 8 . Άρα το πεδίο ορισμού της f είναι

Α = [ -1, Ο) υ [8, -too) .

ΑΣΚΗΣΗ 4 Για ποια τιμή του χ οι αριθμοί

Rnx,tl10lnx , (fnx)eog(Rnx) είναι διαδοχικοί όροι

γεωμετρικής προόδου;

ΛΥΣΗ Πρέπει χ>Ο και Rnx > Ο � χ > Ο και χ > 1 , άρα χ>1 . Οι Rnx, tl10Rnx , (lnx γog(Rnx ) θα είναι διαδοχικοί ό-ροι γεωμετρικής προόδου αν και μόνο αν (tl1 0Rnx )2 = ln x · (R n xγog( en x) �

� ( n )l+fog(f n x) � ν 1ut n x = -e n x � O n [( n )Ι+ Ιog( Ι n x J J2 � 1 -e n x = -e n x �

� 10R nx = (R n x)2+2 Ι og( f n x) ( 1 ) Θέτουμε Rnx = ω > Ο αφού χ > Ο τότε από την ( 1 ) έχουμε 1 0ω = ωz+ztσgω � Rog(1 0ω) = Rogωz+togω � � log10 + logω = (2 + 2logω)logω � � 1 + logω = 2Rogω + 2Rog2ω � � 2log2ω + Rogω - 1 = Ο � � (Rogω =� ή Rogω = -1)� � {ω = 101/2 ή ω = 10-1)� � (ω =Μ η ω = 1�} Αν ω = JϊΟ τότε Rnx =Μ � χ = e.JίO .

1 1 _!_ Αν ω =- τότε Rnx =-� x = e10 • 1 0 10

I

ο . . δ , , .JίΟ 10 > 1 ι τιμες του χ ειναι εκτες αφου e , e . ΑΣΚΗΣΗ S Για χ>Ο και y>O να δεχτεί ότι ισχύει η aνίσωση Χ' . yY � Xy . y' •

ΛΥΣΗ Αν χ = y η ζητούμενη ισχύει ως ισότητα, θα δεί­ξουμε ότι ισχύει και για χ * y Η συνάρτηση f (x ) = Rog x στο (Ο, +οο) είναι γνήσια αύξουσα, τότε η ζητούμενη ανίσωση ισιr δύναμα γράφεται: Rog(x' · yY ) � Rog(xY · y' ) � � Rogx' + RogyY � RogxY + Rogy' � � χ · Rogx + y · Rogy � y · Rogx + x · Rogy� � ( x - y) · ( Rogx - Jiogy ) � Ο ( 1 ) Παρατηρούμε ότι: ί) Α ν ( χ - y) � χ - y > Ο και

logx > Rogy � Rogx - Rogy > Ο ii) Αν χ < y � χ - y < Ο και

Rogx < Rogy � logx - Jiogy < Ο Δηλαδή οι αριθμοί χ - y και Rogx - Rogy είναι ομόσημοι. Άρα για κάθε χ > Ο και y > Ο ισχύει η ( 1 ) . Επομένως ισχύει και η δοθείσα ανίσωση.

ΑΣΚΗΣΗ 6 Δίνεται η συνάρτηση

f(x) = log(l lx2 - 7χ + 10) - Rogx2 - 1 .

i) Να βρεθούν τα σημεία τομής της γραφικής

παράστασης της f με τον χ 'χ άξονα.

ίί) Αν Β, Γ είναι τα παραπάνω σημεία και Α(Ο,4)

σημείο του y 'y να βρεθεί το εμβαδόν του τρι­

γώνου ΑΒΓ.

ΛΥΣΗ Το τριώνυμο 1 1χ2 - 7χ + 10 έχει Δ < Ο άρα για κάθε χ ε IR. ισχύει 1 1χ2 - 7χ + 10 > Ο .

• Έτσι το πεδίο ορισμού της f είναι το R . ί) Οι τετμημένες των σημείων τομής της γραφικής

παράστασης της f με τον χ'χ είναι λύσεις της εξίσωσης f(x) = Ο δηλαδή Rog(1 1x2 - 7χ + 1 0) - Rogx2 - 1 = Ο�

ΕΥΚΛΕΙΔΗΣ Β' λζ' τ.3/52

Page 55: Ευκλειδης Β 51

Μαθηματικά Β ' Λυκείου

Ι Ιχ2 - 7χ + 1 0 Ι <=:> log 2 = <==> χ Ι Ιχ2 - 7χ + 10 Ι Ο <==> = <==> χ2

<=:> Ι Ιχ2 - 7χ + 10 = 10χ2 <=:> <=:> Ι Ιχ2 - 7χ + 1 0 - 10χ2 = Ο <=:> <=:> χ2 - 7χ + 1 0 = 0<::> χ = 2 ή χ = 5 . Άρα τα σημεία τομής είναι Β(2,0) και Γ(5 ,0)

ίί) Από το παρακάτω σχήμα

Α(0,4)

Ο Β(2,0) Γ(5,0) Έχουμε ΒΓ = 3 , ΟΑ = 4 οπότε: Ι Ι Ε ABr = -ΒΓ · ΟΑ = - · 3 · 4 = 6τ.μ . . 2 2

ΑΣΚΗΣΗ 7

( 2003 )'' -3χ ( 2003 )Ζχ-6

Να λυθεί η aνίσωση 2004 < 2004 ΛΥΣΗ

Η συνάρτηση f(x) = ( 2003 )χ είναι εκθετική με 2004 2003 Ι . δ ' . 1!]) βάση Ο < α =--< , εχει πε ιο ορισμου το m.. . 2004

Και είναι γησίως φθίνουσα στο R. Άρα η ανίσωση ( 2003 )x' -Jx < ( 2003 )Ζχ-6 <==> 2004 2004 f(x2 - 3χ) < f(2x - 6) <=:> χ2 - 3χ > 2χ - 6 <==> <==> χ2 - 3χ - 2χ + 6 > ο <==> Χ2 - 5χ + 6 > ο <==> χ < 2

ή χ > 3 . ΑΣΚΗΣΗ 8 Δίνεται η συνάρτηση f(x) = 3' + χ .

i) Να δείξετε ότι είναι γνησίως αύξουσα στο R.

ίί) Ν α λύσετε την εξίσωση 3χ1 -3χ + Χ2 + 5 = 33x-S + 6χ •

iii) Να λύσετε την aνίσωση 3χ1 -3χ + Χ2 + 5 < 33x-S + 6Χ •

ΛΥΣΗ i) Το πεδίο ορισμού της f είναι το R.

Τότε για κάθε χ, , χ2 Ε 1R με χ, < Χ2 . ισχύει 3χ' < 3χ' άρα και 3χ' + χ, < 3χ' + Χ2

Έτσι f(x1 ) < f(x2 ) δηλαδή η f είναι γνήσια αύ­ξουσα.

ii) Η εξίσωση ορίζεται στο JR .

Τότε 3x' -Jx + χ2 + 5 = 33x-s + 6χ <=:> <==> 3x' -Jx + χ2 - 3χ = 33x-S + 6χ - 3χ - 5 <==> <==> 3χ' -3χ + χ2 - 3χ = 33χ-5 + 3χ - 5 <==> <=:> f(x2 - 3χ) = f(3x - 5) . Επειδή η f είναι γνησίως αύξουσα θα είναι και συνάρτηση Ι - Ι . Έτσι χ 2 - 3 χ = 3χ - 5 <=:> χ 2 - 6χ + 5 = Ο <=:> <=:> χ = Ι ή χ = 5 .

ίίί) 3x' -Jx + χ2 + 5 < 33x-S + 6χ <==> <=:> . . . . <=:> f(x2 - 3χ) < f(3x - 5) Επειδή η f είναι γνησίως αύξουσα προκύπτει χ2 - 3χ < 3χ - 5 <=:> χ2 - 6χ + 5 < Ο <==> Ι < χ < 5 .

ΑΣΚΗΣΗ 9 Δίνεται η συνάρτηση f(x) = α' + α2 (χ - 1) - αχ

με ο < α < ι. i) Να δειχτεί ότι η f είναι γνησίως φθίνουσα

στο JR . ίί) Να λυθεί η aνίσωση α' + α2χ < α(χ + α) .

ΛΥΣΗ i) Το πεδίο ορισμού της f είναι το JR

Θα δείξουμε ότι η f είναι γνήσια φθίνουσα δη­λαδή για κάθε χ, , χ2 Ε JR με χ, < χ� θα δεί-ξουμε ότι f ( χ , ) - f ( χ2 ) > Ο . Έτσι f ( x , ) - f ( x2 ) = [αx' + α2 ( χ , - Ι ) - αχ, ] ­[ αχ' + α2 ( χ2 - Ι) - αχ2 ] =

= . . . = αχ' - αχ' + α2 ( χ1 - χ2 ) - α ( χ, - Χ2 ) = =αχ' + αχ' + ( χ , - χ2 ) ( α2 - α) ( Ι ) Επειδή έχουμε Ο < α < 1 για χ1 < χ2 θα ισχύει αχ' > αχ' ή αχ' - αχ' > Ο (2) και α2 - α = α ( α - Ι ) < 0 τότε ( χ, - χ2 ) · α ( α - Ι ) > Ο (3) Από τις Ι , 2, 3 προκύπτει ότι f ( χ, ) - f ( χ2 ) > Ο

ίί) αχ + α2χ < α( χ + α) <=:> αχ + α2χ < αχ + α2 <=:> <=:> αχ + α2χ - α2 - αχ < Ο<=:> <=:>αχ + α2 (χ - Ι) - αχ < O<=:> f(x) < f( I) <=:> χ > Ι αφού f(l) = α' + α2 (Ι - Ι) - α · Ι = Ο και f γνησίως φθίνουσα στο R.

ΕΥΚΛΕΙΔΗΣ Β' λζ' τ.3/53

Page 56: Ευκλειδης Β 51

Μαθηματικά Β ' Λυκείου

Κανονικά Πολύγωνα - Μέτpηαη Κύκλου

ΑΣΚΗΣΕΙΣ

1) Σ' ένα κανονικό ν-γωνο ΑιΑ2 • • . Αν, όπου ν�6 , Λ

ισχύει Α1 Α3 Α6 = 120· . Να βρείτε το ν.

Λύση

Το τόξο Α1Α3Α6 περιέχει 5 πλευρές του -πολυγώνου. Συνεπώς το τόξο Α1Α6 , που δεν περιέχει το Α3, περιέχει ν-5 πλευρές αυτού και συνεπώς αποτελείται από ν-5 τόξα ίσα με 360° 'Ε , --. τσι εχουμε: ν ------- 1 360° 1 80° Α1Α3Α6 = - · (ν - 5)--= (ν - 5) ·- . 2 ν ν

Έχουμε λοιπόν: ------ ο 1 80 Α1Α3Α6 = 120 {:} (ν - 5) ·-= 120 {:} ν

3 <=> (ν - 5) · - = 2 <=> 3ν - 1 5 = 2ν <=> ν = 1 5 . ν 2) Δύο μη διαδοχικές πλευρές ΑΒ και ΜΝ

ενός κανονικού ν-γώνου προεκτεινόμενες

τέμνονται σ' ένα σημείο Κ. Να δείξετε ότι:

ΑΚΝ � 1soo . ν

Λύση -Έστω ότι το τόξο ΒΣΜ (διπλανό σχήμα) περιέχει λ πλευρές του ν-γώνου, οπότε το τόξο --ΑΤΝ θα περιέχει ν-λ-2 πλευρές αυτού. Έτσι, -το τόξο ΒΣΜ αποτελείται από λ τόξα ίσα με 3600 ' ξ Α..-ΤΝ λε' ' λ 2 -- και το το ο αποτε ιται απο ν- -ν ' ξ , 360° 'Ε , το α ισα με -- . τσι, εχουμε: ν

Α

τ

της Χρυσταλλένης Κυριακοπούλου - Κυβερνήτου

ΑΚΝ = k(ΑτΝ-ΒΣΜ) =

= .!_ [(ν - λ - 2) · 360 - λ · 360 ] = 2 ν ν ν - 2λ - 2 360 ---- ·-=? 2 ν ---- 1 80 ΑΚΝ= (ν - 2λ - 2) ·-ν (1 )

Ο αριθμός ν-2λ-2 είναι φυσικός και επειδή από την ( 1 ) έπεται ότι είναι θετικός, συπεραίνουμε ότι ν - 2λ - 2 � 1 . Έτσι, από την ( 1 ) έπεται ότι:

ΑΚΝ � 1 80 . ν 3) Θεωρούμε ένα κανονικό πεντάγωνο

ΑΒΓ ΔΕ με πλευρά λ5 = α . Η διαγώνιος ΑΓ τέμνει τις διαγώνιες ΒΕ και ΒΔ στα σημεία

Ζ και Η, αντιστοίχως. Να δείξετε ότι:

i) ΑΖ = ΗΓ = ; ( J5 - 1)

ii) ΖΗ = ; ( 3 -JS) iii) AΓ =i(1 + J5) .

Λύση --. i) Επειδή ΑΕ = ΓΔ , έχουμε AΓ I I ΕΔ . Όμοια

ΒΕ 1 1 ΓΔ και ΒΔ 1 1 ΑΕ . Έτσι, τα τετράπλευρα ΑΗΔΕ και ΓΖΕΔ είναι παραλληλόγραμμα, οπότε: ΑΗ = ΕΔ = ΖΓ(= α) . Έτσι, έχουμε: ΑΗ= ΖΓ=?ΑΖ+ ΖΗ= ΖΗ+ ΗΓ=? ΑΖ= ΗΓ. Θέτουμε: ΑΖ = χ. Τα τρίγωνα ΑΒΖ και ΑΓΒ -είναι όμοια, γιατί έχουν τη γωνία ΒΑΓ κοινή - -και ΑΒΖ = ΑΓΒ (βαίνουν σε ίσα τόξα):Ετσι, έχουμε: ΑΖ ΑΒ χ α 2 2 - =- <=> - =-- <=> χ + αχ - α = 0 <=> ΑΒ ΑΓ α χ + α <::> x =� (JS- 1)

ΕΥΚΛΕΙΔΗΣ Β' λστ ' τ.l/54

Page 57: Ευκλειδης Β 51

Μαθηματικά Β' Λυκείου

Β

Άρα: ΑΖ = ΗΓ = � ( .J5 - 1 ) . ίί) ΖΗ = ΖΓ - ΗΓ = α- � ( .J5 - 1) = � ( 3 - .Js) . iii) ΑΓ = ΑΖ + ΖΓ = � ( .J5 - 1 ) + α = � ( 1 + .Js) . 4) Θεωρούμε ένα κανονικό 15-γωνο ΑιΑ2 ... Αιs

εγγεγραμμένο σε κύκλο (0, R). Ονομάζουμε

Β το aντιδιαμετρικό σημείο του Αιs· Να

δείξετε ότι: ΒΑ1 • ΒΑ2 • • • • ΒΑ7 = R7 (1).

λίιση Το τρίγωνο ΒΑ1Α1 5 είναι ορθογώνιο και η ευθεία ΒΑ1 5 είναι μεσοκάθετος του τμήματος ΑιΑι4· Έτσι, το εμβαδόν του τριγώνου ΒΑιΑι5 είναι: _!_ ΒΑ1 · Α1Α1 5 και επίσης είναι: 2 _!_ΒΑ . ΑιΑι4 = _!_ · 2R · Α2Αι s = _!_R · Α2Αι . 2 1 5 2 2 2 2 5

'Ετσι έχουμε: _!_ ΒΑ1 · Α1Α1 5 = _!_ R · Α2Α1 5 , 2 2

οπότε: ΒΑ1 · Α1Α1 5 = R · Α2Α1 5

Όμοια: ΒΑ2 . Α2Αι 5 = R . Α4Αι5 ΒΑ3 . Α3Αι 5 = R . Α6Αι 5 ΒΑ4 · Α4Α1 5 (= R · A8A1 5 ) = R · A7A1 5 ΒΑ5 · Α5Α1 5 (= R · Α10Α1 5 ) = R · Α5Α1 5 ΒΑ6 · Α6Α1 5 (= R · Α12Α1 5 ) = R · Α3Α1 5 ΒΑΊ · Α7Αι5 (= R · Αι4Αι 5 ) = R · ΑιΑι 5 ·

5)

Πολλαπλασιάζουμε τις ισότητες αυτές κατά μέλη και μετά τις απλοποιήσεις φθάνουμε στην ζητούμενη ισότητα ( 1 ) .

Η διαφορά των εμβαδών ενός κανονικού

εξαγώνου και ενός τετραγώνου, που είναι

εγγεγραμμένα σ' έναν κύκλο (0, R), είναι

5,5m2• Να βρείτε την ακτίνα R του κύκλου.

Λύση

'Ε Ε 1 Ρ Ε δ ' RJ3 χουμε: 6 = Ξ 6 · α6 . πει η α6 = -2- και Ρ6 = 6λ6 = 6R , έχουμε:

Ε = _!_ · 6R · RJ3 = 3R2J3 . 6 2 2 2

Επίσης, έχουμε: Ε4 = λ� = (RJ 2y = 2R2 .

Σύμφωνα με το πρόβλημα, έχουμε: Ε - Ε - 5 5 3R 2 J3 2R2 - .!..!_ 6 4 - ' {::} - {::}

2 2 3R 2 J3 - 4R 2 = 1 1 {::} R 2 ( 3J3 - 4) = 1 1 {::}

R 2 = 1 1 <=> R 2 = 1 1 ( 3/3 + 4) <=>

3/3 - 4 27 - 16 <:::> R 2 = 3J3 + 4 {::} R = �3J3 + 4 m .

6) Έστω Ο το κέντρο ενός τετραγώνου ΑΒΓ Δ. Να δείξετε ότι τα έγκεντρα των οκτώ

τριγώνων: ΑΟΒ, ΒΟΓ, ΓΟΔ, ΔΟΑ, ΑΒΓ, ΒΓ Δ, Γ ΔΑ και ΔΑΒ είναι κορυφές

κανονικού οκταγώνου. Μετά, να βρείτε τον

λόγω των εμβαδών του οκταγώνου αυτού

και του τετραγώνου.

Λύση

1) Ονομάζουμε Ι ι , l3, ls, 17, Iz, 14, 16 και ls τα έγκεντρα, κατά σειρά, των τριγώνων αυτών.

Α Κ Β ,, ' \ ' ' \ ' \ ' ,

ΕΥΚΛΕΙΔΗΣ Β' λζ' τ.3/55

Page 58: Ευκλειδης Β 51

Μαθηματικά Β' Λυκείου

Έχουμε: - - - 1 ο ο 1 ο 011 12 = 11ΑΟ + Α011 = - · 45 + 45 = - · 1 35 2 2 Επίσης, έχουμε: - - - 1 ο ο 1 ο 01211 = ΒΑ12 +ABI2 = - · 45 + 45 = - · 1 35 2 2 -Έτσι, έχουμε 011 12 = 01211 και συνεπώς ΟΙ1 = ΟΙ2. Όμοια δείχνουμε ότι: 012 = 013 = . . . = 018 και

, --- --- --- ο επειδη Ιι012 = 12013 = . . . . = 18011 = 45 , συμπεραίνουμε ότι το οκτάγωνο 1 112 . . . . 18 είναι κανονικό με κέντρο το Ο.

2) Ονομάζουμε Ε8 το εμβαδόν του οκταγώνου και Ε4 του τετραγώνου. Έστω ότι η ευθεία Ol1 τέμνει την πλευρά ΑΒ στο σημείο Κ. Έχουμε:

Ε8 = 8 · (011 12 ) = 011 · 012 _ ΟΙ� ( 1 ). Ε4 8 · (0ΚΒ) ΟΚ · ΟΒ ΟΚ · ΟΒ Στο τρίγωνο ΟΑΒ, η Al2 είναι διχοτόμος της -γωνίας ΟΑΒ , οπότε: 012 ΑΟ 012 ΑΟ - = - � = � 12Β ΑΒ 012 + 12Β ΑΟ+ ΑΒ � 012

= ΑΟ ____._ ΟΙ = ΟΑ · ΟΒ (2) ΟΒ ΑΟ + ΑΒ �

2 ΟΑ + ΑΒ . Εξάλλου, βρίσκουμε εύκολα ότι:

ΑΒ ΑΒ ΟΑ = ΟΒ = fi και ΟΚ =τ · Έτσι, από τη (2), έχουμε:

ΟΑ2 ΟΑ + ΑΒ

ΑΒ2 2

ΑΒ fi +AB

Αντικαθιστώντας στην ( 1 ), βρίσκουμε:

§_ _ [ν'2� 2 )2

6 +�ν'2 4 Ε4 ΑΒ . ΑΒ fi 6fi+ 8 2 fi 4 = 2 = 2 (3!i- 4) = 3fi- 4 . 3fi + 4 1 8 - 1 6

7) Ένα κανονικό ν-γωνο ΑΒΓ.... είναι εγγεγραμμένο σ' έναν κύκλο (0, R). Φέρ­νουμε την ακτίνα ΟΚ//ΑΒ και τη χορδή .----. Β Κ. Από το μέσο Λ του τόξου ΑΒ φέρνουμε τη χορδή ΛΜ/ΙΒΚ. Να δείξετε ότι το

εμβαδόν Ε ' του μέρους του κυκλικού δί­σκου που περιέχεται μεταξύ των παραλ­λήλων αυτών χορδών ΒΚ και ΛΜ είναι ίσο

ι με το - του εμβαδού Ε του κυκλικού

ν αυτού δίσκου.

Λύση ----Επειδή ΑΛ = ΑΒ = ΚΜ , έχουμε: - - --ΑΟΑ = ΑΟΒ = ΚΟΜ(= σ) . Επίσης, έχουμε: - 360° ΑΟΒ = 2σ =--(= ων ) . Εξάλλου, έχουμε: ν ΟΑ ..l ΑΒ και ΑΒ//ΟΚ, οπότε ΟΑ ..l ΟΚ και συνεπώς ΛΟΚ = 90° . Έτσι, έχουμε: ΛοΜ: = 90° + σ και ΒοΚ = 90° - σ . Παρατηρούμε ότι: ΛοΜ: + ΒοΚ = (90° + σ) + (90° - σ) = 1 80° , οπότε: (ΟΑΜ) = ΟΑ · ΟΜ = R2 = 1 �

(ΟΒΚ) ΟΒ · ΟΚ R2 � (ΟΛΜ) = (ΟΒΚ) .

Λ

Έτσι, έχουμε: Ε'=(εμβ.κυκλ τμημΛΝΜ)-(εμβ.κυκλ τμημΒΝΚF = [(δΛΜ)- ( ΟΛΜ)] - [ (δΒΚ) - ( ΟΒΚ)] = = (δΛΜ)- (δΒΚ) = πR2 πR2 πR2 = 360 · (9ο + σ) - 360 · (9ο - σ) = 36ο

. 2σ = = πR2 . 360 = _!_πR2 = _!_ · Ε . 360 ν ν ν

8) Θεωρούμε δύο ομόκεντρους κύκλους (Ο, R) και (Ο, 2R). Μία χορδή ΑΒ του μεγαλύτερου κύκλου εφάπτεται στο Μ του μικρότερου κύκλου. Από το Α φέρνουμε την εφαπτομένη στον μικρότερο κύκλο και ονομάζουμε Ν το σημείο επαφής. Να δείξετε ότι το εμβαδόν Ε ' της περιοχής που

ΕΥΚΛΕΙΔΗΣ Β' λζ' τ.3/56

Page 59: Ευκλειδης Β 51

Μαθηματικά Β ' Λυκείου

περικλείεται από το (μικρότερο) τ� ΒΑ , το τμήμα ΑΝ, το (μικρότερο) τόξο ΝΜ και το τμήμα ΜΒ, είναι ίσο με το εμβαδόν Ε

του μικρότερου κύκλου.

Λύση

- ο - ο Επίσης ΜΟΒ = 60 και ΑΟΝ = 60 , οπότε - - ο ΑΟΒ = 120° και ΜΟΝ = 1 20 . Επίσης, έχουμε: (ΟΜΑΝ) = 2(0ΜΑ) = (ΟΑΒ) . Έτσι έχουμε: Ε' = (εμβ.κυκλ.τμημ.ΑΚΒ) + +(ΟΜΑΝ) - (ΟΜΝ) = (ΟΑΒ) - (ΟΑΒ) +

--

+(ΟΑΒ) - (ΟΜΝ) = π(2R)2 πR 2 4πR 2 πR 2 ----'--------'---- · 1 20 - - · 1 20 = -- -- = 360 360 3 3

= πR2 = Ε .

9) Σε μία διάμετρο ΑΒ ενός κύκλου με ακτίνα R παίρνουμε ένα σημείο Γ. Με διαμέτρους τα τμήματα ΑΓ και ΓΒ γράφουμε δύο ημικύκλια εκατέρωθεν της διαμέτρου ΑΒ. Τα δύο αυτά ημικύκλια χωρίζουν τον κυκλικό δίσκο σε δύο μέρη με εμβαδά Ε ·

και Ε ", αντιστοίχως (το πρώτο μέρος περιέχει το τμήμα ΑΓ και το δεύτερο το ΓΒ). Να δείξετε ότι: 1) Το άθροισμα των μηκών των δύο ημι­

κυκλίων είναι ίσο το μήκος ενός ημικυ­κλίου με διάμετρο το ΑΒ. Ε' ΑΓ

2) Ε" = ΓΒ

.

Λύση

Θέτουμε: ΑΓ = 2r και ΓΒ = 2r, οπότε: ΑΒ = 2ρ + 2r = 2R και άρα: ρ + r = R .

1) Το άθροισμα των μηκών των δύο ημικυκλίων είναι: 1 1 1 1 - · 2πρ +- · 2πr = - · 2π(ρ + r) = - · (2πR) . 2 2 2 2

2) Έχουμε: 1 2 1 2 1 2

Ε' 2πR + lπρ - 2πr R2 + ρ2 - r2 _ Ε" = _!_πR 2 + _!_πr2 _ _!_πρ2

- R 2 + r2 - ρ2 2 2 2

R2 + (ρ + r)(ρ - r) _ R2 + R(ρ - r) =

R2 + (r + ρ)(r - ρ) R2 + R(r - ρ) R + ρ - r ρ + ρ 2ρ _ ΑΓ = R + r - ρ = r + r = 2r - ΓΒ ·

10) Δίνεται ένας κύκλος C(O,R) . Από ένα

σημείο Μ που δεν ανήκει στο εσωτερικό του κύκλου C διέρχονται δύο κάθετες μεταξύ τους ευθείες που η κάθε μία τέμνει ή εφάπτεται στον C. Να βρείτε το εμβαδόν της περιοχής που διαγράφει το σημείο Μ.

Λύση

Έστω Μ ένα τέτοιο σημείο, οπότε ΟΜ 2:: R . Από το Μ διέρχονται δύο κάθετες μεταξύ τους ευθείες Μχ και My που η κάθε μία τέμνει ή εφάπτεται στον C. Φέρνουμε ΟΑ ..l Μχ και ΟΒ ..l My . Έχουμε: ΟΑ � R και ΟΒ � R . Από το ορθογώνιο τρίγωνο ΟΑΜ, έχουμε: ΟΜ2 = 0Α2 +ΑΜ2 = 0Α2 + 0Β2 :::: R2 + R2 => :::> ΟΜ2 ::; 2R2 :::> ΟΜ ::; R.J2 .

c

Έχουμε λοιπόν: R � ΟΜ � R .J2 . Άρα, το Μ ανήκει στο δακτύλιο όπου σχηματίζουν οι δύο κύκλοι C(O,R) και C'(O, R.J2) .

ΕΥΚΛΕΙΔΗΣ Β' λζ' τ.3/57

Page 60: Ευκλειδης Β 51

Μαθηματικά Β' Λυκείου

Α ντιστρόφως.

Έστω ένα σημείο Ν του δακτυλίου αυτού, οπότε: R :ς ΟΝ :ς R J2

. Φέρνουμε την ημιευθεία Nt με δΝi: = 45° και την ΟΓ ..l Ot . Από το ορθογώνιο και ισοσκελές τρίγωνο ΟΝΓ(ΟΓ = ΓΝ) β ' ' λα ' ΟΓ ΟΝ ρισκουμε ευκο οτι = J2 . 'Ε ' ΟΓ ΟΝ R J2 R δ λ δ ' τσι εχουμε: = J2 :ς J2 = , η α η

ΟΓ :ς R και άρα η Nt τέμνει τον κύκλο C ή εφάπτεται σ' αυτόν. Όμοια και η συμμετρική ημιευθεία Nt' της Nt ως προς την ΝΟ. Προφανώς Nt ..l Nt' . Άρα, από το Ν διέρχ­ονται δύο τέτοιες ευθείες. Συνεπώς, η περιοχή που διαγράφει το σημείο Μ είναι ο αναφερόμενος παραπάνω δακτύλιος. Το εμβαδόν Ε ' του δακτυλίου αυτού είναι:

E = π(RJ2)2 - πR2 = πR2 .

r θεωρία Αριθμών Αντώνη Κ. Τσιπρόπουλο

«Η πρόοδος στη θεωρία αριθμών είναι τόσο προχωρημένη, όχι μόνο για όσα γνωρίζουμε γι' αυτήν, αλλά γιατί είμαστε σε θέση να γνωρίζουμε και τι αγνοούμε γύρω απ' αυτήν».

ΠΑΡΑ ΤΗΡΗΣΕΙΣ 1. Η μέθοδος της μαθηματικής επαγωγής

αποτελείται από δύο βήματα τα οποία είναι απολύτως αναγκαία και τα δύο για την εγκυρότητα ενός ισχυρισμού.

lo ΒΗΜΑ Δείχνουμε πάντοτε ότι η σχέση Ρ(ν) ισχύει για ν = 1 ή ότι ισχύει η σχέση Ρ(ν0), όπου ν0 ο μικρότερος φυσικός αριθμός για τον οποίο ζητείται να αποδειχθεί η Ρ(ν).

2ο ΒΗΜΑ Δεχόμαστε ότι η σχέση Ρ(ν) για έναν αυθαίρετο θετικό ακέραιο ν � 1 και δείχνουμε ότι ισχύει η σχέση Ρ( ν + 1) , δηλαδή δείχνουμε ότι ισχύει για ν+ 1 . (Ενδείκνυται να καταγράψουμε την Ρ( ν + 1) , ώστε να γνωρίζουμε τι ακριβώς θέλουμε να δείξουμε). Για να δείξουμε την Ρ( ν + 1) έχουμε δύο επιλογές: • Ή ξεκινάμε από την Ρ(ν) δημιουργώντας την

Ρ( ν + 1) (παράδειγμα 1 ) . • Ή ξεκινάμε από το 1 ο μέλος της Ρ( ν + 1)

αξιοποιώντας την Ρ(ν) μέχρις ότου δείξουμε αυτό που θέλουμε (παράδειγμα 2).

2 . Συνήθως απαιτείται από την άσκηση να δείξουμε ότι η σχέση Ρ(ν) αληθεύει όχι για κάθε θετικό ακέραιο ν αλλά για κάθε ν � ν 0 με ν0 ε Ν (παράδειγμα 3) .

w . Sieprinski

3. Στις ανισοτεικές σχέσεις πολλές φορές παίρνουμε υπόψη μας τη μεταβατική ιδιότητα (αν α>β και β>γ, τότε α>γ) (παράδειγμα 3).

4. Η μαθηματική επαγωγή δεν εφαρμόζεται μόνο σε ταυτοτικές, ανισοτικές σχέσεις αλλά και σε ασκήσεις διαιρετότητας, πολυωνύμων, ακολουθιών, γεωμετρικών προβλημάτων κ.ά. (παράδειγμα 3).

5. Το γινόμενο των διαδοχικών φυσικών αριθμών από 1 έως ν το συμβολίζουμε με ν! και το διαβάζουμε «ν παραγοντικό», δηλαδή ν ! = 1 · 2 · 3 · · · ν Ισχύουν: i) ν ! = (ν - 1) ! ν και ii) 1 ! = 1 , Ο ! = 1 (εξ ορισμού)

6. Θυμίζουμε ότι: i) Το άθροισμα των ν διαδοχικών θετικών ακεραίων αριθμών είναι:

ν( ν + 1) , • 1 + 2 + 3 + . . . + ν = για καθ ε ν ε Ν 2 ii) Το άθροισμα των ν πρώτων θετικών περιττών αριθμών είναι:

1 + 3 + 5 + . . . + (2ν - 1) = ν2 για κάθε ν ε ΝΌ 7. Ανισότητα του Bernoulli : Έστω α ε � με

-1 � α :;t: Ο . Τότε για όλους τους θετικούς ακεραίους ν με ν � 2 ισχύει (1 + α γ > 1 + να .

ΕΥΚΛΕΙΔΗΣ Β' λζ' τ.3/58

Page 61: Ευκλειδης Β 51

Μαθηματικά Β ' Λυκείου

ΠΑΡΑΔΕΙΓΜΑΤΑ

1) Να αποδείξετε ότι για κάθε φυσικό αριθμό ν ισχύει: συν(νπ) = (-1)• (1).

Απόδειξη Έστω Ρ(ν) η ισότητα ( 1 ) που θέλουμε να αποδεί­ξουμε. • Για ν0 = Ο, η Ρ(ν) γίνεται: συν( Ο · π) = ( -1)0 � 1 = 1 , άρα Ρ( Ο) αληθής. • Έστω ότι αληθεύει η Ρ( ν), τότε θα αποδείξουμε ότι και Ρ( ν + 1) είναι αληθής δηλαδή ότι

συν [(ν + 1)π] = (- 1)"+1 • Πράγματι συν [ (ν + 1)π] = συν( π + νπ) =

( I ) � = -συν(νπ) =- ( -1)ν = ( -1 ) ν I , άρα σύμφωνα με την αρχή της μαθηματικής επαγωγής, η ( 1 ) αλη­θεύει για κάθε ν Ε .Ν .

2) α) Να βρείτε την ελάχιστη τιμή του θετικού ακέραιου ν για την οποία ισχύει η σχέση 3• > (ν + 1)2 (1).

β) Ν α αποδείξετε ότι: 3• > (ν + 1)2 για κάθε ν Ε .Ν με ν>2 (2).

Λύση

α) • Για ν = 1 , γίνεται 31 > 22 και δεν ισχί>ει. • Για ν = 2, γίνεται 32 > 32 και δεν ισχί>ει. • Για ν = 3 , είναι 33 > 42 � 27 > 1 6 που ισχί>ει. Άρα η ελάχιστη τιμή του ν Ε .Ν* για την οποία ισχί>ει η ( 1 ) είναι η ν = 3 .

β) Έστω Ρ(ν) η ανισότητα (2) που θέλουμε να αποδείξουμε. • Για ν = νο = 3, η (2) ισχί>ει, αφού 33 > 42 � 27 > 1 6 αληθής. • Έστω ότι η Ρ(ν) είναι αληθής, τότε θα αποδείξουμε ότι και η Ρ(ν+ 1 ) είναι αληθής, δηλαδή 3ν+Ι > (ν + 2)2 .

(2) Πράγματι 3ν+1 = 3ν · 3 > (ν + 1)2 · 3 , αρκεί να αποδείξουμε ότι 3(ν+ 1)2 > (ν + 2)2 � �3(ν2 + 2ν+ 1) > � + 4ν+4 � 2ν(ν + 1) > 1 που ισχί>ει αφού ν � 3 . Τελικά: 3ν+Ι > 3(ν + 1)2 > (ν + 2)2 δηλαδή Ρ( ν + 1) αληθής.

3) Αν α,β > Ο και α + β = 2, να αποδείξετε ότι: α2ο04 + β2004 � 2 •

Απόδειξη Αφού α,β>Ο και α+β = 2, υπάρχει θ > Ο , τέτοιο, ώστε α = 1 + θ, β = 1 - θ οπότε από ανισότητα Bemoulli και το α) ερώτημα έχουμε α2ο04 + β2οο4 = (1 + θ)2ο04 + (1 - θ)2ο04 > > 1 + 2004θ + 1 - 2004θ = 2 . i) Η ισότητα ισχί>ει όταν α = β = 1 .

ΚΑΙ ΤΩΡ Α Η ΣΕΙΡ Α ΣΑΣ . . . . 1 .

2.

3.

Ν α αποδείξετε ότι για κάθε θετικό ακέραιο ν ισχί>ει: i) 21 + 22 + 23 + . . . . + 2ν = 2(2ν - 1) . ii) 13 + 33 + 53 + . . . + (2ν - 1)3 = ν2 (2ν2 - 1) . Να εξετάσετε αν για κάθε θετικό ακέραιο ν ι­σχί>ει η σχέση 12 + 22 + 32 + . . . + ν2 = 2ν + 3

5 Να αποδείξετε ότι: i) γ > 7ν - 1 για κάθε θετικό ακέραιο. ii) 3ν > ν3 για κάθε ακέραιο ν � 4 . iii) ν ! > 2ν για κάθε ν � 4 .

Β. ΕΥΚΛΕΙΔΕΙΑ ΔΙΑΙΡΕΣΗ

ΠΑΡΑΤΗΡΗΣΕΙΣ 1. Ευκλείδεια ή αλγοριθμική διαίρεση του α με

τον β λέγεται η διαδικασία εύρεσης των κ και υ. Το κ λέγεται πηλίκο και το υ υπόλοιπο της διαίρεσης.

2. Αν σε μια ευκλείδεια διαίρεση είναι υ = Ο, τότε έχουμε α = κβ και η διαίρεση λέγεται τέλεια.

3. Τα δυνατά υπόλοιπα του α με το β>Ο είναι οι αριθμοί: Ο, 1 ,2 , . . . ,β - 1

4. Κάθε ακέραιος α, έχει σύμφωνα με τα παραπάνω, μια από τις μορφές: • α = 2κ, κ Ε Ζ (άρτιος) ή α = 2κ + 1, κ Ε Ζ (περιττός) (όταν διαιρέτης β = 2) .

• α = 3κ, κ Ε Ζ ή α = 3κ + 1, κ Ε Ζ ή • α = 3κ + 2, κ Ε Ζ ή α = 4κ + 3 κ.τ.λ. • α = 4κ, κ Ε Ζ ή α = 4κ + 1, κ Ε Ζ ή • α = 4κ + 2, κ Ε Ζ ή α = 4κ + 3 κ.τ.λ.

5. Το γινόμενο δύο διαδοχικών ακεραίων είναι άρτιος αριθμός.

Γ. ΔΙΑΙΡΕΤΟΤΗΤΑ ΠΑΡΑΤΗΡΗΣΕΙΣ 1. Ο β διαιρεί τον α (β/α) σημαίνει:

• υπολ(α:β) = Ο • υπάρχει ακέραιος κ, τέτοιος, ώστε α = κβ.

ΕΥΚΛΕΙΔΗΣ Β' λ.ζ' τ.3/59

Page 62: Ευκλειδης Β 51

Μαθηματικά Β' Λυκείου

• ο β είναι παράγοντας του α • α διαιρείται με τον β. • ο α είναι πολλαπλάσιο του β (α = πολβ).

2. Α ν α, β ακέραιοι τότε α + β, α - β, α · β ακέραιοι

� ακέραιος, με α = πολβ. β 3) i) Άρτιος ± άρτιος = άρτιος.

Περιττός ± περιττός = άρτιος. Άρτιος ± περιττός = περιττός. Περιττός ± άρτιος = περιττός. (οι αποδείξεις απλές)

ii)

iii)

iv)

4. Αν α, β, γ Ε Ζ με β * Ο, γ * Ο και α = βγ, τότε β/α και γ/α.

5. Αν α, β, γ Ε Ζ με α/(β+γ) και α/β, τότε α/γ. 6. Α ν α, β ακέραιοι

τότε (α - β) /(αν - βν ), ν Ε Ν* . (α + β) /(αν + β ν ) , ν περιττός φυσικός αριθμός. (α + β) /(αν - βν ) , ν άρτιος φυσικός αριθμός. Οπότε έχουμε: i) αν - βν = πολ(α - β) ή

αν = πολ(α - β) + β\ν Ε Ν* , ii) αν + βν = πολ(α + β) ή αν = πολ(α + β) - βν ,

ν περιττός φυσικός. iii) α ν - β ν = πολ( α + β) ' ν άρτιος φυσικός

αριθμός. iv) ( α + β) ν = πολα + β\α, β Ε Ζ, ν Ε Ν* (βλέπε άσκηση 9) . v) ( α - β) ν = πολα + (- 1γβν , α, β Ε Ζ, ν Ε Ν* . Στη συνέχεια γίνεται μια προσπάθεια για ομαδοποίηση αρκετών ασκήσεων με γνώμονα το περιεχόμενό τους ώστε η μετάγγιση της γνώσης να πετυχαίνεται με απλό και κατανοητό τρόπο και με σκοπό να κεντρίζει το ενδιαφέρον του μαθητή, αναπτύσσοντας παράλληλα τις δεξιότητές του και χωρίς να «αιχμαλωτίζεται» ή να αλλοιώνεται η γοητεία της μαθηματικής σκέψης.

Λ ΥΜΕΝΕΣ ΑΣΚΗΣΕΙΣ

1) Να βρείτε τους θετικούς ακέραιους α,β,γ,δ με α < 5, β < 3, γ � 3 και α + 5β + 15γ + 60δ = 1 18 .

Λύση Είναι 1 1 8 = 5(β + 3γ + 12δ) + α, 0 < α < 5 και 1 1 8 = 5 · 23 + 3 , άρα λόγω της μοναδικότητας στην ευκλείδεια διαίρεση του πηλίκου και υπολοίπου έ­χουμε α = 3 και β + 3γ + 1 2δ = 23 . Ομοίως 23 = 3( γ + 4δ) + β, Ο < β < 3 και 23 = 3 · 7 + 2 , άρα . . . β = 2 και γ+4δ = 7. Είναι 7 = 4δ + γ, Ο < γ � 3 και 7 = 4 · 1 + 3 , άρα . . . γ = 3 και δ = 1 . 2) Αν ο α είναι ακέραιος, να αποδείξετε ότι οι

αριθμοί

Α = α3 + 5α

3 ακέραιοι.

και Β = α(α2 + 6α - 13) 3

Απόδειξη

είναι

i) Τα δυνατά υπόλοιπα α με τον 3 είναι 0, 1 ,2. Ο ακέραιος α έχει μια από τις μορφές α = 3κ ή α = 3κ + 1 ή α = 3κ + 2, κ Ε Ζ . • Αν α = 3κ, τότε

Α (3κ)3 + 5 · 3κ = . . . = 9κ3 + 5κ = 3

= κ(9κ2 + 5) Ε Ζ

• Αν α = 3κ+ 1 , τότε α(α2 + 5) (3κ + 1) [(3κ + 1)2 + 5]

Α = 3 3 (3κ + 1)(9κ2 + 6κ + 6) = 3

= (3κ + 1)(3κ2 + 2κ + 2) Ε Ζ .

• Αν α = 3κ+2, τότε

Α α( α2 + 5) ( 3κ+2) . [( 3κ + 2) 2 + 5] 3 3

= . . . = ( 3κ + 2) ( 3κ2 + 4κ + 3) Ε Ζ

ii) Όπως i) ερώτημα ή παρατηρούμε ότι Β =

α3 + 6α2 - 1 3α α3 + 5α + 6α2 - 1 8α = 3 3

α3 + 5α 3(2α2 - 6α) i ) A (2 2 6 ) '7/ --- + = + α - α Ε !L.ι 3 3

3) Να αποδείξετε ότι το τετράγωνο ενός ακεραίου α παίρνει τη μορφή α 2 = 5κ ή α2 = 5κ ± l,κ Ε Ζ .

Απόδειξη

Είναι α = 5ρ + υ με υ = 0, 1 ,2 ,3 ,4, άρα

ΕΥΚΛΕΙΔΗΣ Β' λζ' τ.3/60

Page 63: Ευκλειδης Β 51

Μαθηματικά Β ' Λυκείου

α2 = (5ρ + υ)2 = 25ρ2 + 1 0ρυ + υ2

• Α ν υ = Ο, τότε από την ( 1 ) προκύπτει α2 = 25ρ2 = 5(5ρ2 ) = 5κ , με κ = 5ρ2•

• Αν υ = l , τότε α2 = 25ρ2 + 1 0ρ + l = = 5(5ρ2 + 2ρ) + 1 = 5κ + l , με κ = 5ρ2 + 2ρ .

• Α ν υ = 2, τότε α2 = 25ρ2 + 20ρ + 4 = 25ρ2 + 20ρ + 5 - 1 =

( 1 )

= s ( 5ρ2 + 4ρ + ι ) - 1 = 5κ - 1 ' με κ = 5ρ2 + 4ρ + 1 • Αν υ = 3 , τότε α2 = 25ρ2 + 30ρ + 9 = s ( 5ρ2 + 6ρ + 2) - 1 = 5κ - 1

με κ = 5ρ2 + 6ρ + 2 . • Αν υ =4, τότε α2 = 25ρ2 + 40ρ + 1 6 = s ( 5ρ2 + 8ρ + 3) + 1 = 5κ + 1 ' με κ = 5ρ2 + 8ρ + 3 .

4) Αν α Ε Ζ να αποδείξετε ότι 2 /(α4 + α) .

Απόδειξη Ο ακέραιος α έχει τη μορφή α = 2κ ή α = 2κ + 1, κ Ε Ζ .

• Αν α = 2κ, τότε α4 + α = (2κ)4 + 2κ = 2(8κ4 + κ), άρα 2 /(α4 + α) . • Αν α = 2κ+ 1 , τότε α4 + α = α(α3 + 1) = (2κ + 1) [ (2κ + 1)3 + 1] = . . . = = (2κ + 1)(8κ3 + 12κ2 + 6κ + 2) = = 2 · [(2κ + 1)(4κ3 + 6κ2 + 3κ + l)] άρα 2 /(α4 + α) .

5) Α ν α, β Ε Ζ και οι αριθμοί α+ 3 και 30-β

διαιρούνται με το 9, να αποδείξετε ότι και ο α+β διαιρείται με το 9.

Απόδειξη Έχουμε 9 /(α + 3) και 9 /(30 - β) , άρα υπάρχουν αντιστοίχως κ, λ Ε Ζ τέτοιοι, ώστε α + 3 = 9κ και 30 - β = 9λ � α = 9κ - 3 ( 1 ) και β = 30 - 9λ (2) . Προσθέτουμε τις ( 1 ) και (2) και έχουμε: α + β = 9κ - 3 + 30 - 9λ = 9(κ - λ + 3) , άρα 9 /(α + β) .

6) Να βρεθούν οι πιθανές θετικές τιμές του α, αν α, ν ακέραιοι με 3ν + 6 = πολα και 5ν + 7 = πολα (1).

Λύση Αναζητούμε τρόπο για να απαλείψουμε το ν. Από τις ( 1 ) έχουμε: α/(3ν + 6) και α /(5ν + 7) άρα α / 5(3ν + 6) και α/( -3)(5ν + 7) � α /(1 5ν + 30) και α /( - 1 5ν - 2 1) και συνεπώς

α /(1 5ν + 30 - 1 5ν - 2 1) ή α /9, με α Ε Ν . Επομένως α = 1 ή α = 3 ή α = 9. 7) Ν α αποδείξετε ότι για κάθε ν Ε Ν* ο

αριθμός ι ον + 3 . 4V+2 + 5 διαιρείται με το 9.

Απόδειξη

Θα αποδείξουμε τον ισχυρισμό Ρ(ν) με τη μέθοδο της μαθηματικής επαγωγής. Για ν = 1 είναι: 10 + 3 · 43 + 5 = 207 = 9 · 23 , άρα Ρ( 1 ) αληθής. Έστω ότι Ρ( ν) αληθής, δηλαδή ότι

( 1 ) Τότε θα αποδείξουμε ότι και Ρ( ν+ 1 ) αληθής δηλ. ότι l Ον+Ι + 3 · 4ν+3 + 5 = 9λ, λ Ε Ζ . Από την ( 1 ) προκύπτει l Ov = 9κ - 48 · 4v - 5 (2)

(2) Είναι ι ον+l + 3 · 4ν+3 + 5 = l O · l OV + 3 · 43 · 4ν + 5 = = 1 0(9κ - 48 · 4ν - 5) + 1 92 · 4ν + 5 = = 90κ - 480 · 4ν - 50 + 1 92 · 4ν + 5 = = 9(κ - 32 · 4ν - 5) = 9λ , με λ = κ - 32 · 4ν - 5 .

8) Να βρείτε το υπόλοιπο της διαίρεσης του αριθμού Α = 291950 + 572004 + 4 με τον αριθμό 7.

Απόδειξη

Οι διαιρέσεις του 29 και 57 με το 7 μας δίνουν αντιστοίχως 29 = 4 · 7 + 1 και 57 = 8 · 7 + 1 , οπότε σύμφωνα με το α) ερώτημα έχουμε: Α = (4 · 7 + 1)Ι95ο + (8 . 7 + 1)2οο4 + 4 = = πολ( 4 · 7) + 11950 + πολ(8 · 7) + 12004 + 4 = = πολ7 + 1 + πολ7 + 1 + 4 = πολ7 + 6 , οπότε υ = 6.

9) Να αποδείξετε ότι ο αριθμός 4 δε διαιρεί τον α2 + 5,α Ε Ζ .

Απόδειξη Έστω ότι 4 /( α2 + 5) , τότε υπάρχει κ Ε Ζ τέτοιος, ώστε α2 + 5 = 4κ ( 1 ) . • Αν α = 2λ, λ Ε Ζ , τότε η

ΕΥΚΛΕΙΔΗΣ Β' λζ' τ.3/61

Page 64: Ευκλειδης Β 51

Μαθηματικά Β ' Λυκείου

(1) {::> (2λ)2 + 5 = 4κ {::>

2 2 5 {::> 5 = 4( κ - λ ) {::> κ - λ =- , άτοπο, αφού

5 (κ - λ2 ) ε Ζ και - � Ζ . 4 • Α ν α = 2λ + 1, λ ε Ζ τότε

4

(1) {::> (2λ + 1)2 + 5 = 4κ {::> 4λ2 + 4λ + 1 = 4κ - 5 {::> 4(λ2 + λ - κ) = -6 {::> 2(� + λ - κ) = -3 , άτοπο (άρτιος = περιπός), επομένως 4 1 (α2 + 5) .

ΚΑΙ ΤΩΡ Α Η ΣΕΙΡ Α ΣΑΣ .. . .

1) Από τις παρακάτω ισότητες: i) 59 = 7 . 8 + 3 ii) 63 = 8 . 7 + 7 iii) -45 = -4 · 1 0 - 5 iν) 2004 = 39 · 50 + 54 ποιες είναι ισότητες ευκλείδειας διαίρεσης και γιατί;

2) Στη διαίρεση του 280 με τον θετικό ακέραιο κ το πηλίκο είναι ίσο με 1 1 . Να βρείτε τις τιμές του κ καθώς και το υπόλοιπο της διαίρεσης σε κάθε περίπτωση.

3) Να βρείτε τη μορφή που πρέπει να έχει ο , , , θ , α3 + 2α + 1 ακεραιος α, ετσι ωστε ο αρι μος 4 να

είναι ακέραιος. 4) Αν α, β, γ, δ, κ ε Ζ και ο κ διαιρεί τους αριθ­

μούς αβ- 1 και αδ+γ, να αποδείξετε ότι διαιρεί και τον αριθμό βγ+δ.

5) Αν α + 3 = πολ2 και (α - 3) / 1 0 να βρεθεί ο ακέραιος θετικός αριθμός α.

6) Να αποδείξετε ότι για κάθε ν ε Ν• ο ν3 + 2ν διαιρείται με το 3 .

Κωνικές τομές

Ι . Οι κωνικές τομές :

παραβολή, έλλειψη, υπερβολή ως γεωμετρικός τόπος Κέντρου Κύκλου.

1 . ΠΑΡΆΒΟΛΗ

Είναι ο γεωμετρικός τόπος των κέντρων Κ των κύκλων οι οποίοι διέρχονται από σταθερό σημείο Ε και εφάπτονται σταθερής ευθείας δ. (σχήμα 1).

χ

Σχήμα 1 Παράδειγμα: Ο γεωμετρικός τόπος των κέντρων των κύκλων που διέρχονται από το σημείο Ε(2,0) και εφάπτο­νται στην ευθεία: δ : χ = -2 είναι παραβολή με ε­ξίσωση C : y2

= 8χ .

Γ. Τσικαλουδάκης

Πρακτική εύρεση σημείων της παραβολής με εστία Ε και διευθετούσα δ.

Υ

χ

Σχήμα 2 Θεωρούμε τυχαίο σημείο Β της δ. Το ΒΕ προφα­νώς διχοτομείται από τον άξονα y'y. Φέρνουμε τη μεσοκάθετη του ΒΕ η οποία τέμνει, την κάθετη στο Β της δ, στο Μ το οποίο προφανώς είναι ση­μείο της παραβολής αφού ΜΕ = ΜΒ.

2. ΕΛΛΕΙΨΗ

Είναι ο γεωμετρικός τόπος των κέντρων Κ των κύκλων οι οποίοι εφάπτονται εσωτερικά του κύκλου C(E, 2α) και διέρχονται από σταθερό σημείο Ε ', εσωτερικό του C. (σχήμα 3) με ΕΈ < 2α .

ΕΥΚΛΕΙΔΗΣ Β' λζ' τ.3/62

Page 65: Ευκλειδης Β 51

Μαθηματικά Β ' Λυκείου

Σχήμα 3 Πραγματικά: Αν ο κύκλος (Κ,ΚΕ') εφάπτεται στον (Ε, 2α) στο Β, θα έχουμε: ΚΒ = ΚΕ' , οπότε: ΚΕ + ΚΕ' = ΚΕ + ΚΒ = ΕΒ = 2α ( 1 ) Πρακτικά για να βρούμε ένα σημείο της έλλειψης θεωρούμε σημείο Β του κύκλου (Ε, 2α) φέρνουμε τη μεσοκάθετη του ΒΕ ' η οποία τέμνει το ΒΕ στο Κ που είναι σημείο της έλλειψης. (λόγω των ισο­τήτων ( 1 )) .

ΠΑΡΆΔΕΙΓΜΑ Ο γεωμετρικός τόπος των κέντρων των κύκλων που διέρχονται από το σημείο Ε'(-3, 0) και εφά-πτονται εσωτερικά του κύκλου με κέντρο Ε(3 ,0)

2 2 και ακτίνα ρ = 5 είναι η έλλειψη : C : .;- + Υ 2 = 1 .

5 4

3. ΥΠΕΡΒΟΛΉ Ο γεωμετρικός τόπος των κέντρων των κύκλων που διέρχονται από σταθερό σημείο Ε και εφάπτονται εξωτερικά του κύκλου C(E', 2α) , με

ΕΈ > 2α , είναι κλάδος υπερβολής (σχήμα 3).

Πραγματικά, αν ο κύκλος (Κ, ΚΕ) εφάπτεται εξω­τερικά στον (Ε ' , 2α) στο σημείο Α έχουμε:

ΚΕ' - ΚΕ = ΚΕ' - ΚΑ = Ε' Α = 2α Πρακτικά για να βρούμε ένα σημείο του (δεξιού) κλάδου της υπερβολής με εστίες Ε' , Ε, θεωρούμε

τυχαίο σημείο Α του κύκλου (Ε ' , 2α), φέρνουμε τη μεσοκάθετη του ΕΑ που τέμνει την Ε' Α στο Κ, το οποίο είναι σημείο της υπερβολής.

ΣΗΜΕΙΩΣΗ: Τα κέντρα των κύκλων που διέρχονται από το Ε και ο κύκλος C(Ε' , 2α) εφάπτεται εσωτερικά σ' αυτούς, σχηματίζουν τον άλλο (αριστερό) κλάδο της παραπάνω υπερβολής.

11. ΑΣΚΗΣΕΙΣ

1 . Ν α αποδείξετε ότι η εξίσωση : χ3 + y2 - χ2 + xy2 - Sx - 3 = 0 (1).

Παριστάνει ένα κύκλο και μια ευθεία εφα­πτομένη σ' αυτόν.

Λύση

Έχουμε: χ3 + y2 - χ2 + xy2 - 5χ - 3 = 0 {::} χ(χ2 + y2 ) + y2 - χ2 - 5χ - 3 = ο {::} χ(χ2 + y2 ) + χ2 + y2 - (2χ2 + 5χ + 3) = ο {::} (χ2 + y2 )(x + 1) - (χ + 1)(2χ + 3) = Ο {::} (χ + 1)(χ2 + y2 - 2χ - 3) = 0 {::} ( ε1 ) : χ = - 1 ή C : χ 2 + y2 - 2χ - 3 η (C) είναι εξίσωση κύκλου με κέντρο Κ( 1 ,0) και ακτίνα ρ = 2 . Ακόμα είναι d(Κ, ε1 ) = j- 1 - 1 j = 2 , οπότε η (ει ) εφάπτεται στον C.

2 . Δίνεται η εξίσωση: C : χ2

+ y2 - 2χ - 2y + α2 + 2α = Ο, α Ε JR .

i) Να βρεθεί για ποιες τιμές του α Ε JR η (C) είναι εξίσωση κύκλου.

ii) Να βρεθεί ο γεωμετρικός τόπος των ση­μείων από τα οποία δεν διέρχεται κανέ­νας από τους παραπάνω κύκλους C.

Λύση

i) Η (C) είναι εξίσωση κύκλου όταν: 8 - 4α2 - 8α > 0 δηλαδή όταν: α Ε ( -J3 - 1, J3 - 1 ) .

ii) Η (C) θεωρούμενη ως δευτεροβάθμια εξίσωση ως προς α έχει διακρίνουσα:

Δ = 4 - 4(χ2 + y2 - 2x - 2y) . ΕΥΚΛΕΙΔΗΣ Β ' λζ' τ.3/63

Page 66: Ευκλειδης Β 51

Μαθηματικά Β' Λυκείου

Επομένως από το σημείο M(x0 , y0 ) δεν διέρχεται κανένας από τους κύκλους C, όταν Δ<Ο δηλαδή όταν:

χ� + Υ� - 2χ0 - 2y0 - 1 > Ο ( 1 ) .

Η εξίσωση : C0 : χ2 + y2 - 2x - 2y - 1 = Ο είναι εξίσωση κύκλου με κέντρο Κ( 1 , 1 ) και ακτίνα ρ = J3 . Επομένως από κάθε εξωτερικό σημείο του κυκλικού δίσκου που ορίζει η Co, δεν διέρχεται κανένας από τους κύκλους C.

3 . Έστω C ο κύκλος ο οποίος διέρχεται από το

σημείο Ε ( �,0} p>O και εφάπτεται στον

άξονα y'y. Να βρεθεί ο γεωμετρικός τόπος τον αντιδιαμετρικού σημείου του Ε.

Λύση Υ

χ

δ

Αν πάρουμε την ευθεία δ : χ = _ Ε. και το κάθετο 2

τμήμα ΕΉ στην δ που τέμνει τον y'y στο Β, έχο� με: ΗΒ = Ε. και στο τραπέζιο ΟΕΕ'Β η ΑΚ είναι

2 διάμεσος, οπότε: 2ΚΑ = Ε'Β + ΟΒ, Ε'Β = 2ΚΑ - Ε. και συνεπώς Ε'Η = 2ΚΑ = Ε'Ε .

2 Δηλαδή το Ε ' ισαπέχει από το Ε και την ευθεία δ, οπότε ο γεωμετρικός τόπος του Ε' είναι η παραβο­λή με εστία το Ε και διευθετούσα τη δ.

ΑΣΚΗΣΗ Να βρεθεί ο γεωμετρικός τόπος των κέντρων των κύκλων οι οποίοι εφάπτονται στον άξονα y'y και στον κύκλο: C : (χ - 2)2 + y2 = 4 .

4. Ευθύγραμμο τμήμα ΑΒ σταθερού μήκους έχει το άκρο Α στον άξονα χ' χ και το Β στον y 'y. Αν για ένα σημείο Μ του ΑΒ

ισχύει ΜΑ = β και ΜΒ = α. Να βρεθεί ο γεωμετρικός τόπος που διαγράφει το Μ.

Λύση

Από το Μ φέρνουμε στους άξονες χ'χ, y'y κάθετα τμήματα ΜΚ, ΜΛ αντίστοιχα. Από τα όμοια τρίγωνα:

Υ Β

Α χ

y' ΑΟΒ, ΜΚΑ, ΜΛΒ έχουμε:

ΛΜ α ΚΜ β -- =-- και -- = --ΟΑ α + β ΟΒ α + β ( 1 )*

οπότε αν M(x,y) είναι οι συντεταγμένες του Μ,

έχουμε: Η_ _ α I YI β και - = ΟΑ α + β ΟΒ α + β χ2 ΟΑ2 y2 ΟΒ2 α2 (α + β)2

και β2 (α + β)2

, χ2 y2 ΟΑ2 + 0Β2 ΑΒ2 οποτε: - +-= = = 1

α2 β2 (α + β)2 (α + β)2

' χ2 y2 Άρα: -2 + -τ = 1 .

α β

ή

Επομένως ο γεωμετρικός τόπος του Μ είναι έλλει­ψη, αν α += β και κύκλος όταν α = β. ΣΗΜΕΙΩΣΗ: Οι ( 1 ) ισχύουν και στις οριακές θέσεις του ΑΒ, που το Α ή το Β συμπίπτει με το Ο.

ΜΑ 3 ΕΦΑΡΜΟΓΗ: ΑΒ = 10 και - = ­ΜΒ 4

5. Δίνονται οι ευθείες:

ει : y = ! χ + β, εz : y = -!χ - β α α

και το σημείο Κ( α, Ο), α>Ο, β>Ο. Ευθεία δ διέρχεται από το Κ και τέμνει τις ει, ε2 στα σημεία Α, Β αντίστοιχα. Να βρεθεί ο γεωμετρικός τόπος των μέσων Μ των τμημάτων ΑΒ.

Λύση Έστω A(x1 , y1 ), B(x2 , y2 ) και M(x0 , y0 ) , το μέ-σο του ΑΒ. Είναι:

ΕΥΚΛΕΙΔΗΣ Β' λζ' τ.3/64

Page 67: Ευκλειδης Β 51

Μαθηματικά Β ' Λυκείου

β β Υι = -χι + β, Yz = --Xz - β , α α 2χσ = χι + Χz , 2yo = Yι + Yz ,

Υι - y2 = � 0 2χ0 + 2β και α

χ'

Ακόμα επειδή τα Α, Μ, Κ, Β είναι συνευθειακά έ­χουμε: det ( ΚΜ,ΑΒ) = Ο , οπότε:

(Υι - ΥzΧχσ - α) - Υο (χι - χ2 ) = 0 (2)0 Η (2) λόγω των ( 1 ) γίνεται: (� ο 2χ0 + 2β) (χ0 - α) - 2y� ο � = 0 {::}

2β 2 fi fi 2α 2 -χ0 + 2yx0 - 2yx0 - 2αβ --y0 = 0 {::} α β

χ2 y2 β2χ� - α2y� = α2β2 {::} α� -

β� = 1 0 (C)

Συνεπώς ο γεωμετρικός τόπος του Μ είναι η υπερ,­βολή (C) o

7. Δίνονται οι κύκλοι:

C" : χ2 + y2 + Ακχ + Βκy - 25 = 0,

Έστω ότι οι κύκλοι Cκ έχουν με τον κύ­

κλο: C : χ2 + y2 = 5 κοινό σημείο το M(l, 2).

ί) Να αποδειχθεί ότι τα κέντρα των Cκ εί-ναι συνευθειακά σημεία.

ίί) Να αποδείξετε ότι μόνο ένας κύκλος από τους Cκ εφάπτεται στον C.

Λύση

ί) Οι κύκλοι Cκ διέρχονται από το Μ(1 , 2), οπότε θα είναι: Ακ + 2Βκ - 20 = Ο

ο C , , κ ( Ακ Βκ ) κ εχει κεντρο -2,-2 ο

( 1 )

Η ( 1 ) γράφεται - Ακ - 2 Βκ + 1 0 = Ο 2 2 (2)

Από τη (2) προκύπτει ότι οι συντεταγμένες των σημείων ( Α κ Β κ ) , θ �τ* Κ -- -- για κα ε κ Ε 1'1

2 ' 2 '

Επαληθεύουν την εξίσωση : ε: χ + 2y + 1 0 = Ο

Επομένως τα σημεία κ (-�κ ,-�κ ) είναι συ-νευθειακάο

ii) Ο Cκ εφάπτεται στον C, αν και μόνο αν �Α; + Β; + 1 00 ± JS = loκl 2

ισοδύναμα: �A; + B� + l00 ± 2J5 = �A� + B� (3)

Θέτουμε Α� + Β� = ρ , οπότε η (3) γίνεται: Jρ + lOO ± 2J5 = JP (4)

Εύκολα διαπιστώνουμε ότι η (4) είναι ισοδύναμη με Jρ + 1 00 - 2J5 = JP <:::> 20 = J5P <:::> ρ = 80 Επομένως, λόγω και της ( 1 ) έχουμε το σύστημα:

κ 1C κ 1C Α2 + Β2 = 80 } Α = 20 - 2Β } Ακ + 2Βκ = 20

<=>Β� - 1 6Βκ + 64 = 0

<=>

<:::> Β κ = 8 και Ακ = 4 Συνεπώς ο κύκλος:

CK : χ2 + y2 + 4χ + 8y - 25 = ο

εφάπτεται (εσωτερικά) με τον Co

111. ΟΙ ΤΥΠΟΙ VIET Α ΣΤΙΣ ΚΩΝΙΚΕΣ ΤΟΜΕΣ

6. Δίνεται ο κύκλος C : χ2 + y2 = 10 και η ευθεία ε : y = λχ - 5 . Να προσδιοριστεί ο λ ε JR. ώστε η (ε) να τέ­μνει τον (C) σε δύο σημεία Α, Β έτσι ώστε:

ΑΟΒ = 90° . Λύση

Έστω A(x1 , y1 ) και B(x2 , y2 ) o Οι τετμημένες χ 1 , χ2 των Α, Β (κοινών σημείων των ε, C) επαληθεύουν την εξίσωση:

(λ2 + 1)χ2 - 1 0λχ + 1 5 = 0 ι ! Ι ΕΥΚΛΕΙΔΗΣ Β' λζ' τ.3/65

Page 68: Ευκλειδης Β 51

Μαθηματικά Β' Λυκείου

(που προκύπτει από το σύστημα των (ε), (C)). Υ

χ

Ε . θ . 1 5 πομενως α ειναι: Χι χ2 = -2 - , λ + 1 10λ Χι + xz =-2 -, Υι = λχι - 5, Υ2 = λχ2 - 5 (2) λ + 1

Λ __, __, οπότε έχουμε: ΑΟΒ = 90° {::} ΟΑ l_ ΟΒ {::} (2) ΧιΧz + ΥιΥ2 = 0{:}χιχ2 (λχι - 5)(λχ2 - 5) = 0 (2) {:} (λ2 + 1)χι · χ2 - 5λ(χι + χ2 ) + 25{:}

50λ2 1 5 --2 -+ 25 = ο {::} λ 2 = 4 {::} λ = ±2 . λ + 1 7. Δίνεται ο κύκλος Cλ και η ευθεία ελ με:

Cλ : χ2 + y 2 + λχ - 1 = Ο και ε λ : y = λχ

λ Ε JR* . i) Δείξτε ότι ο Cλ και η ελ έχουν δύο κοινά

σημεία για κάθε λ Ε JR* . ii) Δείξτε ότι το μέσο Μ της χορδής που ο

Cλ ορίζει επί της ελ ανήκει σε σταθερό κύκλο.

Λύση

i) Τα κοινά σημεία των ελ, Cλ βρίσκονται από τη λύση της εξίσωσης,

(λ2 + 1)χ2 + λχ - 1 = 0 ( 1 )

Συνεπώς είναι: 4χ2 = λ2 2 1 και λ = - 1 --ο (λ2 + 1)2 2yo

- 1 - -1-οπότε: 4χ� = ---2..:...Υ-"-ο 1 ισοδύναμα:

4y� 8x�y0 + 4y� + 8y� = 0 {::} 8yo (χ� + Υ� -±Υ ο ) = Ο {::} (λ 7: Ο)

2 2 1 Co : Χ ο + Υ ο -- Υ ο = Ο · 2 Η C0 είναι εξίσωση κύκλου με κέντρο Κ (Ο,�)

. 1 . . Μ και ακτινα ρ = - , στον οποιο ανηκει το . 4 8. Δίνεται η παραβολή : (C) και η ευθεία (ε) με

εξισώσεις: ι

C : x2 = -2--y και ε : y = λχ + l, λ ε !R . λ + 1

Δείξτε ότι: i) η (ε) και η (C) έχουν δύο κοινά σημεία

Α,Β για κάθε λ Ε iR •

ii) το μέσο Μ του ΑΒ ανήκει σε σταθερό κυκλο.

Λύση ί) Από την λύση του συστήματος των εξισώσεων

των ε, C έχουμε: (λ2 + 1)χ2 - λχ - 1 = 0 ( 1 )

Η ( 1 ) έχει δια κρίνουσα Δ = 8λ 2 + 4 > Ο , οπότε έχει δύο άνισες ρίζες χι , Xz και συνεπώς η (ε) και η (C) τέμνονται σε δύο σημεία Α(χι , Υι ) και B(x2 , y2 ) για κάθε λ Ε !R .

η ( 1 ) έχει διακρίνουσα Δ = 5λ 2 + 4 > Ο και ii) Έστω Μ(χ0 , y0 ) με το μέσο του ΑΒ. Είναι: συνεπώς ο Cλ και η ευθεία ελ έχουν δύο κοινά σημεία για κάθε τιμή του λ Ε JR .

ii) Αν Α(χι , Υι ), B(x2 , y2 ) είναι τα σημεία τομής των Cλ, ελ και M(x0 , y0 ) είναι το μέσο του ΑΒ . 2 λ εχουμε: χ0 = Χι + χ2 = --2 -, λ + 1

1 2yo = Υι + Υ2 = --2- · λ + 1

λ 2χ0 = Χι + χ2 =-2- και λ + 1 λ2 2yo = Υι + Υ2 = λ(χι + χ2 ) + 2 =-2-+ 2 . λ + 1

Άρα είναι: 2χ0 =+- και 2y0 =---Ρ----- + 2 λ + 1 λ + 1 οπότε:

ΕΥΚΛΕΙΔΗΣ Β' λζ' τ.3/66

Page 69: Ευκλειδης Β 51

Μαθηματικά Β ' Λυκείου

2 λ2 2 2 - 2y (1) 4χ0 = 2 2 και λ = 0 (2) (λ + 1) 2y0 - 3

Η ( 1 ) λόγω της (2) γίνεται: 4χ� = (2 - 2y0 )(2y0 - 3) ή 4χ� + 4y� - 1 0y0 + 6 = 0 ή

2 2 5 3 Cxo + Υο - -yo + - = 0 2 2 (3)

Η (3) είναι εξίσωση κύκλου με κέντρο Κ (i _i) και ακτίνα ρ = J34

5 ' 4 4 .

Συνεπώς το Μ( χ0 , y0 ) ανήκει στον κύκλο C.

Η Η Η

Αγαπητέ Ευκλείδη

Ονομάζομαι Γεώργιος Σούμπλης και είμαι μα­θητής της Α' τάξης του Λυκείου στο Αρσάκειο Ψυχικού. Μου αρέσουν τα μαθηματικά και προμη­θεύομαι τακτικά το περιοδικό «Ευκλείδης Β '» . Το περιοδικό αυτό με βοηθάει στην κατανόηση των μαθηματικών και οφείλω ένα μεγάλο ευχαριστώ σε όσους συντελούν στην έκδοσή του.

Σας στέλνω δύο ασκήσεις, μια Άλγεβρας και μια Γεωμετρίας. Νομίζω ότι είναι καλές και ελπίζω να τις δημοσιεύσετε.

Άσκηση Άλγεβρας Μία ομάδα αποτελείται από πέντε παιδιά, τα οποία είναι τουλάχιστον, ενός έτους το καθένα (την ηλι­κία τους την υπολογίζουμε . σε ακέραια έτη). Σε

καθένα από τα παιδιά αυτά δίνουμε τόσα ευρώ όση είναι η ηλικία του. Έτσι δώσαμε 14 ευρώ. Να δεί­ξετε ότι τουλάχιστον δύο από τα παιδιά αυτά έχουν την ίδια ηλικία.

Άσκηση Γεωμετρίας Στο εσωτερικό ενός τετραγώνου ΑΒΓ Δ παίρνουμε τρία σημεία Ε, Ζ και Η έτσι, ώστε το τρίγωνο ΕΑΒ να είναι ισοσκελές με κορυφή το Ε, το τρίγωνο ΖΑΔ να είναι ισοσκελές με κορυφή το Ζ και τα τρίγωνα ΑΕΖ και ΕΖΗ να είναι ισόπλευρα. Η ευ­θεία ΑΕ τέμνει την πλευρά ΒΓ στο σημείο Θ και η ευθεία ΑΖ τέμνει την πλευρά ΔΓ στο Ι. Ν α δείξετε ότι: 1 ) ΔΖ = ΓΗ 2) Τα σημεία I, Η και Θ είναι σ�r νευθειακά. 3) Τα σημεία Α, Η και Γ είναι συνευ­θειακά.

: · · · · · · · · · · · · · · · · · · · · · · · · · · · · · · · · · · · · · · · · · · · · · · · · · · · · · · · · · · · · · · · · · · · · · · · · · · · · � • • • • • • • •

Κ α λ ό Π ά σ χ α • • • • • • • • • • • • • • • • • • • • • • • • • • • • • • • • • • • • • • • • • • • • • • • • • • • • • • • • • • • • • • • • • • • • • • • • • • • • • • • • • • • • • • • • •

ΕΥΚΛΕΙΔΗΣ Β' λζ' τ.3/67

Page 70: Ευκλειδης Β 51

υ

rJ � Αnό6cιfη Ανιιιοτικών Εχcr.ιcων και

Εnfλuιιη Ανιιιώιιeων

των Ν. Ζανταρίδη - Κ. Παπαδόπουλο

Η απόδειξη ανισοτικών σχέσεων στην Ανάλυση παρουσιάζει ιδιαίτερο ενδιαφέρον, ενώ δημιουργεί α�

κετές δυσκολίες στους μαθητές. Στο άρθρο αυτό ασχολούμαστε με την απόδειξη ανισοτικών σχέσεων και

την επίλυση ανισώσεων με τη βοήθεια θεωρημάτων που αναφέρονται: 1) στη μονοτονία παραγωγίσιμων

συναρτήσεων, 2) στα ακρότατα παραγωγίσιμων συναρτήσεων. Ακόμη αποδεικνύουμε ανισότητες με τη χρή­

ση του Θεωρήματος Μέσης Τιμής του Διαφορικού Λο ισμού.

Έστω η παραγωγίσιμη συνάρτηση Α : (Ο, +οο) � JR . Αν η συνάρτηση g(x) = x · f'(x),

χ ε (Ο, +οο) είναι γνησίως αύξουσα στο (Ο, +οο) να

δείξετε ότι για κάθε χ>Ο ισχύει

f(x) + r(�J � 2f(l) .

Λύση

Θεωρούμε τη συνάρτηση h(x)-f(x) + f(�}x > O . Είναι h'(x) - f'(x) + f'(�}(�J' = f'(x) -:2 f'(x) Βρίσκουμε το πρόσημο της h'(x) . 'Εχουμε h'(x) > O <::> f'(x) - :2 f'(�J > O<=> <=>ftx) > :2 f'(�J�xrtx)>� · f'(�J<=>g(x)>g(�J (αφού g(x) = χ · f'(x) ) <=> χ >_!_ (αφού η g είναι χ γνησίως αύξουσα στο R) <::::> χ2 > 1 <=> χ > 1 (αφού χ ε (Ο, +οο) .

χ ο 1 +οο h'(x) - ο + h(x) "-.. ulin/

Από το πρόσημο της h'(x) που φαίνεται στον πί­νακα προκύπτει ότι η h είναι γνησίως φθίνουσα

στο (0, 1 ] και γνησίως αύξουσα στο [1, +οο) , οπότε η h παρουσιάζει στο Χο = 1 (ολικό) ελάχιστο το ο­ποίο είναι hmin = h(l) = f(1) + r( Τ J = 2f(l) . Άρα ι-

σχύει h(x) � hmin = 2f(l) ή f(x) + f(�J � 2f(l) για κάθε χ>Ο. Άσκηση 2η :

Α. Ν α δείξετε ότι για κάθε χ ε JR • ισχύει

x.enx � χ - 1 .

Β. Α ν α, β, γ ε JR • και α+β+γ = 3 να δείξετε ότι

αα · ββ · γΎ � � . Λύση

Α. Θεωρούμε την συνάρτηση f (χ) = x.enx - χ + 1, χ > Ο .

Είναι f'(x) = . . . . = .enx . Βρίσκουμε το πρόσημο της f'(x) . Έχουμε:

(enxf) f'(x) > Ο <=> .enx > Ο<=> .enx > fn1. <=> χ > 1 . χ ο 1 +οο

f(x) - ο + f(x) "-.. nlin/

Από το πρόσημο της f'(x) που φαίνεται στον πίνακα προκυπτει ότι η f είναι γνησίως φθίνουσα στο (0, 1 ] και γνησίως αύξουσα στο

ΕΥΚΛΕΙΔΗΣ Β' λστ' τ.l/68

Page 71: Ευκλειδης Β 51

Μαθηματικά Γ Λυκείου

[1, +οο) , όπότε η f παρουσιάζει στο Χο = 1 (ολικό) ελάχιστο το οποίο είναι το ζ;" = f(l) = 1 · Rn1 - 1 + 1 = Ο . Επειδή το (ολικό) ελάχιστο της f είναι το ζ;" = Ο έπεται ότι για κάθε χ>Ο ισχύει f(x) � ζ;" = Ο � � xt'nx - x + 1 � 0 � xfnx � x - 1 : ( 1 ) .

Β. Έστω α, β, γ Ε �· με α+β+γ = 3 (2). Λόγω της ( 1 ) ισχύουν: αt'ηα � α - 1 (3) �� � β - 1 �) γt'ηβ � γ - 1 (5) Από τις (3), (4), (5) με πρόσθεση κατά μέλη έχουμε: αt'ηα + βt'ηβ + γ Rηγ � (α - 1) + (β - 1) + (γ - 1) � t'η(αα ) + Rη(ββ ) + t'η(γ1 ) � (α + β + γ) - 3 (2) �Rη(αα · ββ · γ1 ) � 3 - 3 = 0 � � t'η(αα · ββ · γ1 ) � t'η1 � α α . ββ . γΎ � 1 .

Άσκηση 3η : Α. Ν α μελετηθεί ως προς την μονοτονία η

συνάρτηση f(x) =� , χ > Ο .

ι ι

Β. Να συγκριθούν οι αριθμοί: ιοJϊΟ , 1 1Jil .

Λύση

Α. Είναι

f'(ψ ( � )' = ( fnx)' . � ;.;:xJ{ Fx}' =

..Γχ Rnx --;: -Νχ 2 - Rnx 'Ε = = Γ . χουμε

χ 2χνχ 2 - Rnx f'(x) > O � ..Γχ > 0 � 2 - Rnx > O � 2χ χ

(enxt) � Rnx < 2 � Rnx < Rn( e2 ) � Ο < χ < e2

χ ο e2 +α: f(x) + ο -f(x) ? ι �

Από το πρόσημο της f'(x) , που φαίνεται στον πίνακα προκύπτει ότι η f είναι γνησίως αύξουσα στο (O, e2 ] και γνησίως φθίνουσα στο [e2 , +οο) .

Β. Επειδή είναι e2 < 1 Ο < 1 1 και η f είναι γνησiως φθίνουσα στο [ e2 , +οο) έπεται ότι ισχύει

f(I O) > f(1 1) � Rnl O > Rn1 1 � Jlδ .fιl

� -1- · Rnl O > -1- · Rn1 1

JIO m

� Rn IOJiO > Rn 1 1JU � 10Jί0 > 1 1Jί1 . ( I ) ( I ) (fnx1) I I

Άσκηση 4η : Για την παραγωγίσιμη συνάρτηση

2 f : � � � ισχύει f'(x) = ex για κάθε Χ Ε � . α+Ρ Να δείξετε ότι J0xf(t)dt + J:+P-x f(t)dt � 2fσ2 f(t)dt

για κάθε χ Ε � •

Λύση

Θεωρούμε τη συνάρτηση g(x) = fσχ f(t)dt + fσα+β-χ f(t)dt Χ Ε � .

Είναι g'(x) = (Jox f(t)dt) + (Joα+β-x f(t)dt) = = f(x) + f(α+ β - χ) · (α+ β - χ)' = f(x) - (α+ β- χ) . Επειδή f'(x) = ex' > ο για κάθε χ Ε � ,έπεται ότι η f είναι γνησίως αύξουσα στο R. Βρίσουμε το πρό­σημο της g'(x) . Έχουμε g'(x) > O � f(x) - f(α + β - x) > O � � f(x) > f(α + β - χ) � χ > α + β - χ ( αφού η f είναι γνησίως αύξουσα στο R)

α + β � 2χ > α + β � χ > -- . 2 α+β

χ -α:J -2- +α: g'(x) - ο + g(x) "-. I ?

mιn

Από το πρόσημο της g' (χ) που φαίνεται στον πί-, ( α + β ] , . νακα προκυπτει στο -οο,-2- και γνησιως αυ-

ξουσα στο [α ; β , +οο) οπότε η g παρουσιάζει στο α + β λ ' λ ' ' ' Χ0 = -2

- ο ικο ε αχιστο, το οποιο ειναι το

ΕΥΚΛΕΙΔΗΣ Β' λζ' τ.3/69

Page 72: Ευκλειδης Β 51

Μαθηματικά Γ Λυκείου

( α + β ) α+β β α+β gmin = g -2- = fo2 f(t)dt + fσα+ --2 f(t2 )dt =

α+β = 2 fo2 f(t)dt . Επομένως για κάθε χ Ε JR ισχύει ( α + β ) α+β g(x) � gmin = 2 -2- = fo2 f(t)dt <=>

α+β <=? fox f(t)dt + fσα+β-χ f(t)dt � fo2 f(t)dt .

Άσκηση Sη :

Να λυθεί η aνίσωση χ2 - χ < fn( χ4 + 1 ) για χ2 + 1

κάθε Χ Ε JR . Λύση

' 2 ( χ4 + 1 ) Ε χουμε χ - χ < fn -2- <=>

χ + 1 <=> χ2 - χ < fn(x4 + 1) - fn(x2 + 1) <=> <=> χ2 - fn [ (χ2 )2 + 1 J < χ - t'n(x2 + 1) <=> <=> f(x2 ) < f(x) όπου f(x) = χ - fn(x2 + 1) ,

χ Ε 1R . Ει'vαι f'( ) ( χ - 1) 2 'Ε ' θ χ = 2 • τσι για κα ε χ + 1 χ Ε JR - {1} είναι f'(x) > Ο και f'(l) = Ο , οπότε η f είναι γνησίως αύξουσα στο R. Έτσι έχουμε f(x2 ) < f(x) <=> χ2 < χ <=> χ2 - χ < Ο <=> <=? Χ( Χ - 1) < 0 <=? Χ Ε (0, 1)

ο 1 + l +

Άσκηση 6'1 :

Ν α συγκριθούν οι αριθμοί α, β Ε ( e, +οο) αν είναι

γνωστό ότι αΡ > βα .

Λύση

αβ > βα <=> fη(αβ ) > fη(βα )

<=> βfηα > αfηβ <=> fnα > fnβ <=> f(α) > f(β) α β

, f( ) fnx οπου χ = -χ

Θεωρούμε τη συνάρτηση f(x) = fnx όπου x>e. χ Ε, f'( ) 1 - fnx

0 , θ ιναι χ = 2 < για κα ε χ Ε ( e, +οο) , χ οπότε η f είναι γνησίως φθίνουσα στο ( e, +οο) .

Έχουμε fnxt

αβ > βα <=> fη(αβ ) > fη(βα ) <=> βt'ηα > αt'ηβ <=> t'nα t'nβ <=> - > - <=> f(α) > f(β) και αφού η f είναι α β

γνησίως φθίνουσα στο ( e, +οο) , είναι α< β.

Άσκηση 7η :

Έστω μία συνάρτηση f συνεχής στο [α,β] παραγωγίσιμη στο (α,β) και κυρτή στο [α,β] . Αν είναι f(α) < f(β) να δείξετε ότι: i) υπάρχει χ0 Ε (α, β) τέτοιο ώστε:

f(xo ) = f(α) + f(β)

2 ii) υπάρχει χ, Ε (α, χ0 ) και χ2 Ε (χ0 , β) τέτοια

ώστε: -1- + -1- = 2 · β - α και να

f'(x1 ) f'(x2 ) f(β) - f(α)

δείξετε ότι το Χο βρίσκεται πλησιέστερα στο β απ' ότι στο α.

Λύση i) Έχουμε f(α) = f(α) < f(β) τότε

2f(α) < f(α) + f(β) < 2f(β)

f( α) < f(β) = f(β) => f( α) < f( α) + f(β) < f(β) ( 1 ) . 2 Επειδή η f είναι συνεχής στο [α, β] και ισχύει η ( 1 ) έπεται ότι σύμφωνα με το θεώρημα Ενδιάμεσων τιμών, ότι υπάρχει Χο Ε (α, β)

τέτοιο ώστε: f(x0 ) = f(α) + f(β) . 2 ii) Επειδή η f είναι κυρτή στο [α, β] έπεται ότι η Γ

είναι συνεχής στο [α, β] και παραγωγίσιμη στο [α, χ0 ] , [χ0 , β] και παραγωγίσιμη στα διαστήματα (α, χ0 ), ( χ0 , β) . Άρα η f ικανοποιεί τις προϋποθέσεις του Θ.Μ.Τ. σε καθένα από τα διαστήματα [α, χ0 ] και [χ0 , β] , οπότε υπάρχουν Χ1 Ε (α, χ0 ) και χ2 Ε (χ0 , β) τέτοιοι ώστε:

f(α) + f(β) f(α) f'(χ ι ) = f(xo ) - f(α) = _ __,2!:....._ __ _

χ0 - α χ0 - α

= f(β) - f(α) και f'(x2 ) = f(β) - f(x0 )

2(χ0 - α) β - χ0 f(β) - f(α) + f(β)

___ ,_2 _ = f(β) - f(α)

ΕΥΚΛΕΙΔΗΣ Β' λζ' τ.3/70

Page 73: Ευκλειδης Β 51

Μαθηματικά Γ' Λυκείου

Έχουμε: α < χ, < Χ0 < Χ2 < β ::::::> α < χ1 < χ2 < β ::::> f'(x 1 ) < f'(x2 ) (αφού η f'

είναι γνησίως αύξουσα στο (α, β) τότε f(β) - f(α) f(β) - f(α) , 1 < 1 < η ---2(χσ - α) 2(β - χ0 ) 2(χ0 -α) 2(β-χ0 ) (αφού f(β) - f(α) > Ο) � 2(χ0 - α) > 2(β - χ0 ) (αφού 2(χ0 - α) > Ο και 2(β - χ0 ) > Ο )

α + β � χ0 - α > β - Χ0 ::::::> 2χ0 > α + β ::::::> χ0 > -- . 2 Ε δ , , α + β , , πει η ειναι χ0 > -2- επεται οτι το Χο

βρίσκεται πλησιέστερα στο β απ' ότι στο α.

Άσκηση 8η : Η συνάρτηση f : (0, +οο) � IR είναι παραγωγίσιμη και κυρτή στο (Ο, +οο) . Α ν η ευθεία ε : y = 3χ + 4 είναι aσύμπτωτη της Cr στο +οο τότε: 1) Να βρείτε το όριο: L = lim { f(x + l) - f(x)) .

Χ-Η«>

2) Να δείξετε ότι

f(x) - f(x - 1) < f'(x) < f(x + 1) - f(x)

για κάθε χ > ι . 3 ) Να βρείτε το lim f'(x) .

Χ-Η«>

Λύση 1) Επειδή η ευθεία ε : y = 3χ + 4 είναι aσύμπτωτη

της Cr στο +οο έπεται ότι lim ( f(x) - 3x - 4) = 0 ( 1 ).

Χ-Η«>

Θεωρούμε τη συνάρτηση g(x) = f(x) - 3χ - 4, χ > Ο .

Λόγω της ( 1 ) έχουμε lim g(x) = Ο . Χ -Η«>

Για κάθε χ>Ο είναι f(x) = g(x) + 3χ + 4 , οπότε f(x + l) - f(x) = = ( g(x + 1) + 3(x + 1) + 4) - ( g(x) + 3x - 4) = = g(x + 1) - g(x) + 3 : (2).

(ω=χ+Ι) Είναι lim g(x) = Ο τότε lim g(x + 1) = x�+co χ�

= lim (ω) = Ο , οπότε λόγω της (2) έχουμε: ω->+«>

lim (f(x + 1) - f(x)) = lim (g(x + 1) - g(x) + 3) = Χ -Η<(> Χ -Η«>

= lim g(x + 1) - lim g(x) + t'im3 = Ο - Ο + 3 = 3 . Χ -Η<Ό X---1-f«>

2) Έστω χ> 1 . Επειδή η f είναι παραγωγίσιμη στο (0, +οο) έπεται ότι η f ικανοποιεί τις προϋποθέ-σεις του Θ.Μ.Τ. σε καθένα από τα διαστήματα

[χ - 1, χ] και [χ, χ + 1] , οπότε υπάρχουν ξ, Ε (χ - 1, χ) και ξ2 Ε (χ, χ + 1) τέτοιοι ώστε

f'(ξ1 ) = f(x) - f(x - 1) = f(x) - f(x - 1) (3) και χ - (χ - 1)

f'(ξ2 ) = f(x + 1) - f(x) = f(x + 1) - f(x) (4) (χ + 1) - χ

Επειδή η f είναι κυρτή στο (0, +οο) , έπεται ότι η f' είναι γνησίως αύξουσα στο (Ο, +οο) . Έχoυ-

(f'tR: J με: Ο < χ - 1 < ξ, < χ < ξ2 < χ + 1 ::::::> ισχύει f'(ξ, ) < f'(x) < f'(ξ2 ) . Από (3), (4) f(x) - f(x - 1) < f'(x) < f(x + 1) - f(x) . (5) για κάθε χ > 1 .

u=x--1

3) Είναι lim ( f(x + 1) - f(x)) = 3 = χ�+«:ι

x=u+l

= lim ( f(u + 1) - f(u)) = 3 , οπότε λόγω της (5) u-H«>

προκύπτει ότι lim f' (χ) = 3 . Χ-++«>

ΠΡΟΤΕΙΝΟΜΕΝΕΣ

1) α) Να δείξετε ότι 2x (-Fx - 1) ;::: Rnx για κάθε χ>Ο.

β) Αν α, β, γ Ε IR* και αβγ = 1 να δείξετε ότι α.Jα + βJβ + γJΎ ;::: α + β + γ .

2) α) Να δείξετε ότι χ4 ;::: 4χ - 3 για κάθε χ E IR .

β) Να δείξετε ότι για κάθε α, β, γ Ε IR* ισχύει α4 β3 γ3 1 1 1 -+-+- ;::: - + - +- . β4 γ4 α• α β γ

3) Να λυθεί η aνίσωση ex3 +x - e2x > χ - χ3 •

4) Να συγκριθούν οι αριθμοί Α = 20042005 , Β = 20052004 .

5) Α ' β δ 'ξ ' α-β α2 + 1 ν ειναι α> να ει ετε οτι: e > -2- . β + 1

6) Έστω οι συναρτήσεις f, g : IR � IR για τις οποί­ες ισχύει e

f(x ) + f(x) = g(x) για κάθε χ Ε IR . Α ν η g είναι γνησίως φθίνουσα στο R, τότε: i) Να δείξετε ότι η f είναι γνησίως φθίνουσα

στο R. ii) Να λυθεί η aνίσωση f( f(x2 +2χ)} <f( f(x+2) 1 .

ΕΥΚΛΕΙΔΗΣ Β' λζ' τ.3/71

Page 74: Ευκλειδης Β 51

Πιθανότητες Κώστας Παπαδόπουλος - Χαρούλα Σαραφοπούλου

Το κεφάλαιο των Πιθανοτήτων για τα Μαθηματικά Γενικής Παιδείας της Γ' Λυκείου, έχει μία αυτοτ&

λεια σχετικά με τα άλλα δύο κεφάλαια που προηγούνται. Η ευχέρεια στην αντιμετώπιση θεμάτων, έχει να

κάνει με την καλή κατανόηση κάποιων βασικών σχέσεων και εννοιών. Ενδιαφέρον παρουσιάζουν ακόμη

οι ασκήσεις που συνδυάζουν στοιχεία από τα κεφάλαια της Στατιστικής και των Συναρτήσεων μέσα από το

άρθρο αυτό θα προσπαθήσουμε να παρουσιάσουμε μια επιλογή ασκήσεων που να ανταποκρίνονται στις α­παιτήσεις αυτού του κεφαλαίου.

Άσκηση 1 '1

Έστω Ω ο δειγματικός χώρος ενός πειράματος τύχης και Α, Β ενδεχόμενά του με πιθανότητες Ρ(Α) = 0,6 και Ρ(Β) = 0,7. Να δείξετε ότι: i) τα Α και Β δεν είναι ασυμβίβαστα στα ii) Ρ( Α) + Ρ(Β)- 1 � Ρ( ΑΠ Β) � min{P(A),P(B)}

iii) 0, 3 � Ρ(Α Π Β) � 0, 6 ίν) P(A U B) z 0, 7 .

Λύση i) Αν τα Α, Β είναι ασυμβίβαστα θα ισχύει:

P(A U Β) = Ρ( Α) + Ρ(Β) = 0, 6 + Ο, 7 = 1 ,3 > 1 το οποίο είναι άτοπο. Άρα τα Α και Β δεν είναι ασυμβίβαστα.

ii) Είναι Α Π Β ς Α και Α Π Β ς Β άρα Ρ(Α Π Β) s Ρ(Α) ή Ρ(Α Π Β) s Ρ ( Β) δηλ. P(A Π B) � min {P(A), P(B)} . Είναι P(A U B) = Ρ(Α) + Ρ(Β) - Ρ(ΑΠ Β) . Ισχύει P(A U B) s 1 � Ρ(Α) + Ρ(Β) - Ρ(ΑΠ B) s 1 � � Ρ(Α) + Ρ(Β) - 1 s Ρ(ΑΠ Β) . Τελικά ισχύει Ρ(Α) + Ρ(Β) - 1 � Ρ(ΑΠ Β) � min {P(A), P(B)} .

iii) Με απλή εφαρμογή της προηγούμενης σχέσης έχουμε Ρ( Α) + Ρ(Β) - 1 = 0, 6 + Ο, 7 - 1 = 0, 3 και ακόμη Ρ( Α Π Β) s Ρ( Α) = 0, 6 . Άρα τελικά 0, 3 � Ρ(Α Π Β) � 0, 6 .

iv) Είναι Β ς Α U Β άρα Ρ(Β) s (Α U Β) δηλαδή P(A U B) z 0, 7 .

Άσκηση 2'1

Α ν Α, Β, Γ ενδεχόμενα του δειγματικού χώρου Ω να δείξετε ότι

Ρ(Α Π Β Π Γ) � Ρ(Α) + Ρ(Β) + Ρ(Γ).

3

Λύση

Ισχύει Α Π Β Π Γ ς Α άρα Ρ(Α Π Β Π Γ) � Ρ(Α)

Όμοια Α Π Β Π Γ ς Β άρα Ρ(Α Π Β Π Γ) � Ρ(Β)

Α Π Β Π Γ ς Γ άρα Ρ(Α Π Β Π Γ) s Ρ(Γ)

( 1 ) .

(2).

(3) .

Με πρόσθεση των τριών σχέσεων παίρνουμε 3Ρ(Α Π Β Π Γ) s Ρ( Α) + Ρ(Β) + Ρ(Γ) �

� Ρ(Α Π Β Π Γ) � Ρ(Α) + Ρ(Β) + Ρ(Γ) . 3

Άσκηση 3η Α ν για τα ενδεχόμενα Α και Β ενός δειγματικού Ω ισχύει Ρ( Α Π Β) = Ρ( Α) · Ρ(Β) να δείξετε ότι Ρ( Α U Β)' = Ρ( Α') · Ρ(Β') .

Λύση Είναι P(A U B)' = 1 - P(A U Β) = = 1 - [Ρ(Α) + Ρ(Β) - Ρ(ΑΠ Β)] = = 1 - Ρ(Α) - Ρ(Β) + Ρ(ΑΠ Β) Ρ(ΑΠΒ)=Ρ(Α)· Ρ(Β)

1 - Ρ( Α) - Ρ(Β) · Ρ(Α)Ρ(Β) =

= [1 - Ρ( Α)] - [1 - Ρ( Α)] · Ρ(Β) =

[1 - Ρ( Α)] · [1 - Ρ(Β)] = Ρ( Α') · Ρ(Β') .

Άσκηση 4η Έστω Α, Β δύο ενδεχόμένα ενός δ.χ. Ω με Α Π Β = 0 και Α U Β = Ω . Αν ισχύει:

λΡ(Α) - Ρ(Β) = 3λ2 - 6λ + 2 (1)

Ρ( Α) + λΡ(Β) = 5λ2 + λ = 1 (2)

(λ ε �) να δειχθεί ότι Ρ( Α) = Ρ(Β) = .!. . 2

ΕΥΚΛΕΙΔΗΣ Β' λστ' τ.l/72

Page 75: Ευκλειδης Β 51

Μαθηματικά Γ' Λυκείου

Λύση

Οι ισότητες ( 1 ) και (2) σχηματίζουν σύστημα με αγνώστους Ρ(Α) και Ρ(Β).

Είναι D = I � �1 1 = λ2 + 1 * ο για κάθε λ Ε � •

3λ2 - 6λ + 2 -1 D = = Ρ(Α) 5λ2 + λ + -1 λ

= 3� - 6λ2 + 2λ + 5λ2 + λ - 1 = = 3� - λ2 + 3λ - 1 = λ2 (3λ - 1) + (3λ - 1) =

= (3λ - 1) . (λ2 + 1) . λ 3λ2 - 6λ + 2 D = =

Ρ( Β) 1 51! + λ - 1

= 5� + λ2 - λ - 3λ2 + 6λ - 2 = = 5� - 2λ2 + 5λ - 2 = λ2 (5λ - 2) + (5λ - 2) = = (5λ - 2)(1! + 1) .

Επειδή D * Ο το σύστημα έχει μοναδική λύση που δίνεται από τους τύπους. Ρ( Α) = DP(A) = (3λ - 1)(λ2 + 1) = 3λ - 1 D λ2 + 1

Ρ(Β) = DP(A) = (5λ - 2) · (λ2 + 1) = 5λ - 2 . D (λ2 + 1) Γνωρίζουμε ότι Ρ( Α U Β) = Ρ( Ω) � Ρ( Α U Β) = 1 � Ρ(Α) + Ρ(Β) = Ρ(Α Π Β) = 1 � Ρ(Α) +Ρ(Β) =1 . Είναι Ρ(Α Π Β) = Ο .

Δηλαδή 3λ - 1 + 5λ - 2 = 1 � 8λ = 4 � λ =.!:. . 2

1 Για λ = -2 1 1 1 5 1 Ρ(Α) = 3 · - - 1 = - Ρ(Β) = 5 · - - 2 = -- 2 = - . 2 . 2 ' 2 2 2

Άρα Ρ( Α) = Ρ(Β) = .!:. . 2

Άσκηση Sη

Αν Α, Β είναι δύο ενδεχόμενα ενός δ.χ. Ω και ,

Ρ(Α Π Β) 2Ρ(Α) + Ρ(Β)

δ 'ξ ,

ισχυει = να ει ετε οτι 3

Ρ(Α) = Ρ(Β) .

Λύση

Γνωρίζουμε ότι Α Π Β � Α και Α Π Β � Β οπότε Ρ( Α Π Β) :::; Ρ( Α) και Ρ( Α Π Β) :::; Ρ(Β) . Έχουμε Ρ(Α Π Β) :s; Ρ(Α)

� 2Ρ(Α) + Ρ(Β) :::; Ρ( Α) � Ρ(Β) :::; Ρ( Α) 3 Όμοια Ρ(Α Π Β) :s; Ρ(Β)

� 2Ρ(Α) + Ρ(Β) :::; Ρ(Β) � 3 � Ρ( Α) :::; Ρ(Β) (2). Από τις ( 1 ) και (2) έχουμε Ρ(Α) = Ρ(Β) .

Άσκηση 6η

Δίνεται η συνάρτηση Α( χ) = χ4 - 2χ2 + 2 .

i) Να βρεθούν τα ακρότατα της f.

( 1 ) .

ii) Αν Ω = {(α, β)/ α, β Ε �} και α τοπικό ακρότατο της f και ακόμη α = f(β) να βρεθεί

το σύνολο Ω. iii) Α ν Ω δειγματικός χώρος να δείξετε ότι για

την εξίσωση χ2 - 2(α + β)χ + 4α = Ο η

πιθανότητα να μην έχει πραγματικές ρίζες, είναι διπλάσια από την πιθανότητα να έχει ρίζα τον αριθμό 2.

Λύση

Είναι _f'(x) = 4χ3 - 4χ = 4χ · (χ2 - 1) f'(1) = 0 � 4x · (x2 - 1) = 0 � x = O ή χ = - 1 ή χ = 1 . Κατασκευάζουμε τον πίνακα μεταβολής προσήμου για την f .

χ -οο -1 ο 1 +οο

χ - - + +

r-1 + - - +

f(x) - + - +

f(x) � / � / Επομένως η συνάρτηση f παρουσιάζεις το χ = - 1 τοπικό ελάχιστο, στο χ = Ο τοπικό μέγιστο και στο χ = 1 τοπικό ελάχιστο. ii) Είναι f(-1) = 1 άρα (α, β) = (1, - 1) .

f(0) = 2 άρα (α, β) = (2, 0) . f(l) = 1 άρα (α, β) = (1, 1) . Π. χ. Άρα το σύνολο Ω είναι: Ω = { (1, - 1), (2, 0), (1, 1)} .

iii) Η εξίσωση δεν έχει πραγματικές ρίζες όταν Δ < 0 � 4(α+ β)2 - 16α < Ο� (α+ β)2 - 4α < Ο . Παρατηρούμε ότι τα ζεύγη από το Ω, που την επαληθεύουν είναι το ( 1 ,- 1 ) και (2,0). Έτσι αν

ΕΥΚΛΕΙΔΗΣ Β' ί.ζ τ.3Π3

Page 76: Ευκλειδης Β 51

Μαθηματικά Γ Λυκείου

θεωρήσουμε Α το ενδεχόμενο η εξίσωση δεν έχει πραγματικές ρίζες, τότε σύμφωνα με τον κλασσικό ορισμό της πιθανότητας θα έχουμε Ρ( Α) = Ν(Α) = � . Η εξίσωση έχει ρίζα τον Ν(Ω) 3 αριθμό ρ = 2 ισοδυναμεί : 22 - 2(α + β) · 2 + 4α = Ο � 4 - 4f!l, - 4β + .Α' α = Ο 4β = 4 � β = 1 . Για β = 1 έχουμε το ζεύγος ( 1 , 1 ). Άρα αν θεωρήσουμε Β το ενδεχόμενο η εξίσωση έχει ρίζα τον ρ = 2 τότε Ρ(Β) =

Ν(Β) = .!_ . Δηλαδή Ρ( Α) = 2Ρ ( Β) . Ν(Ω) 3

Άσκη ση 711

Μία κληρωτίδα περιέχει άγνωστο αριθμό κλήρων ν, aριθμημένων: ι, 2, 3, ... , ν. Επιλέyουμε στην τύχη ένα κλήρο. Αν η mθανότητα να επιλέξουμε περιττό αριθμό είναι κατά 4% μεγαλύτερη από τον να επιλέξουμε άρτιο, να βρεθεί ο ν.

Λύση Επειδή, ο δειγματικός χώρος αποτελείται από απλά ισοπίθανα ενδεχόμενα και ακόμη η mθανότητα να ε­mλέξουμε περιττό αριθμό είναι μεγαλύτερη από το να εmλέξουμε άρτιο. Άρα ο ν είναι περιττός αριθμός. Έστω Α το ενδεχόμενο, επιλέγουμε περιττό αριθ-μό . Τότε Ρ( Α) = Ν(Α) = ν + 1 = ν + 1 . Ν(Ω) 2 2ν

ν Ε δ , λ ' θ , , ν + 1 πει η το π η ος των περιττων ειναι -2- . Όμοια αν Β το ενδεχόμενο να επιλέξουμε άρτιο,

ν - 1 τότε Ρ(Β) = Ν(Α) = 2 = ν - 1 . Ν(Ω) ν 2ν Η πιθανότητα Ρ(Α) είναι κατά 4% μεγαλύτερη από την πιθανότητα Ρ(Β) . , ν + l ν - 1 Άρα - =-+0,04 � ν + 1 = ν - 1 + 2ν · Ο, Ο4 � 2ν 2ν � 2 = Ο 08 · ν � ν = -2- = 25 . ' ο 08 ' Άσκηση 8'1 Έστω Ω = {ι ,2,3,4,5} ο δ.χ. Ω ενός πειράματος

Ρ(ι) Ρ(2) Ρ(3) Ρ(4) Ρ(5) -- = -- = -- = -- =--2 5 3 4 6

α) Βρείτε τις πιθανότητες των στοιχειωδών ενδεχομένων του Ω.

β) Θεωρούμε τη συνάρτηση f(x) = eμ• , χ Ε IR , με Ω και τα ενδεχόμενα

Α = {μ Ε Ω/ f"(Ο) - 3f '(0) + 2f(O) = Ο} . Β = {μ Ε Ω/ f"(0) > 3} .

i) Να παρασταθούν με αναγραφή τα ενδεχόμενα Α και Β.

ii) Να βρεθούν οι πιθανότητες των ενδεχομένων Α Π Β, Α U Β, Α' Π Β .

Λύση α) Είναι Ρ(Ω) = 1 � P( l )+Ρ(2)+Ρ(3)+Ρ( 4)+Ρ(5) =

5 3 = 1 � P(l) +-P(l) +-P(l) + 2P(l) + 3P(l) = 1 � 2 2 1 � l OP(l) = 1 � P(l) = - . 10

Α ' ' ' θ ' p· (2) 5 1 1 πο την αρχικη σχεση α εχουμε = 2" ·ΪΟ = '4 ,

3 1 3 1 1 P(3) = 2 ·w = 20 , P(4) = 2P(1) = 2 ·IO = S και

Ρ(5) = 3Ρ(1) =2 . 10 β) i) Είναι f'(x) = μeμχ , f"(x) = μ2eμχ , f(O) = 1,

f'(O) = μ · e0μ, f"(O) = μ2 • e0 = μ2 άρα Α = {μ Ε Ω / f"(Ο) - 3f'(O) + 2f(O) = Ο} = = {μ Ε Ω / μ2 - 3μ + 2 = 0} = {1, 2} Β = {μ Ε Ω / f"(0) > 4} . Όμως f"(O) > 4 � μ2 > 4 � μ > 2 ή μ < -2. Επειδή μ Ε Ω άρα δεχόμαστε μ>2 δηλ. μ Ε {3, 4, 5} άρα Β = {3 ,4,5 } .

ii) Είναι Α Π Β = 0 άρα Ρ(Α Π Β) = Ο Α U Β = {1, 2, 3, 4, 5} = Ω άρα P(A U B) = Ρ(Ω) = 1 Β Π Α' = Β -Α = Β άρα Ρ(Β Π Α') = Ρ(Β - Α) = Ρ(Β) =

3 1 3 1 3 = Ρ(3) + Ρ(4) + Ρ(5) =-+- + - = - . 20 5 1 0 20 Άσκηση 9'1

τύχης. Για τις πιθανότητες των στοιχειωδών Έστω η συνάρτηση ι ι

f(x) = - + ­x ι - χ

όπου

ενδεχομένων του Ω ισχύει: Χ Ε (Ο, ι) .

ΕΥΚΛΕΙΔΗΣ Β' λζ' τ.3/74

Page 77: Ευκλειδης Β 51

Μαθηματικά Γ' Λυκείου

i) Να βρείτε την ελάχιστη τιμή της f.

ii) Α ν Α ένα ενδεχόμενο του δειγματικού χώρου

Ω, το οποίο δεν είναι αδύνατο αλλά ούτε και

βέβαιο, να δείξετε ότι: _ι_+ _ι_ � 4 •

Ρ(Α) Ρ(Α')

Λύση

i) Είναι f'(x) = (�+-1-) ' = --1 + 1 = . χ 1 - χ χ2 (l - x)2

Είναι f'(x) = 0 <::::> 2x - 1 = 0 <::::> x = ..!. . 2 ι 2χ - 1 1 f (x) > O <::::> 2 2 > Ο <::::> χ > -χ (1 - χ) 2 1 2Χ - 1 1 f (x) < O <::::> 2 2 < Ο <::::> χ < -χ (1 - χ) 2

Κάνουμε τον πίνακα μεταβολής της f. Ά 1 ' ζ λ ' ' ρα στο χ = "2 παρουσια ει ε αχιστο το οποιο

είναι r (�) = 2 + 2 = 4 . Δηλαδή για κάθε

χ ε (0, 1) f(x) � 4 .

χ ο 1/2 1 Γ( χ) - +

f(x) � / ii) Έστω Ρ( Α) = χ , οπότε Ρ( Α') = 1 - χ άρα

1 1 1 1 -- +-- � 4 <::::> - +-- � 4 <::::> f(x) � το Ρ( Α) Ρ( Α') χ 1 - χ οποίο ισχύει από το προηγούμενο ερώτημα.

Άσκηση l Οη

Από τους μαθητές ενός Λυκείου το 20% αυτών

συμμετέχει στο διαγωνισμό ης ΕΜΕ ενώ το

85% δεν συμμετέχει στο διαγωνισμό της Ε.Ε.Φ.

και το 8%, συμμετέχει και στους δύο

διαγωνισμούς. Επιλέγουμε τυχαία ένα μαθητή.

Να βρείτε την πιθανότητα των ενδεχομένων:

Γ: Ο μαθητής να μη συμμετέχει σε κανένα από

τους 2 διαγωνισμούς.

Δ: Ο μαθητής να συμμετέχει σ' ένα μόνο

διαγωνισμό.

Ε: Ο μαθητής να συμμετέχει μόνο στο

διαγωνισμό της Ε.Μ.Ε.

Ζ: Ο μαθητής να συμμετέχει το «πολύ» σ' ένα

διαγωνισμό.

Λύση

Θεωρούμε τα ενδεχόμενα: Α: Ο μαθητής συμμετέχει στο διαγωνισμό της

ΕΜΕ. Β: Ο μαθητής συμμετέχει στο διαγωνισμό της

Ε.Ε.Φ. Από τα ενδεχόμενα έχουμε:

20 85 8 Ρ(Α) = - και Ρ(Β') = - και Ρ(ΑΠΒ) =- . 1 00 1 00 100 Έχουμε: P(A U B) = Ρ(Α) + Ρ(Β) = Ρ(Α Π Β) =

20 1 5 8 27 = -+- -- = -100 1 00 1 00 1 00 .

Είναι: Γ = Α' Π Β' = (A U Β)' . Οπότε

Ρ(Γ) = Ρ[ (AU Β)'] = 1 - P(AU Β) = 1 -Ε._ =.Ε_ 100 1 00

ή 73%. Δ = (A - B) U (B -A) με Α - Β, Β - Α ασυμβίβαστα. Άρα Ρ(Δ) = Ρ(Α - Β) + (Β - Α) = = Ρ(Α) - Ρ(Α Π Β) + Ρ(Β) - Ρ(Α Π Β) =

= Ρ( Α U Β) - Ρ(Α Π Β) = Ε_ -__!__ = _!2_ ' 1 00 1 00 1 00 η

1 9% Ε = Β - Α ::::> Ρ(Ε) = Ρ(Β -Α) = Ρ(Β) - Ρ(Α Π Β) ' Ρ(Ε) = 20 _ __!__ = � ' 1 2%. η 1 00 1 00 1 00 η Ζ = (Α Π Β)' ::::> Ρ(Ζ) = Ρ [(Α Π Β)'] =

= 1 - Ρ(Α Π Β) - 1 -_!_ = 92 ή 92%. 1 00 1 00

ΕΥΚΛΕΙΔΗΣ Β' λζ' τ.3/75

Page 78: Ευκλειδης Β 51

r Θcώpημα Μέιιη,� Τιμή� Β

� , � Δ ' .... αιrιιιες εφαpμογες και . .. «ανοι«τα vεματα»

Το Θεώρημα Μέσης τιμής (Θ.Μ.Τ.) είναι από τα πιο βασικά θεωρήματα του Διαφορικού Λογι­σμού. Κάποιες θεωρήσεις πάνω στο θεώρημα αυτό δημιουργούν σημαντικές μαθηματικές επινοήσεις για την επίλυση προβλημάτων. Εξυπακούεται ότι θα περιοριστούμε σε ζητήματα συναφή με το επί­σημο σχολικό πρόγραμμα. Ας ξεκινήσουμε με τη διατύπωση του Θεωρήματος.

Έστω f : [α, β] � IR συνεχής συνάρτηση, η ο-ποία είναι παραγωγίσιμη στο ανοικτό διάστημα (α,β) . Τότε υπάρχει ένα τουλάχιστον ξ Ε (α, β) τέ-

τοιος ώστε: f'(ξ) = f(β) - f(α) . β - α

Γενικότερα: Αν μια συνάρτηση f, ορισμένη σ' ένα διάστημα Δ, είναι συνεχής στο Δ και παραγ� γίσιμη στο εσωτερικό του Δ, τότε για οποιουσδή­ποτε χ1 Ε Δ, χ2 Ε Δ ισχύει: f(x1 ) - f(x2 ) = f'(x0 )(x1 - χ2 ) ·

όπου ο χ0 είναι συγκεκριμένο στοιχείο του ε­σωτερικού μέρους του Δ.

Σχόλια - Μεθοδεύσεις: • Το συμπέρασμα του Θ.Μ.Τ. μας λέει ότι η

εξίσωση (β - α)f'(χ) :: f(β) - f(α) έχει μία τουλάχιστον λύση στο (α, β).

• Ακόμη σε αρκετά ζητήματα που παρουσιάζεται ο λο' γος f(β) - f(α) , , μπορει να εφαρμοστει το

β - α Θ.Μ.Τ.

Παράδειγμα:

ι . π

Να αποδειχθεί «αν Ο ::;; α < β < - , 2

t'n( συνα) - t'n( συνβ) β εφα < < εφ ».

β - α

Απάντηση

τότε:

Αρχικά φαίνεται δύσκολο. Με μεγαλύτερη προσο­χή όμως παρατηρούμε ότι αν θέσουμε

Τριαντάφυλλος Τριανταφύλλου

f(x) = t'η(συνχ) /[α, β] , τότε: t'n( συνα) - t'n( συν β) f( α) - f(β)

β - α β - α και συνεπώς η αποδεικτέα παραπέμπει στο Θ.Μ.Τ. Η συνάρτηση f(x) = t'η(συνχ) /[α, β] ικανοποιεί τις υποθέσεις του Θ.Μ.τ. Άρα υπάρχει ξ Ε (α, β)

τέτοιος ώστε: f '(ξ) = f(β) - f(α) . β - α

Δηλαδή : -ημξ ·-1- = t'η(συνβ) - t'η(συνα) = συνξ β - α

t'n( συνα) - t'n( συν β) = β - α

Ά ξ t'n( συ να) - fn( συν β) ρα: εφ = . β - α

Η αποδεικτέα λοιπόν γίνεται: εφα < εφξ < εφβ και είναι αληθής διότι η συνάρτηση «εφαπτομένη» είναι γνήσια αύξουσα στο πρώτο τεταρτημόριο. 2. Να αποδειχθεί ότι: «αν α>Ο, τότε

ι α + ι ι -- < t'n -- < - » α + l α α

(την αφήνουμε για εξάσκηση)

• Μια χρήσιμη παρατήρηση : .,. Α ν μια συνάρτηση f : [α, β] � IR ικανοποιεί

τις υποθέσεις του Θ.Μ.Τ. και υπάρχει ρ>Ο τέτοιος ώστε lf'(x) l < ρ για κάθε Χ Ε (α,β) ,

τότε: lf(β) - f(α) l < ρ(β - α) .

Α ν μια συνάρτηση f : [α, β] � IR ικανοποιεί

τις υποθέσεις του Θ.Μ.Τ., m = min { f(x) / χ Ε [α, β]} ,

Μ = max {f(x) / x Ε [α, βJ} για κάθε

χ Ε (α, β) ισχύει: m ::;; f'(x) ::;; Μ τότε

f(α) + m(x - α) ::;; f(x) ::;; f(α) + Μ( χ - α) ,

για κάθε χ Ε [α, β] .

ΕΥΚΛΕΙΔΗΣ Β' λστ' τ.1Π6

Page 79: Ευκλειδης Β 51

Μαθηματικά Γ Λυκείου

Παραδείγματα:

ι . Αν μια συνάρτηση f : [Ο, ι] � JR ικανοποιεί

τις υποθέσεις του Θ.Μ.Τ., f(O) = -2 και για

κάθε χ ε (Ο, ι) ισχύει: lf'(x) l < 5 , να

αποδειχθεί ότι: lf(ι) Ι < 7 .

Απάντηση Σύμφωνα με το Θ.Μ.Τ. υπάρχει ξ Ε (0, 1) τέτοιος

ώστε: f ' (ξ) = f(l) - f(O) = f(l) + 2 . 'Ε χουμε ακό-1 - 0 μη : lf ' Cξ> l < 5 {:} l fω + 21 < 5 . Κι επειδή : lf(l)l - 2 � l f(l) + 21 , συμπεραίνουμε ό­τι: jf(l)j - 2 < 5 ή lf(1) j < 7 .

2 . Ας πάρουμε μια συνάρτηση f : [α, +οο) � JR ,

που είναι συνεχής στο α και παραγωγίσιμη στο (α, +οο) . Α ν υπάρχει k>O τέτοιος ώστε:

lf'(x) l 2:: k , για κάθε χ ε (α, +οο) , να

αποδειχθεί ότι: lim lf(x) l = +οο . Χ-Η«>

Απάντηση Έστω χ>α. Εφαρμόζουμε το Θ.Μ.Τ. στο [α,χ] . Ο­πότε υπάρχει ξ Ε (α, χ) τέτοιος ώστε:

f'(ξ) = f(x) - f(α) . Κι επειδή lf '(ξ)l 2:: k συμπε-χ - α ραίνουμε ότι: jf(x) - f(α)l 2:: k(x - α) . Ωστόσο: lf(x)j + jf(α)j 2:: j f(x) - f(α)j . Άρα: j f(x) j 2:: k(x - α) - jf(α)j . Κι επειδή lim {k(x - α) - jf(α)j} = +oo , συ-

χ--++οο μπεραίνουμε ότι: lim j f(x)j = +οο .

χ-.+οο

Ζητήματα που αφορούν στην ύπαρξη κάποιου αριθμού. ι . Για μια συνάρτηση f : [α, β] � JR δεχόμα-

στε ότι ικανοποιεί τις υποθέσεις του Θ.Μ.Τ. και ότι f(α) = f(β) .

Να αποδειχθεί ότι: «υπάρχουν ξ1,ξ2,� με α < ξι < ξz < ξ3 < β τέτοιοι ώστε:

2f'(ξι ) + 3f'(ξz ) + 4f'(ξ3 ) = Ο .

Απάντηση Ορίζουμε: κ, λ Ε (α, β) τέτοιους ώστε:

2 4 κ - α = 9(β - α) , β - λ = "9(β - α) .

Δηλαδή: κ = 7α + 2β' λ = 4α + 5β . 9 9

Οπότε: β - α λ - κ = -- και εφαρμόζουμε το 3 Θ.Μ.Τ. σε καθένα από τα κλειστά διαστήματα: [α, κ] , [κ, λ] , [λ, β] . Συνεπώς υπάρχουν ξι Ε (α, κ), ξ2 Ε (κ, λ), ξ3 Ε (λ, β) τέτοιοι ώστε:

f'(ξι ) = f(κ) - f(α) = .2. · f(κ) - f(α) , κ - α 2 β - α

f'(ξz ) = 3 · f(λ) - f(κ) , f '(ξ3 ) = .2. . f(β) - f(λ) . β - α 4 β - α

Κι επομένως: 2f' (ξι ) + 3f' (ξ2 ) + 4f' (ξ3 ) = = 9[(f(κ) - f(α)) + (f(λ) - f(κ)) + (f(β) - f(λ))] =

= 9 f(β) - f(α) = Ο β -α

2. Για μια συνάρτηση f : [α,β] � JR

δεχόμαστε ότι ικανοποιεί τις υποθέσεις του Θ.Μ.Τ. και ότι f(α) < f(β) . Να δειχθεί ότι

υπάρχουν ξι,ξ2,� με α < ξι < ξz < ξ3 < β

τέτοιοι ώστε:

__ 2_ + __ 3_ + __ 4_ = 9 · β - α

f'(ξι ) f'(ξz ) f'(ξ3 ) f(β) - f(α)

Απάντηση Θεωρούμε: Υι ,Υz τέτοιους ώστε Υι -f(α) =i (f(β)-f(α)) , f(β) -y2 =� (f(β) -f(α)) .

7f(α)+ 2f(β) 4f(α) + 5f(β) Δηλαδή : Υι 9 ' Yz = 9 .

Επειδή f(α) < Υι < f(β) συμπεραίνουμε, κατά το Θεώρημα Ενδιάμεσων Τιμών, ότι υπάρχει Χι Ε (α, β) τέτοιος ώστε: f(χι ) = Υι ή

f(χι ) = 7f(α) + 2f(β) (2). 9 Κι επειδή f(χι ) < y2 < f(β) υπάρχει χ2 Ε (χι , β)

τέτοιος ώστε f(x2 ) = y2 ή f(x2) 4f(α)+5f(β) (3). 9

Οπότε: f(α) < f(χι ) < f(x2 ) < f(β) . Εφαρμόζουμε το Θ.Μ.Τ. σε καθένα από τα δια­στήματα: [α, χι ] , [χι , χ2 ] , [χ2 , β] . Και συμπεραί-νουμε ότι υπάρχουν ξι ,ξz,ξ3 με α < ξι < Χι < ξ2 < χ2 < ξ3 < β τέτοιοι ώστε:

ΕΥΚΛΕΙΔΗΣ Β' λζ' τ.3/77

Page 80: Ευκλειδης Β 51

Μαθηματικά Γ Λυκείου

fΙ(ξι ) = f(χ ι ) - f(α)' χ1 - α fΙ(ξ2 ) = f(x2 ) - f(χι ) ,

Χ2 - χι

f1(ξ3 ) = f(β) - f(x2 ) ή f1(ξι )� 2 (f(β) - f(α)) ,

β - χ2 9(χ1 - α) Ι � Ι � ) 2 ( f(β) - f(α)) I (3) 4 (f(β) - f(α))

f (� , ) = , f (ξ3 ) = . -- ι 3 ι 3(χ2 - χ1 ) 9(β - Χ2 )

κ ' 2 3 4 αι συνεπως: fΙ(ξι )

+ fΙ(ξ2 )

+ fΙ(ξ3 )

=

= 9 · (χ ι - α) + (χ2 - χι ) + (β - χ2 ) = f(β) - f(α)

= 9-=--β_-_α_ f(β) - f(α)

3 ) Δίνεται η συνάρτηση

f(x) = {(χ - 1)-..Γχ, Ο ::; χ < 1

(Χ - 1)2 ημχ, Χ � 1

i) Να εξεταστεί αν η συνάρτηση αυτή

ικανοποιεί τις υποθέσεις του Θ.Μ.Τ. στο

κλειστό διάστημα [0,1 ] .

ii) Α ν είναι καταφατική η απάντηση στο

προηγούμενο ερώτημα, να βρεθούν τα

αντίστοιχα ξ του Θεωρήματος. ίίί) Να δειχθεί ότι υπάρχει χ0 ε (0, 1)

τέτοιος ώστε: «η εφαπτομένη της Cr στο Ρ0 ( χ0 , f ( χ0 )) είναι παράλληλη προς τον

άξονα χ ' χ.»

Απάντηση

• Για το (i) Για κάθε χ Ε (0, 1) ισχύει:

1 Γ Χ - 1 3χ - 1 f (χ) = νχ + Γ = Γ

Κι επειδή 2νχ 2νχ

lim f(x) = lim [ <χ - l)fx] = ο = f(O) χ-+0 χ-+0 lim f(x) = lim [ <χ - 1)fx] = Ο = Χ-+Γ χ-+Γ = f(l) = lim (χ - 1)2 ημχ = lim f(x) Χ-+1+ Χ-+)+

( 1 ) .

και

Και συνεπώς η f /[0, 1] ικανοποιεί τις υποθέσεις του Θ.Μ.Τ. • Για το (ii) :

Κατά το Θ.Μ.Τ. υπάρχει ξ Ε (0, 1) τέτοιος ώστε:

fl(ξ) --f(l) - f(O) --Ο . Λ , ( 1 ) , ογω της εχουμε: 1 - 0 Ι 1 f (ξ) = ο {::} ξ = - . 3

• Για το (iii) :

Η εφαπτομένη της Cr στο Ρ0 ( � , 31) είναι πα-ράλληλη προς τον άξονα χ 'χ αφού f ( �) = Ο .

Παράδειγμα:

4) Ας πάρουμε μια συνάρτηση f, που είναι δύο

φορές παραγωγίσιμη σ' όλο το R. Α ν η

εφαπτομένη της Cr σ' ένα σημείο Α (α,f(α)) τέμνει τη Cr σε δύο άλλα σημεία

Β (β,f(β) ) , Γ( γ,f(γ)) με β<γ, να δειχθεί ότι:

«η εξίσωση f"(x) = Ο έχει τουλάχιστον δύο

λύσεις».

Υ Απάντηση

α � γ χ

Η εξίσωση της (ε) είναι: y = f(α) + f1(α)(χ - α) ( 1 ) . Άρα: f(β) = f(α) + f1(α)(β - α) (2) και f(γ) = f(α) + f1(α)(γ - α) (3) . Η f ικανοποιεί τις υποθέσεις του Θ.Μ.Τ. στο κλει­στό διάστημα [α, β] και στο κλειστό διάστημα [β,γ] . Συνεπώς υπάρχουν ξ1 Ε (α, β), ξ2 Ε (β, γ) τέ-

f(β) - f(α) (2) τοιοι ώστε: f1(ξ1 ) = = f1(α) (4) και β - α

fl(ξ2 ) = f(β) - f(γ) � β - γ (3)

[f(α) + f1(α)(β - α)] - [f(α) + f1(α)(γ - α)] β - γ

(4) = f(α) = f1(ξ1 )

ΕΥΚΛΕΙΔΗΣ Β' λζ' τ.3/78

Page 81: Ευκλειδης Β 51

Μαθηματικά Γ' Λυκείου

Η συνάρτηση τώρα f ' ικανοποιεί τις υποθέσεις του Θεωρήματος Rolle στο κλειστό διάστημα [α, ξι ] και στο διάστημα [ξι , ξ2 ] .

3. Για μια συνάρτηση f : [α, β] ----7 JR: δεχόμαστε ότι ικανοποιεί τις υποθέσεις του Θ.Μ.Τ . . Αν f"(x) > Ο , για κάθε χ ε (α, β) τότε η συνάρ-

Άρα υπάρχουν κ ε (α, ξι ) και λ ε (ξι , ξ2 ) τέτοιοι ώστε: f"(κ) = Ο και f"(λ) = Ο .

( ) f(x) - f(α) , , ' ξ τηση g χ = ειναι γνησια αυ ουσα χ - α στο (α,β] .

4. Α ν μια συνάρτηση f : [α, β] ----7 JR: είναι κυρτή Μερικά ζητήματα για περαιτέρω εμβάθυνση (γιατί το . . . . Θ.Μ. τ. είναι ανεξάντλητο)

( στρέφει δηλαδή τα κοίλα άνω), τότε για κάθε χ ε (α, β) ισχύει: f(x) - f(α) < f(β) - f(x) . χ - α β - χ 1 . Αν μια συνάρτηση f : JR: ----7 JR: είναι δύο φορές

παραγωγίσιμη με f"(x) < Ο για κάθε χ ε JR: , 5. Αν μια συνάρτηση f, ορισμένη σ' ένα διάστημα Δ είναι κοίλη (δηλαδή στρέφει τα κοίλα κάτω) κατά μήκος του Δ, τότε για οποιουσδήποτε

τότε για τον οποιονδήποτε α ε JR: ισχύει: f(x) � f(α) + f'(α)(χ - α) , για κάθε χ.

2. Για μια συνάρτηση f : JR: ----7 JR: με f(O) = Ο δε- α ε Δ, β ε Δ ισχύει: f(α)+f(β) �2f(α;β) )) . χόμαστε ότι: «στρέφει τα κοίλα κάτω» δηλαδή είναι κοίλη . Να αποδειχθεί ότι: «η συνάρτηση g(x) = f(x) /(0, +οο) είναι γνήσια φθίνουσα». χ

::::; .... ι fΕΝΙΚΑ θΕΜΑΤΑ

ΜΑθΗΜΑtΙΚΩΝ

ΓΕΝ Ι ΚΑ ΘΕΜΑΤΑ ΜΑΘΗΜΑτΙΚΩΝ

Γ' ΛΥΚΕΙΟΥ ΘΗΙΚΗΣ • ΤΕΧΝΟΛΟΓΙΚΗΣ

ΚΑΤΕΥΘΥΝΣΗΣ

Συγγραφική ομάCiα Μαθηματικών: Β . ΒΛΑΧΟΣ · Π . ΚΟΥrΣΟγΚQΣ ­Π . ΞΗ ΡΟΚΩΣΤΑΣ - Χ. ΠΛΑΤΗΣ • 320 θέματα από όλη την ύλη όλα aπαντη­

μ έ ν α . Σ υ ν δ υ α σ τ ι κ ά θ έ μ α τ α . Τ α λ ε π τ ά σ η μ ε ία των Μ α θ η ματικών. Γ ι α τ ο υ ς μαθ η-

τές που στοχεύουν σ το 30 και 40 θέμα. Γ ι α τους καθηγητές π ου διδάσκουν σ το σχολείο, σ το φροντιστ ή ριο, σ το ιδιαίτε ρο. Ένα β ιβλίο που ή δ η αγαπήσατε.

ΦΡΟΝτΙΗΗ ΡΙΑΚΗ ΦΥΣ Ι ΚΗ Β' ΛΥΚ Ε Ι ΟΥ

ΓΕΝΙΚΗΣ ΠΑΙΔΕΙΑΣ

Συγγραφική ομάCiα Φυσικών: Β. ΒΛΑΧΟΣ - Δ. Φ Ι Λ Ι Π ΠΑΚΟΠ QγΛQΣ ­Κ. ΑΝΔΡΟΥrΣΑΚΗΣ • Όλη η θ ε ω ρ ία, ε ρ ω τ ή σ ε ι ς aπαντ η μένες και χω ρίς απάντ ηση, πολλές ασκήσεις λυμέ­νες αναλυτικά. Τα λεπτά ση μεία τ η ς φυσικής ανά παράγραφο (να προσέχετε ότι :) . Δοκιμασμένο στην πράξη:

κΛΕκmtχοt ΦPOJiliΠHPIAI!!I

ΦΥΣΙΚΗ Β' ΛVKEIOV

Μ α θ η τ ή ς π ο υ θα διαβάσει τ η θ ε ω ρ ία, τ ι ς "-'--=------' παρατ η ρ ήσεις και τ ις λυμένες ασκήσεις γράφει πάνω από 15 . Μια διαφορετική φ ιλοσοφία σ τ ο σχολικό β ο ή θ η μ α .

ΕΚΔΟΣΕΙΣ: ΒΑΣΙΛΗΣ ΒΛΑΧΟΣ (Μαθημαrικός και Φυσικό� Παv. Αθήνας)

Παπαφλέσσα 3 Ζεφύρι Τ.Κ. 1 3451 Τηλ.: 2102384834, Fax: 210 2320954

ΕΥΚΛΕΙΔΗΣ Β' λζ' τ.3/79

Page 82: Ευκλειδης Β 51

21� ΠΑΝΕΛΑΙΙΝΙΟ ΣΥΝΕΔΡΙΟ ΜΑθΗΜΑΤΙΚΗΣ ΠΑΙΔΕΙΑΣ

Τα Μαθηματικά εκτός από γνωστικό αντικείμενο είναι και μια συνιστώσα που ενυπάρχει σε κάθε σκεπτόμενο άτομο και που ζητάει να βοηθηθεί από το διδάσκοντα στο να παρουσιαστεί στην πραγματική της μορφή ως εργαλείο και μέθοδος ζωής και ως ρυθμός αρμονίας του βίου και της προσωπικότητας του ατόμου.

Ο σχεδιασμός του Αναλυτικού Προγράμματος των Μαθηματικών θα πρέπει να προβλέπει την οικοδόμηση μαθηματικής σκέψης από το μαθητή και ταυτόχρονα την ανάπτυξη μαθηματικής γνώσης. Η σχολική πρακτική θα πρέπει να αναπτύσσει τρόπους διδασκαλίας σε πραγματικές συνθήκες, ώστε να υλοποιεί τους στόχους του Αναλυτικού Προγράμματος, να αξιολογεί τη μάθηση και, κατά συνέπεια, την καταλληλότητα του σχεδιασμού.

Με σκοπό τη συμβολή στην έμπρακτη εφαρμογή των παραπάνω σκέψεων, τα συνέδρια Μαθηματικής Παιδείας της Ε.Μ.Ε. πέρα από κάπω θέματα διδακτικής με εφαρμογές στα Μαθηματικά, θα εξετάζουν και πιο συγκεκριμένα θέματα που αφορούν αποκλειστικά τα Μαθηματικά ως αντικείμενο και ως μάθημα.

Ο στόχος του 21ου Συνεδρίου Μαθηματικής Παιδείας της Ε.Μ.Ε. είναι να αναδείξει συγκεκριμένα προβλήματα σχεδιασμού = εφαρμογής του Αναλυτικού Προγράμματος που σχετίζονται με τη φύση, τη δομή και τις εφαρμογές της μαθηματικής επιστήμης και να επεξεργαστεί προτάσεις σχετικές με τις εκπαιδευτικές βαθμίδες και τις τάξεις σύνδεσής τους.

Το επιστημονικό πρόγραμμα του Συνεδρίου θα περιλαμβάνει εργασίες δύο κατηγοριών κατανεμημένες σε τέσσερις θεματικές ενότητες. Οι εργασίες της πρώτης κατηγορίας θα αφορούν στη σύγχρονη διδασκαλία συy-ιεκριμένων θεμάτων των Μαθηματικών στις διάφορες βαθμίδες της

εκπαίδευσης. Οι εργασίες της δεύτερης κατηγορίας θα αφορούν στον προβληματισμό μας γύρω από το Αναλυτικό Πρόγραμμα, τη ρεαλιστική αντιμετώπιση της εφαρμογής του , το κατά πόσο επιτυγχάνει τους σκοπούς του κ.λ.π.

Για παράδειγμα, είναι γνωστό ότι ένα μεγάλο πρόβλημα που αντιμετωπίζουν και οι τρε ίς εκπαιδευυιές βαθμίδες είναι ο βαθμός ανάπτυξης της μαθηματικής σκέψης, καταρχή, και οι μαθηματικές γνώσεις των νεοεισερχομένων σε κάθε βαθμίδα, όπως αυτά προετοιμάσθηκαν από την προηγούμενη εκπαιδευτική βαθμίδα. Έτσι, π.χ., μπορεί οι νεοεισαγόμενοι στην τριτοβάθμια εκπαίδευση (σε σχολές που απαιτούν μαθηματικές γνώσεις) να είναι σε θέση να υπολογίσουν την παράγωγο μιας συνάρτησης ή να βρούνε το όριό της σε κάποιο σημείο με <<ειδικές συνταγές>> από το σχολείο ή τα φροντιστήρια, αλλά αδυνατούν να κατανοήσουν το τι σημαίνει και τι μας χρειάζεται, το αν μια συνάρτηση συγκλίνει ή όχι ή το αν έχει παράγωγο ή όχι κ.λ.π.

Οι εργασίες που θα υποβληθούν στις διάφορες ενότητες θα πρέπει να αφορούν κυρίως σε συγκεκριμένα μαθηματικά αντικείμενα και τη διδακτική τους προσέγγιση, παρά στο πώς γενικές αρχές της διδακτικής μπορούν να εφαρμοσθούν στη διδασχαλία των Μαθηματικών. Στο γενικό μέρος του προγράμματος προβλέπεται να υπάρξουν τρε ις ομιλίες, κατόπιν προσκλήσεως, από ειδικούς επιστήμονες εγνωσμένου χύρους. Από τις ομιλίες αυτές η πρώτη θα αφορά στην παρουσίαση του γενιχού θέματος του Συνεδρίου, η δεύτερη θα ε ίναι γενικότερου ενδιαφέροντος και θα αφορά στη συμβολή των Μαθηματιχών στους κοινωνιχούς αγώνες, παλαιότερα και σήμερα, χαι η τρίτη θα παρουσιάσει, με όσο το δυνατό πιο κατανοητό τρόπο, την εξέλιξη χαι την αξιοποίηση των σύγχρονων Μαθηματιχών των Εφαρμογών στην οιχονομία, στην τεχνολογία χαι στις βιοεπιστήμες με σχοπό την ενημέρωση των συναδέλφων πάνω στη σύγχρονη ανάπτυξη της επιστή μης μας.

Θ ε μ α τ ικ έ ς Ε ν ό τ η τ ε ς τ ο υ Σ υ ν ε δ ρ ί ο υ Α. Το Αναλυτικό Πρόγραμμα των Μαθηματικών στο Δημοτικό και η Διδακτική του Προσέγγιση. (Περιεχόμενο, Δυσκολίες, Προβληματισμοί) Στην ενότητα αυτή θα περιληφθούν εργασίες που θα αφορούν σε προβλήματα που παρουσιάζονται κατά τη διδασκαλία των Μαθηματικών στα παιδιά του Δημοτικού, στην χαταλληλότητα του περιεχομένου του υπάρχοντος, για τη βαθμίδα αυτή, Αναλυτικού Προγράμματος, στον τρόπο παρουσίασης συγκεκριμένων μαθηματικών εννοιών στο Δημοτικό, σε τρόπους ανάπτυξης της μαθηματικής κρίσης και της δεξιοτεχνίας χρήσεως των μαθηματικών εργαλείων κ.λ.π. Θα έχει ιδιαίτερο ενδιαφέρον να υποβληθούν στην ενότητα αυτή εργασίες από διδάσχοντες της πρωτοβάθμιας εκπαίδευσης. Β. Το Αναλυτικό Πρόγραμμα των Μαθηματικών στο Γυμνάσω και η Διδακτική του Προσέγγιση. (Περιεχόμενο, Δυσκολίες, Προβληματισμοί) Οι εργασίες που θα υποβληθούν στην ενότητα αυτιj θα πρέπει να αφορούν στον τρόπο διδασκαλίας συγκεκριμένων θεμάτων των Μαθηματικών, όπως π.χ. η παρουσίαση μαθηματικών προτάσεων μέσω γεωμετρικών αναπαραστάσεων, οι δυσκολίες στην κατανόηση της έννοιας του πραγματιχσύ αριθμού, η εισαγωγή της έννοιας της συνάρτησης και η παρουσία της στην καθημερινή ζωή κ.α Επίσης στο κατά πόσο επιτυγχάνονται οι στόχοι του υπάρχοντος, για τη βαθμίδα αυτή, Αναλυτικού Προγράμματος, όσον αφορά την εισαγωγή συγκεκριμένων μαθηματικών εννοιών, την ανάπτυξη της μαθηματικής κρίσης στην ηλικία αυτή, την εξάσκηm] των μαθητών σε μαθηματικές δεξιότητες κ.λ.π. Γ. Το Αναλυτικό Πρόγραμμα των Μαθηματικών στο Λύκειο και η Διδακτική του Προσέγγιση. Το Λύκεw ως αυτοτελής βαθμίδα εκπαίδευσης και ως

σχολείο προετοιμασίας για τις εισαγωγικές εξετάσεις της Τριτοβάθμιας Εκπαίδευσης. (Περιεχόμενο, Δυσκολίες, Προβληματισμοί) Όπως και στην προηγούμενη ενότητα, οι εργασίες που θα υποβληθούν στην ενότητα αυτή θα πρέπει να αφορούν στον τρόπο διδασχαλίας συγκεκριμένων θεμάτων των Μαθηματικών του Λυχείου, όπως π.χ. η εισαγωγή της έννοιας του ορίου ή η εισαγωγή της έννοιας της παραγώγου ως ρυθμού (ταχύτητας) εξέλιξης των φαινομένων, η παρουσίαση μαθηματιχών προτάσεων μέσω γεωμετρχών αναπαραστάσεων, κ. α. Επίσης, στο χατά πόσο επιτυγχάνονται οι στόχοι του υπάρχοντος, για τη βαθμίδα αυτή, Αναλυτιχού Προγράμματος, όσον αφορά στην εισαγωγή συγχεχριμένων μαθηματικών εννοιών, στην ανάπτυξη της μαθηματιχής χρίσης στην ηλικία αυτή, στην εξάσχηση των μαθητών σε μαθηματικές δεξιότητες κ.λ.π. Τέλος, η ενότητα αυτή θα περιλάβει εργασίες που θα αφορούν στον τρόπο και στο περιεχόμενο της διδασ-ιαλίας, ώστε το Λύκειο να λειτουργεί, τουλάχιστον όσον αφορά τα Μαθηματικά, ως αυτοτελής βα(tιίδα και όχι αποκλειστικά ως <<Φροντιστήριο» για την τριτοβάθμια εκπαίδευm]. Δ. Διασύνδεση των Μαθηματικών προγραμμάτων των τριών εκπαιδευτικών βαθμίδων Στην ενότητα αυτή θα περιληφθούν εργασίες που αφορούν στη διασύνδεση και εξάρτηση των μαθηματιχών ΠρJγραμμάτων των τριών βαθμίδων, στην προετοιμασία των μαθητών, όσον αφορά τα Μαθηματιχά, για την είσοδό τους στην επόμενη βαθμίδα κ.λ.π. Όπως αναφέρθηκε χαι παραπάνω, ενδιαφέρουν ιδιαίτερα παρουσιάσεις που αφορούν στην κατάσταση χαι τις σύγχρονες απαιτήσεις, ως προς τις μαθηματιχές γνώσεις χαι τη μαθηματική χρίση χαι ιχανότητα των νεοεισερχομένων στη δευτεροβάθμια (α. Γυμνάσιο, β. Λύχειο) χαι την τριτοβάθμια α.;ταίδευση.

Σ τ ρ ο γ γ υ λά Τ ρ α π έ ζ ια Α. <<Οι απαιτήσεις της Τριτοβάθμιας Εκπαίδευσης και η ικανότητα των νεοεισερχόμενων φοιτητών-σπουδαστών να τις αντιμετωπίσουν» Παρουσίαση των απαιτήσεων της Τριτοβάθμιας Εκπαίδευσης, σχολιασμός και συζήτηση επί των προτάσεων της τρίτης θεματικής ενότηrας και εξαγωγή, ε ι δυνατόν. χάποιων συμπερασμάτων. Β. <<Απόψεις για μια πρόταση Αναλυτικού Προγράμματος για το Λύκεw» Για την προετοιμασία του τραπεζιού αυτού προτείνεται να σταλούν στα παραρτήματα οι θέσεις του Ερευνητικού Κέντρου Αξιολόγησης και Επιμόρφωσι; της Ε.Μ.Ε. για το πρόγραμμα του Λυκείου και να ζητηθεί από αυτά να οργανώσουν ημερίδα συζήτησης και προτάσεων πάνω στο θέμα μέσα στον Σεπτέμβριο 1j Οκτώβριο του 2004, ίσως και με την παρουσία αντιπροσώπου του Δ.Σ. της Ε.Μ.Ε. Στη συνέχεια, οι εκπρόσωποι των παραρτημάτων να παρουσιάσουν τα συμπεράσματα των ημερίδων αυτών στο παραπάνω στρογγυλό τραπέζι και να γίνει προσπάθεια, για να τεθούν οι αρχές διαμόρφωσης μιας τελικής πρότασης προς το ΥΠΕΠΘ.

ΣΗΜΑΝΊΙΚΗ HMEPOMHNIA 15 ΣεΠτεμβρίου 2004 Τα πλ1jρη κείμενα των εργασιών (μέχρι 10 σελίδες) θα πρέπει να υποβληθούν μέχρι τις 15 Σεπτεμβρίου 2004 στην παρακάτω διεύθυνση:

Ελληνική Μαθηματική Εταιρεία (για το 21ο ΣυνεΟρw των Τροοjλων) Πανεπιστημίου 34 - 106 79 Αθήνα

και στην Ηλεκτρονική Διεύθυνση: e-maίl: [email protected]

Page 83: Ευκλειδης Β 51

Εκδόσεις της ΕΛΛΗΝΙΚΗΣ ΜΑΘΗΜΑΤΙΚΗΣ ΕΤΑΙΡΕΙΑΣ

���![�ης�,

\=10\01 ω .. ΙΙ Ι fι.Ι Ι \ Ι υ \ 1 111(1\ I I'] Ω ' \lt l t �lt\1\! \�11 �'' tltl'"""'"l"

I •Ht"I�H .... ,Ιf .. ΑΥιιι;JΙ Hλlt'tt• � ......... .......... ...... & ... •

ΕΥΚΛΕΙΔΗΣ γ ·

E U C L ι D E S γ ·

.,\ ΗΙ'\11\11 \I\HH\1\ fltol RIK \101 1'\ΦΙ \

Ι/ ι- ι. ι Τ \ Ι ι Ε ι ι< ο ι• ι.. t \

R ,_ _.. ,. ι Η ι· ιι n '

\-I ι r ι ι f Η ι ι ι f ' ' , , ,�

( -. lf O H Ιt t I I( � ' Y .J

rrη,"" "" ' ' 4 " ιο ,,,.,.,. ι -., " .. �..... ' • ll "

Η \'11(1 \11<'\lt..Ιt� \I\:�111\\IIΙ<.tt�t 1'\\tH\'-0\1.1 rτι'\

Σ·rnιχι-�rι:ΜΗΣ Γι:ΩΜΕΤJ>fΛ \ηΟ ANftΠ.r>tf ΣΚΟΠΙλ.

LOUIS BI�ND

ΜΑθΗΜΑτJΚΗ ΑΝΑΛVΣΗ

ΕυκλεlδηςΑ': Τεύχος 2 ευρώ Διεθνής Μαθηματικtς Ολυμπιόδες 1 1ου Παvελλη\iου Συνεδρίου 17 ευρώ (Μετρικό · Διόmρα) 20 ευρώ Συνδρομή (4 τεύχη) 10 ευρώ 1959 • 1999: 20 ευρώ 14ou Παvελλη\iου Συνεδρίου 17 ευρώ ευρώΔιαλtξεις: Ο τόμος 5 ευρώ

(4 τεύχη + 2 ευρώ ταχυδρομικά) Βαλκανικtς Μαθηματικtς 15ou Παvελλη\iου Σιmφου 17 ευρώ Μαθηματική Ανόλυση Σχολεία: 8 ευρώ δρχ. Ολυμnιόδες 1984-2001 : 15 ευρώ 16ou ΓΙανελληνίου Σιmφου 17 ευρώ (Loυls Brand) 25 ευρώ

ΕυκλεlδηςΒ': Τεύχος 2,5 ευρώ Θtματα εξετόσεων στα Α.Ε.Ι

�: �

Σιινεδρίαι�ου 25 εuρώ Διαφορικtς Εξισώσεις

Συνδρομή (4 τεύχη) 12 ευρώ 1976 • 19θ9: 6 ευρώ 19ou π�·;Ί•ι":;: Συνεδρίου

25 ευρώ (Steρhenson) 10 ευρώ (4 τεύχη + 2 ευρώ ταχυδρομικά) Πρακτικό: ........ ι•� 25 ευρώ Ιστορlα Μαθηματικών Lorla

Σχολεία: 10 ευρώ 1 ου ΓΙaνFλληνίου Συνεδρίου & ευρώ 2Οοο ΠανελληWJυ Συνεδρίου 25 ευρώ (4'Ιόμοι) Α, Β, Γ Α• Γa ο tόμος 8 ευρώ ΕυκλείδηςΓ: Τεύχος 5 ευρώ 2ou Πaνrλλη\iου Συνεδρίου 8 ευρώ Συνtδριο Hermls '92 (Αγγλικό)

70 Χρόνια Ε.Μ.Ε. 4 ευρώ 25 ευρώ Συνδρομή (2 τεύχη) 10 ευρώ 3ou ΠCI\/Fλλrrνioυ Συνεδρίου 8 ευρώ Συνtδριο Hermls -94 (Αγγλικό) Ελληνική Μαθηματική Μαθημ. Επιθεώρηση: 4ooQou Γfcι\.fλλη\οiου ΣιΜ:δρίου θ ευρώ 2 τόμοι 0 τόμος 15 ευρώ Βιβλιοyραφlα 4 ευρώ

Τεύχος 5 ευρώ 6ou ΓΙaνFλληνίου Σuνεδρίου 8 ευρώ Γραμμική Άλγεβρα Στοιχειώδης Γεωμετρlα από Συνδρομή (2 τεύχη) 10 ευρώ 7ου ΓΙανελληνίου Συνεδρίου 8 ευρώ (Gr. Mυncres) 6 ευρώ Ανώτερη Σκοπιό 8 ευρώ

Αστρολόβος: Τεύχος 5 ευρώ θου Πανελληνίου Συνεδρίου 8 ευρώ ΗΡΩΝΟΣ ΑΛΕΞΑΝΔΡΕΩΣ Θεωρlα Αριθμών 17 ευρώ Συνδρομή (2 τεύχη) 10 ευρώ 9ου ΠCI\/Fλλrrνioυ Συνεδρίου 8 ευρώ (Ονόματα Γεωμετρικών όρων 10 Χρόνια 'ΈΥΚΛΕΙΔΗΣ"

Δελτίο (Bυlletln): Τεύχος 7,30 ευρώ 10ou ΓΙaνFλληνίου Συνεδρίου 17 ευρώ ΓΕΩΜΕτΡΙΚΑ) 20 ευρώ ΣΕΙΡΑ Ρ: (1990-1999) CD 25 ευρώ

Τα παλα ιότερα τεύχη όλων των εκδόσεων πωλούντα ι με τις τρέχουσες τι μές του 2004

Page 84: Ευκλειδης Β 51